Women's health exam 2 clinical 1

Pataasin ang iyong marka sa homework at exams ngayon gamit ang Quizwiz!

Infants sx of UTI

(1 mo - 2 years) o FUO often only ssx o Everything above plus: o Strong smelling urine o GI illness often masks bladder problems (colic, irritability, screaming) o Dx often delayed d/t lack of localizing ssx = highest risk of renal injury d/t delay in abx tx (3 day rule) **NO INFANT SHOULD BE OBSERVED FOR > 3 DAYS WITH FUO WITHOUT CHECKING URINE**

Adolescents sx of UTI

(12 - 18 years) 1. Classic adult sx 2. Dysuria, frequency, urgency, small voids and incomplete emptying 3. Gross hematuria 4. Nocturia 5. Foul smelling/cloudy urine 6. Suprapubic pain/pressure and lower ABD pain

What are the POSITIVE diagnostic signs indicating pregnancy?

(Positive Signs (diagnostic)) Fetal heart sounds Fetal movements felt by examiner Ultrasound

Neonates sx of UTI

(birth - 28 days): o Fever (FUO) = MC o Failure to thrive, poor feeding, jaundice (LC) o V/D o Likely have burning and increased frequency as well o Often found during workup for neonatal sepsis

Menopause Facts:

* #1 killer of women is heart disease * heart disease risk incr after menopause * women more likely to develop osteoporotic fx's than men (1 in 2 women will fx after menopause) * osteoporosis risk incr after menopause * bone loss is accelerated during first 4 years after menopause

Manifestations of Menopause

* bone loss (5-6 years accelerated bone loss after menopause starts; why women reach menopause earlier than men) * impaired balance & forearm fx * hyperlipidemia * incr risk CHD (esp after menopause) * depression * cognitive decline (after hormones decr) * decr cutaneous collagen * incr fat deposition & decr lean muscle mass (more issues w/ weight gain/obesity; exercise critical at this stage to help with lean body mass)

ADEs of Fluconazole

*Pregnant women* should use only topical "-azoles"; oral fluconazole is CONTRAINDICATED

How to be reasonably sure that a woman is not pregnant

- < 7 days after the start of normal menses - No intercourse since start of last menses - Correctly and consistently using a reliable method of contraception - < 7 days after a spontaneous or induced abortion - Within 4 weeks postpartum - Fully or nearly fully breastfeeding (>85% of feeds), amenorrheic, and <6 months postpartum

Dilation and Evacuation

- >14-<20 wks -Cervical Dilation: at 16wks, osmotic dilator w/Misoprostol prn -Vacuum x 2, forceps, currette -Paracervical block + Mod/deep IV sedation to 16wks; general anesthesia -Post Op Abx -Contraception: Immediate (to w/in 5d) -2-3 visits (after 16 wks)

What is Sunday start?

- Start COC, patch, ring on Sunday after menses begin • With standard use, monthly menses will occur during week, not over weekend, if using a hormone-free week monthly - Backup for 7 days

Menopause: definition

- abscence of periods for 12 months - mean age ~ 51 yrs - women live avg 30 yrs beyond - occurs 2 years earlier in smokers *always ask about sx: night sweats, hot flashes, sleep disturbances

Thromboembolic Disease and estrogen:

- absolute risk in nonusers: 1:10,000 - estrogen + progestin have 2-fold incr in risk (should screen for clotting factors/hx) - highest risk in first year of use - lower doses of estrogen have a lower risk - transdermal delivery decr the risk - should be screening routinely for clottign factors (don't-not cost effective)

Hot flashes treatment:

- associated with thermoregulatory dysfunction - initiated at level of hypothalamus by low estrogen levels - treat: * mild: non-pharmacologic * mod-severe: hormonal therapy or non hormonal therapies (gabapentin, SSRIs, SNRIs)

CVD and Estrogen:

- estrogen has positive effects on lipid panel - early observational studies demonstrated a decr risk of MI in patients receiving estrogen - RCTs showed incr risk in MI in patients taking estrogen+progesterone - women who initiate therapy 10 yrs after menopause may be at higher risk of CHD - estrogen therapy should not be initiated or continued for prevention of cardiac disease

Breast Cancer and estrogen therapy:

- estrogen+progesterone incr risk invasive breast cancer - risk incr with 5 or more years of use (hard to pick up breast cancer in mammogram when estrogen levels higher - cancers were more advanced and harder to diagnose - risk breast cancer returns to baseline after d/c

Staying healthy after menopause:

- evaluate risks for CHD & treat - choose healthy lifestyle-nutrition&exercise - maintain healthy weight - get mammogram - annually - get colonoscopy - at age 50 then Q 5yrs - get DEXA - ensure adequate calcium & vit D intake

management options

-A wide variety of therapeutic approaches have been recommended for symptoms of PMS and PMDD -Treat women individually and conservatively

Nuclear cortical scanning:

-AKA renal scintigraphy- flouroscopy 1.Dimercaptosuccinic acid (DMSA) injected by IV and kidney uptake measured 2-4 hours later 2. Areas of dec uptake in the cortex=damage -Detects acute and chronic renal scarring -Shows acute pyelonephritis o Low radiation exposure o Shows renal outline, but not collecting system 3. Used acutely, shows extent of renal parenchymal damage -Moderate and severe VUR will have a positive test result but Misses Grades I and II 4. Used to F/U abnormalities seen on US

Gonadotropin-releasing hormone agonists (GnRH)

-Another drug that suppresses ovulation found to have benefits -progestin (eg. Lupron-leuprolide) 3.75 mg IM monthly -maximum treatment period is 6 months, and add-back therapy has to be given to reduce hypogestrogenic side effects -Costly -Add-back may cause recurrence of symptoms

Meds and Effects on UI

-Anticholinergic SE such as first-gen antihistamines, tricyclics, antipsychotics: increase urinary retention, overflow incontinence -Opioids: urinary retention, overflow, fecal impaction -Alpha-adrenergic antagonists (prazosin, doxazosin, terazosin): relaxes internal urethral sphincter- desired effect on man with BPH.

GU infections

-BV: on microscopic exam: vaginal pH > 4.5, clue cells, positive whiff test, few WBCs. Flagyl 500mg BID x 7 days -Candidiasis: candida albicans- itching, burning, thick white to yellow curd-like dc, miconazole, fluconazole -Chancroid: caused by h. ducreyi, painless genital ulcer, treat with azithromycin 1g bomb -Herpes simplex: HHV-2, painful ulcerated lesions with lymphadenopathy, tx with acyclovir 400mg PO TID x 7-10 days -Lymphogranuloma venereum: vesicular or ulcerative lesion, caused by LI-3 serovar of c. trachomatis: doxycycline 100mg PO BID x 21 days -nongonococcal urethritis and cervicitis: c. trachomatis: Azithromycin 1g PO bomb -gonococcal urethritis and vaginitis: caused by n. gonorrhoeae, single dose tx for uncomplicated infection- cefixime 400mg PO, ceftriaxone 125mg IM -PID: caused by n. gonorrhoeae, c. tachomatis, e. coli, mycoplasma, take ofloxacin 400mg PO BID with flagyl 500mg BID x 14 days. -Trichomoniasis: caused by t. vaginalis, *cervical petechial hemorrhages (strawberry spots) in 30%. On Exam, motile organisms and WBCs. Take Flagyl 2g Bomb. -Genital warts: caused by HPV, patient applied tx included podofilox 0.5% or imiquimod 5% cream.

Empirically Based (based on evidence since 2000) Med Abortion

-Buccal Misoprostol -Gestational age: Up to 63 days after LMP -Ultrasound at initial visit -min visits: 2 -Mifepristone 200 mg (swallowed in office) -Misoprostol 800 mcg. , Buccal ,At home 24-48 hours after taking mifepristone -f/u visit:-14 d , sono -Contraception: day after miso or 7d after mife IUC: after sono confirms complete

Psychoactive drugs

-Buspirone = mild anxiety

Hale's Lactation Risk Categories

-Categories L1-L5. L1 safest, L2 ok (SSRIs)- no risks noted, L5 containdicated (radiation, cocaine).

Tx of Neisseria gonorrhoeae endocervicitis

-Ceftriaxone, 125 mg intramuscularly (single dose), or Ciprofloxacin, 500 mg orally (single dose)a, or

STIs

-Chlamydia: gram- bacteria. Many women are asymptomatic with mucopurulent vaginal discharge, dysuria, dyspareunia and postcoital bleeding. -C. trachomatis: mucopurlent dc with adherent to friable cervix. Need tx that works against intracellular organisms: doxycycline, erythromycin, azithromycin. Zpack as one dose highly effective. -N. gonorrhoeae: gram - diplococcus. Short incubation- 1-5 days. Men asymptomatic. Women have dysuria with milky to purulent, blood-tinged dc. Rectal infection with proctitis seen. Need activity against beta-lactamase: fluoroquinolones, ceftriaxone and cefixime.

Ectopic pregnancy

-Classic triad: abd pain, vag bleeding and adnexal mass is present in <50% of women. Dx includes beta-hCG level. Neg result r/o dx. Usually, serum quant beta-hCG in ectopic pregnancy at weeks 6-10 (time most common for presentation) is 1000-6000 IU/m. This compares with =>40,000 IU/m for viable pregnancy. With + beta-hCG => 1000 IU/m, a gestational sac schould be identifiable within uterus on transvaginal sonogram. Presence r/o dx. 20-30% will have nondx US, so normal US does not r/o condition. Level of <15ng/mL serum progesterone is seen in only 11% of normal intrauterine pregnancies and most ectopic/inevitable abortions.

When should diapers be used?

-Cognitively and functionally impaired patients -Refractory/Intractable incontinence

What drugs can cause overflow incontinence?

-Drugs with anticholinergic or antimuscarinic activity

Title X

-Federal program to decrease disparities -Enacted 1970 to fund clinics providing preventive, family planning services free of charge or sliding scale -Required to provide other preventive services as well -Helped to prevent millions of pregnancies over 2 decades

Affordable Care Act

-Federal program to decrease disparities -Potential to extend contraceptive coverage to 30 million women -Expansion of medicaid -Private insurers required to fully cover BC

Manual Vaccuum Aspiration or Electromechanical Suction

-First Trimester: to 14 wks -Cervical Dilation: at > 12 wks w/Misoprostol (pv, po) -Vacuum x1 -Local analgesia only or Mild sedation or mod/ deep IV sedation -Post Op Abx -Contraception: immediate (to w/in 5d) -2 visits

Why study unintended pregnancy?

-Fundamental measure of a population's reproductive health -Helps us to know who is in need of improved services -Helps us to know what kind of services are needed -All healthcare providers need to be prepared to provide options counseling for patients experiencing unintended pregnancy

Treatment for Chlamydia

-If *only chlamydia* is found: doxycycline or azithromycin -If co-infection: ceph + azithromycin

What is the initial therapy for urge incontinence?

-Kegels

Medical Abortion- Contraindications

-Known Allergies to Meds -Unable to follow instructions -Chronic Renal Insufficiency -Severe Anemia -Hemorrhagic Disorder -Long Term Corticosteroid Use -IUC in place ->9wks EGA -Unable to follow-up -Surgical contraindicated

Menopause

-LH and FSH levels increase as pituitary gland sends out abundance of these in attempt to induce ovulation, with ovaries failing to respond. Estrogen is reduced with androgens (testosterone). Hot flashes occur d/t LH surge occur in 80%. HT is needed. Estrogen 0.325-0.625 mg/day reduces by 50-75%. -If she cannot use HT- low-dose antidepressants can help. Venlafaxine (Effexor) 37.5mg/day, Zoloft 25-50mg/day, Paxil 10-20mg. Vit E 800IU/day helps reduce hot flashes. Vitamin B6, 25mg/day helps with remenstrual dysphoria. -When menstruating but has many symptoms OC can help. She will continue to have monthly bleeding, so if she has hot flashes during placebo week- she may be in menopause. Check LH/FSH on day 7 of pill-free interval to confirm. Ratio of > 1 and estradiol < 25pg/mL indicates menopause. COCs have 3-4x estrogen as HT. -Androgen supplementation may help in women with low libido and hotflashes. Chances such as acne, hirsutism, alopecia and clitoral enlargement. -Estrogen deficiency risk factor for osteroporosis. Calcium supplements needed. Venous TE events and invasive breast ca with HT, other meds such as bisphosphonates and salmon calcitonin (Miacalcin) should be used.

Mifepristone

-Mifepristone: RU 486: "mife" Approved FDA 2000 -Antiprogesterone -Binds to progesterone receptor -Weak antiglucorticoid and antiandrogen -Softens and dilates the cervix -Causes decidual necrosis -Increases prostaglandin release

Pregnancy Needs

-Need 3-4 servings of dairy for adequate calcium intake. -Iron requirements increase in 2-3rd trimester. Elemental iron is given= in absence of IDA, 30mg/day, multiple-gestation pregnancy 60-100mg/day, and with IDA 200mg/day. One 325mg ferrous sulfate contains 65mg elemental iron, whereas most prenatals contain 30-65mg.

NSAIDS

-Non-steroidal anti-inflammatory drugs -for headaches and cramps -These drugs include ibuprofen and naprosyn

Diagnostic criteria of PMS

-ONE of the affective and somatic symptoms during the 5 days before menses in three prior menstrual cycles *Affective = depression, angry outbursts, irritability, anxiety, confsuion, social withdrawal *Somatic = breast tenderness, abd bloating, headache, swelling of extremities -These symptoms are relieved within 4 days of the onset of menses without recurrence until at least cycle day 12 -The symptoms are present in the absence of any pharmacologic therapy, hormone ingestion, or drug or alcohol use -The patient suffers from identifiable dysfunction in social or economic performance

FDA Approved Med Abortion

-Oral Misoprostol -Gestational age: Up to 49 days after LMP -Ultrasound at initial visit -min visits: 3 -Mifepristone 600 mg. (swallowed in the office) -Misoprostol 400 mcg. Oral In the office 48 hours after taking mifepristone -f/u visit: 7-14 d sono -Contraception: day after miso or 7d after mife IUC: after sono confirms complete

Pain with Medical Abortion

-PAINFUL/INDUCED MISCARRIAGE -Analgesia only pain option

Premenstrual dysphoric disorder

-PMDD represents a more severe disorder with marked behavioral and emotional symptoms

Non-oral contraceptives

-Patch (OrthoEvra) and ring (NuvaRing) contain estrogen and progestin. New patch/ring needed q3wks. AE and contraindications same as COC. -DMPA (Depo-Provera): given q90days very reliable (99.7%). Suited for women who don't want to be pregnant for at least 18 months (resumption to fertility is 6-12 months). When given within first few days of menses, contraceptive effect is immediate. When started 5 days after onset of menses, use backup method for 1 week. May be started immediately post-partum in women not breastfeeding and at 3-6 weeks for those who are. Earlier use may diminish quanitity (not quality) of milk. Irregular bleeding in first few months common- minimize with ibuprofen TID for 3-5 days. After 1 year, 30-50% have amenorrhea. Bone Density may be reduced- calcium 1000-1500mg/day recommended. -IUDs: copper-containing ParaGard T are effective. Not understood- not likely abortifacients. Not widely used d/t increased bleeding and infections. -Mirena is levonorgestrel-containing intrauterine system of drug delivery, produces marked endometrial atrophy. 50% amenorrheic at end of 1 year of use.

What are the mainstays of treatment for stress incontinence?

-Pelvic Floor exercises -Hormone replacement

What is the Bonney (Marshall) Test?

-Place two fingers in lateral vagina and ask patient to cough -If urine leakage occurs on coughing, -but is prevented by finger pressure, -then significant pelvic laxity

Syphilis

-Primary: manifestations in 2-4 weeks- painless ulcer, palpable inguinal nodes, spontaneously healing lesion -Secondary: generalized rash, arthralgia, lymphadenopathy -Tertiary: gumma -caused by spirochete treponema pallidum

Misoprostol (Miso)

-Prostaglandin analogue -Binds to myometrial cells -Causes Contractions (also used for prevention of gastric ulcers)

Meds in pregnancy

-SSRI with longest T1/2: Fluoxetine -Asthma controller tx: ICS. Use steroids for flare. -Sertraline: D -Clonazepam: C -Bupropion: B -Tricyclics: D/ Benzos -Erythromycin: B with penicillins. Cipro is cat B until 36 wks. -SSRIs: C

Pain with Surgical Abortion

-Several Pain management options

Reproductive Health Care Outlook- One Key Question

-Shift from funding/payment for volume of visits to quality of care -Attempt in Oregon to promote One Key Question as quality metric: -Makes reproductive health core component of primary care -Decrease unintended pregnancies, increase healthier pregnancies -Potential for national metric Ask: observe effects on preconception and contraceptive care

Contraception and Unintended Pregnancy

-Slightly more than half of unintended pregnancies occur among women who used no method of contraception during the month in which they conceived -more than four in 10 occur among women who used a method inconsistently or incorrectly. Only one in 20 are attributable to method failure.

Oral contraceptive

-Suppression of ovulation -Monophasic type aids breast pain, bloating, acne and appetite -Extended use reduces propensity for monthly symptoms

Abortion facts

-There were roughly 1.2 million abortions performed in in the US in 2008 - Approximately 2% of American women aged 15-44 had an abortion in 2008. -1 in 10 women will have an abortion by age 20, 1 in 4 by age 30 and 3 in 10 by age 45. -Rates of abortion are highest in in the developing world, in which the vast majority of the worlds unsafe abortions occur. -Eastern Europe has the highest rate of abortion, Western Europe the lowest. The US has the highest abortion rate of comparable developed countries.

Intravaginal Drug Administration Considerations

-Topical effects desired but systemic exposure possible -Systemic effect is the goal via vaginal transmucosal absorption; local drug effect possible

Total public expenditures from these births

-Total public expenditures for births resulting from unintended pregnancies: $11.1 billion. -Federal: $6.5 billion -State: $4.6 billion

What can cause overflow incontinence?

-Tumors, etc -atonic or flaccid bladder that has lost its neuronal innervation

Unintended Pregnancy: Costs (2006)

-Two-thirds of the 1.6 million births resulting from unintended pregnancies were paid for by public insurance programs, primarily Medicaid.

The best way to initially assess incontinence is by a combination of:

-Urinalysis -Culture and Sensitivity -Residual Volume Determination -Clinical Symptoms

Assessment

-Urine Pregnancy Test: detects if bHCG is above 25 -Estimated gestational age based on LMP -Bimanual exam and/or TVUS -r/o ectopic pregnancy

Iatrogenic causes of AUB

. Glucocorticoids, tamoxifen, anticoagulants . Ginseng, ginkgo, soy products associated with alterations in estrogen levels affecting clotting . Frequently observed in women taking contraceptives the 1st time

Will provide essential clues for making a diagnosis

. Symptom- tracking charts & calendars . Menstrual & health history data

Key criteria for a diagnosis of PMS

. Symptoms consistent with PMS . Consistent occurrence of symptoms only during the luteal phase of the menstrual cycle . Negative impact of the symptoms on some facets of the woman's life . Exclusion of other diagnosis

Laparoscopy

. The outside surfaces of the uterus, tubes, and ovaries can be observed via a laparoscope, which is inserted into the abdomen through the umbilicus . Pelvic organs are examined for any abnormalities, including structural alterations, endometriosis, or pelvic adhesions . Can be used not only for diagnosis of endometriosis and pelvic adhesions, but also for treatment

4 symptom clusters that accounted for much of the variance in women's experience of premenstrual phase

. Turmoil: depression, anger, tearfulness, loneliness, impatience . Fluid retention: weight gain, abdominal bloating, skin disorders . Somatic symptoms: nausea, lowered desire to move,decreased sexual desire, aches & pains . Arousal: bursts of energy or activity, cravings for food, increased sense of well being

ABNORMAL UTERINE BLEEDING

...

INFERTILITY

...

MENOPAUSE

...

Menopause occur

1 1/2 years earlier among smokers versus non-smokers

Mutations in EPCAM found in what percent of families with LS?

1-3%

other commonly used antibiotics

1. Ampicillin: IV/IM only 2. Used with Claforan (Cefotaxime) or Gentamicin 3. Amoxicillin (Amoxil, Trimox) 4. Amoxicillin-Clavulanate (Augmentin) 5. Trimethoprim sulfamethoxazole (Bactrim) -Caution: allergic reactions common 6. Cephalexin (Keflex): 1sr gen. cephalosporin o ALL OF THESE ANTIBIOTICS HAVE KNOWN RESISTANCE

What is the relation between estrogen and the cardiovascular system

1. Estrogen acts as a vasodilator via NO (nitric oxide) and serves to vasodilate BV= lower BP of premenopausal women 2. Vasomotor stability 3. Decreased formation of athersclerotic lesions 4. Stimulates production of endothelial cells-- angiogenesis 5. Increased the synthesis of coagulation factor-- Increased blotting risk in women

most suspicious for a pediatric UTI

1. FUO and sustained ABD sx without explanation >>EXAMINE URINE and get CULTURE>> 3 day rule 2. Children with known GU abnormalities >> suspect UTI early with FUO or other sx

Urine culture

1. GOLD STANDARD 2. Classic criteria for + UTI a. >100,000 colony-forming units/mL (CFU) of a single organism on CCMS sample b. >50,000 CFU/mL from urethral cath c. >1000 CFU/mL from suprapubic aspiration of gram positive pathogens d. Any gram negative growth from suprapubic aspiration is significant

differential diagnosis for pediatric UTIs.

1. Gastroenteritis 2. Appendicitis 3. Bacteremia/sepsis 4. Pinworms 5. Renal calculi 6. Urinary obstruction 7. Urethral stricture 8. Male urethritis 9. Vaginitis/Vulvovaginits 10. Cervicitis 11. Prostatitis or epididymitis in males 12. Hypersensitivity to soaps/detergents 13. Neoplasm

Sx of menopause

1. Genitourinary atrophy: painful intercourse, dysuria, genitourinary infections 2. Vasomotor instability: hot flashes, insomnia 3. Osteoporosis 4. Increased risk of CVD

consideration for pediatric UTIs presentations by age

1. Presentation varies greatly by age 2. Infants and young children- few specific sx = easily missed (d/t inability to communicate) 3. Older pediatric pts -more likely to have sx similar to adults

Progestin-only EC (Plan B®, Plan B One-Step®, My Way®, Next Choice®), levonorgestrel _____ mg total dose Take dose within 72h but also effective up to 120 h post-intercourse, with improved efficacy when taken earlier in this time frame For two-tablet regimens, take both pills in single dose or 1 pill with a 2d dose 12 h later. Single dose option also available.

1.5mg

What are the risk factors for Breast Cancer

1.Age 2. Gender 3. Early Menarche- before 12 4. Late menopause- after 55 5. Hormone therapy 6. Nulliparity 7. first pregnancy >30 years old 8. High dose radiation 9. Family history 10. Obesity

indication for serum labs for a pediatric UTIs

1.CBC and Basic Metabolic Panel are indicated for all children with fever and presumed to have pyelonephritis. 2. blood cx for all febrile infants dt 1/3 of infants with UTI have bacteremia or meningitis. -Blood cx: for older pt who are clinically ill, toxic, or severely febrile.

The 4 types of breast CA

1.Carcinoma in situ 2. Invasive breast cancer 3 Paget's disease 4 Inflammatory carcinoma

Copper-containing IUD (Cu-IUD) Approved to remain in place for ____ years , likely effective up to 20 years.

10

Length of Surgical Abortion

10 MIN-30 MIN PROCEDURE Bleeding: mild for 1 wk post procedure

How many women visit doctor from vaginitis?

10 million/yr

Microsatellie Instability measures high in

10-15% of sporatic colorectal tumors, 1-23% of sporadic endometrial tumors, and 90-95% of LS tumors. Note not all high MSIs are Lynch Syndrome

What is the OTC dose of phenazopyridine during UTI?

100 mg TID x 2 days

How common is urinary incontinence amongst the elderly population?

12%

Time for a D and E:

14-24 weeks

Upon what levels of BhCG can a gestational sac be seen in a sono?

1500-2000

Transdermal Patch HRT Estrogen Regimen

17B Estradiol Advantage; lower risk of clots Disadvantage; Smaller improvement on lipid profile.

Woman's lifetime risk of being diagnosed with breast cancer

1:8

Time Frame of Imaging

1Typically delayed 3 to 6 weeks after infection -OP f-up with pediatric urologist -Exception: urinary obstruction or non-response to tx

Perimenopausal period

2 to 8 years prior to the last menstrual period and for the 12 months of amenorrhea preceding menopause . Fewer ovarian follicles develop in each menstrual cycle . The follicles that do develop are less responsive to follicle stimulating hormone, & the ovaries produce less estradiol, progesterone, and androgens . Thus the usual negative feedback effect from elevated estrogen and progesterone levels on hypothalamic production of GnRh is lost, and the anterior pituitary production of FSH and LH continues . Eventually ovarian follicle production stops, estrogen and progesterone levels remain low, FSH and LH levels remain high and menstruation ceases

Highest age group percentage of abortion

20-24 y/o

What age group has the highest RATE of unintended pregnancies?

20-29 year olds

What is the prescription dose of phenazopyridine during UTI?

200 mg po TID x 2 days.

Ethnic disparities in unintended pregnancy

2001- Black 2006- Hispanic Continues to Shift

normal menstrual cycle occurs every __ - __ days, lasting __ - __ days.

21-35; 3-5

Up to when can someone have a surgical abortion?

24 weeks

every individual with low bone density should have this lab work done (one)

25-hydroxy vitamin D (<20 = vitamin D deficiency) (20-30 = risk for hip fracture)

normal adult menstrual cycle: mean number of days in cycle?

28

Etonogestrel (Nexplanon®, Implanon®) - Proves daily constant release of low dose progestin - Effective for at least ___ years

3

In =>95% of women using progestin-only EC, next menstrual period occurs within ___ weeks of taking medication. Obtain pregnancy test if menses delayed beyond 1 week of anticipated date of onset.

3

UPA and LNG ECPs - Similar effectiveness when taken within ___ days after unprotected intercourse in many circumstances

3

Highest % Abortions by Education Level

39.5% have had some college

Duration of menses in the normal adult menstrual cycle?

4 days

Normal vaginal pH

4.0 (acidic)

menopause before this age is considered premature ovarian failure

40

Natural menopause occurs for most women between the ages of

48 and 55 years

Mirena®- Approved to remain in place for ___ years.

5

changes during menopause she did not last longer than ___ .

5 years

average age of menopause in the United States

51

What percentage of pregnancies are unintended (2006) ?

51% (6.7 million in 2006)

Women aged _____ years and older, _____ of breast masses malignant

55 and 85%

Highest % abortions by weeks gestation

59% occur by 9 weeks gestation

Chlamydia treatment recommendations

7 day abstinence after therapy. Test of cure is not recommended at this time.

Vulvovaginal Candidiasis (VVC) An estimated ...... of women experience at least one episode of vulvovaginal candidiasis (VVC) during their lifetimes.

75%

What percentage of women will have at least one vaginitis infection?

75%; 40% will have two and 5% will have recurrent infections

Up to when can someone have a medical abortion?

9 weeks

What percentage of women know about the 3 main types of vaginitis?

95%-yeast 37%-trichomonas 34%-bacterial vaginitis

treatment is indicated for all women with T-scores of ___ 10 for all patients who have had fragility fractures

<-2.5 (osteoporosis)

polymenorrhea

<21 days between periods

Menstrual Diagnoses

> Dysmenorrhea Premenstrual conditions: > Premenstrual syndrome (PMS) > Premenstrual dysphoric disorder (PMDD) > Premenstrual magnification (PMM)

BP changes in preeclampsia include increases:

>30/15 or reading of >140's/90's. With minimal proteinuria nd pathologic edema, with presentation after 20th wk. Other problems include RUQ pain, NV. Can progress to hemolysis or HELLP. *Gestational HTN is HTN dx after 20th wk but resolves within 6 weeks postpartum. Preeclampsia is HTN after 20th wk with proteinuria (>300mg in 24 hrs) usually accompanied by edema.

Contraindications to oral contraceptives

>35 y/o female smoker, history of thromboembolism, stroke, or history of estrogen dependent tumors

A 23-year-old woman is breastfeeding her healthy newborn. She wishes to use hormonal contraception. Which of the following represents an unacceptable choice? A. Combined oral contraception initiated at 2 weeks B. Progestin-only oral contraception initiated at 3 weeks C. Medroxyprogesterone acetate (Depo-Provera®) given day 1 postpartum D. CU-containing IUD (ParaGard®)

A

cystitis 2

A 4-day course of an oral antibiotic agent is recommended for the treatment of cystitis. If the clinical response is not satisfactory after 2-3 days, alter therapy on the basis of antibiotic susceptibility.

In women of reproductive age, the most common cause of a bleeding pattern that is suddenly different from the established menstrual pattern is

A complication of pregnancy, including threatened or incomplete abortion, ectopic, retained products of conception, & gestational trophoblastic disease (latter 2 the least) * always exclude pregnancy first!

Placenta Previa

A multipara who is in the late second to third trimester complains of new onset of painless vaginal bleeding that is worsened by intercourse. Blood is bright red in color. Uterus is soft and nontender. If cervix is not dilated, treatment is strict bed rest. Intravenous magnesium sulfate if there is uterine cramping. Uterus will usually reimplant itself if mild. Any vaginal or rectal insertion or stimulation is an absolute contraindication (can precipitate hemorrhage). If cervix is dilated or if hemorrhaging, fetus is delivered by C-section. Severe cases cause hemorrhage; fetus must be delivered to save mother's life.

Danazol

A synthetic androgen Suppresses ovulation and mastalgia Side effects of masculinzation

The risk for developing cancer increases as women age.

A woman's risk for developing breast cancer is approximately 1 out of 30 at the age of 50 . 1 out of 9 for a woman into her 80s . Lifetime risk for a woman to develop breast cancer is about 1 in 7

What are the PRESUMPTIVE (Subjective) and symptoms of pregnancy?

Absence of menses Nausea, vomiting, "morning sickness" Urinary frequency Breast tenderness, pigmentation, discharge "quickening" Pigmentation of the skin Enlargement of the uterus

What are the advantages to estrogen effects on lipids. Disadvantages?

Advantage: Decreased risk of CVD compared to men Disadvantage: Increased incidence of gallstones (eliminating the LDL via bile)

Category C Drugs

Adverse effects seen in animal studies. No human data available. ■ Sulfa drugs - Category C in third trimester because it can cause hyperbilirubinemia. Sulfa drugs displace bilirubin from albumin. High blood levels of unconjugated bilirubin can cross the blood-brain barrier and cause brain damage (mental retardation, seizures, deafness, etc.). ■ Trimethoprim-sulfamethoxazole (e.g., Bactrim DS, Septra) - Clarithryomycin (Biaxin) is the only category C macrolide antibiotic. Avoid using in pregnant women. Consult with physician before using category C drugs during pregnancy. ■ NSAIDs - Avoid using in the third trimester (especially the last 2 weeks) because it blocks prostaglandins. Category B or C (depends on type of NSAID). - Ibuprofen (Advil) is category B (fi rst and second trimesters) and category D (third trimester).

Vaginal dryness was more common among

African-American and Hispanic women . Hispanic women were more likely to report urine leakage, forgetfulness, and heart pounding or racing than Caucasians

The severity of vasomotor symptoms was highest among

African-American women

Diagnosis of anovulatory uterine bleeding or dysfunctional uterine bleeding (DUB) is made

After excluding all other causes of abnormal bleeding

etilogy

Almost all UTIs are ascending in origin. Most infections begin in the bladder; from there, pathogens can spread up the urinary tract to the kidneys (pyelonephritis) and possibly to the bloodstream (bacteremia). Most episodes of UTI during the first year of life are pyelonephritis.

Spontaneous Abortions

Also known as a miscarriage. Spontaneous loss of the fetus before it is viable <20 weeks

RhoGAM

Also known as anti-D immune globulin. It is made from pooled IgG antibodies against Rh (rhesus) factor. It is an immunoglobulin that helps prevent maternal isoimmunization (self-immunization) or alloimmunization (immunity against another individual of the same species). If RhoGAM is not given to Rh-negative pregnant women, it will result in fetal hemolysis and fetal anemia in her future pregnancies.

Absence of menses

Amenorrhea . Primary- no menses by age 14 with delay in maturation of secondary sexual characteristics or no menses by age 16 with secondary sexual characteristics . Secondary- absence of periods for a length of time equivalent to a total of at least 3 of the previous cycle intervals or 6 months of amenorrhea

Name 3 types of Abnormal and Dysfunctional Uterine Bleeding is a sign of?

Amenorrhea, oligomenorrhea, menorrhagia.

What agents are safe to treat UTI in pregnant women?

Amoxicilin (near term is safe), nitrofurantoin, oral cephalosporine and fosfomycin.

Placenta Previa

An abnormally implanted placenta. The placenta implants too low either on top of cervix or on the cervical isthmus/neck. Most cases get better spontaneously (will reimplant itself). Some cases are asymptomatic. Higher risk if previous history of placenta previa or C-section, multiparas, older age, smoking, fibroids, or cocaine use. Classic Case A woman who is a multipara who is at the late second to third trimester of pregnancy complains of the sudden onset of bright red vaginal bleeding (light to heavy) accompanied by mild contractions. The uterus feels soft and is not tender. Treatment Plan ■ Refer to ED. ■ Avoid bimanual examination since palpation of the uterus may cause severe hemorrhage. ■ Abdominal ultrasound only. No intravaginal ultrasound. No rectal exams. ■ Avoid any vaginal/rectal sexual intercourse. ■ Bed rest. Close fetal and maternal monitoring. ■ If contractions, give magnesium sulfate (MgSO4) IV. If mild cases, pregnancy can be salvaged and the placenta will reimplant. Perform C-section if mother's life is in danger.

Diseases or syndromes causing insulin resistance, such as PCOS, increase circulating levels of insulin which in turn leads to

An elevation of androgen production and concomittant anovulation . Relationship between insulin & androgens is believed to be an underlying cause of PCOS

Anovulatory cycles characterized by a lack of progesterone in the luteal phase lead to

An unstable, excessively vascular endometrium . Heavy & irregular bleeding results from an imbalance in the vasoconstricting & vasodilating properties of prostaglandins & an imbalance in platelet aggregation & inhibition

Category A Drugs

Animal and human data show no risk to pregnant women. Examples are prenatal vitamins, insulin, and thyroid hormone. High-dosed multivitamins are not used during pregnancy.

Category B Drugs

Animal studies show no risk. No human data available. ■ Antacids (Tums, Maalox). ■ Docusate sodium (Colace). Colace is a stool softener and is approved for pregnant women. It is not a laxative. Avoid laxatives (e.g., Ex-lax, Bisacodyl), especially in the third trimester (may induce labor). ■ Analgesics (acetaminophen preferred to NSAIDs especially in the third trimester).

Breast Cancer

Annual breast exam: age > 40 years Risks: menarche < 12 years, menopause > 55 yrs Estrogen supplementation Dx: Mammography for > 30 with breast complaint. If suspicious, make sure to order b/l diagnostic mammogram US differentiates fluid-filled cyst from solid mass. Used in pregnancy and age < 30

First line breast cancer treatment

Aromatase Inhibitors and Tamoxifen

Anastrozole MOA and usage

Aromatase inhibitors used to treat post-menopausal women with breast cancer

approach considerations 2

As a means of documenting the presence of UTI, urinalysis is not a substitute for urine culture. However, it can help in identifying those febrile children who should receive antibacterial treatment while culture results from a properly collected urine specimen are pending

What is quick start?

Ascertain that patient is not pregnant by history, timing of menses, pregnancy test likely not needed - Start COC, patch, ring that day - Use backup for 7 days

Discuss the position of the fundus during the corresponding weeks of pregnancy

At 12 weeks the fundus is at or below the pubic symphasis. At 16 weeks the fundus is halfway between the pubic symphasis and the umbilicus At 20 weeks the fundus is at the umbilicus From 22 to 36 weeks the fundus position increases in height by 1 cm

What risk factors predispose some women to AUB?

Athletic women and those with eating disorders at risk for oligomenorrhea and amenorrhea. Illicit drugs inhibit ovulation. Postmenopausal women who are alcohol dependent can have hyperprolactinemia, increased MCV, or both

Inheritance pattern of LS

Autosomal dominance/ highly penetrant

MSI Testing

Available on a commercial basis and cost varies. Consensus panel of 5 microsatellite markers-Bethesda panel- most labs test more to increase sensitivity: BAT25, BAT26, D58346, D2S123, D17S250.

Is there any contraindication with phenazopyridine?

Avoid use in renal impairment (CrCl < 50 ml/min).

Tx of Chlamydia trachomatis endocervicitis

Azithromycin, 1 g orally (single dose), or Doxycycline, 100 mg orally twice daily for 7 days, or Ofloxacin, 300 mg orally twice daily for 7 days, or Levofloxacin, 500 mg orally for 7 days

What is the brand name of phenazopyridine?

Azo, Uristat, Pyridium.

TV Morbidity associated with trichomonal vaginitis may be related to ....

BV

The most common form of vaginitis .? repeated alkalinization of the vagina, which occurs with frequent ....... or use of ....., plays a role.

BV -sexual intercourse, douches

urine collection and analysis

Because it may take as long as 48 hours for a urine culture to be positive, urinalysis is often needed to help make the initial diagnosis of UTI. A urine specimen that is found to be positive on dipstick for nitrite, leukocyte esterase, or blood may indicate a UTI. Dipstick tests have sensitivities approaching 85-90%

Constitutional mismatch repair-deficiency syndrome, i.e. CMMR-D syndrome

Bi-allelic mutations or deletions in MLH1, MSH2, PMS2 genes. Autosomal recessive inheritance. Clinical feature is cafe au lait spots.

MOA of estrogen

Binds to estrogen receptors

Intermenstrual bleeding

Bleeding or spotting between periods

Affects 70% of menstruating women, these symptoms should not be considered a "disorder"

Bloating & breast tenderness

What is the relationship between estrogen and osteoclasts/osteoblasts?

Both osteoblasts and osteoclasts have estrogen receptors. Estrogen increases activity of osteoblasts and inhibits the activity/# of osteoclasts.

What are the nine candida types?

C. albicans C. glabrata C. tropicalis C. krusei C. parapsilosis C. briglis C. limbica C. lipolytica C. lusitaniae

New drug approved for hot flashes: Duavee

CEE 0.45mg and bazedoxifene 20mg (estrogen agonist/antagonist) - approved to treat mod-severe hot flashes & prevent osteoporosis - used in women with intact uterus - bazedoxifene decr risk uterine hyperplasia

Examples of vaginal estrogens: Premarin

CEE Cream twice weekly (initially, two weeks straight to build lining, then drop to twice weekly) * should not have intercourse after application of the cream-avoid for that day

Nonemergent AUB due to anovulation can be treated with?

COCs

Revised Bethesda Guidelines, Tumors should be tested for MSI or IHC when...

CRC dx in patient under 50 years. Presence of synchronous, metachronous colorectal orother Lynch tumor, regardless of age. CRC w/the MSI-H histology dx in a patient who is under 60. CRC dx in a patient with one or more 1st degree relatives with a LS tumor, one dx under age 50. CRC dx w/two or more 1st or 2nd degree relatives with Lynch-related tumors regardless of age.

16 week sono

CRL 11-12

8 week sono

CRL: 16-20mm

9 week sono

CRL: 20-30mm

24 week sono

CRL: 30

12 week sono

CRL=5-6cm

Vitamin and mineral supplementation

Calcium Citrate 500 - 1,000 mg daily, and vitamin D, 400 IU daily Several studies suggest that calcium may reduce PMS symptoms Magnesium 400 mg daily Studies suggest magnesium may help reduce breast tenderness, bloating, migraine, and fluid retention Vitamin E 400 iu daily studies suggest improvement of somatic symptoms and mastalgia Vitamin B6 50 mg per day. The evidence is not overwhelming, but a few studies suggest vitamin B6 may help some women reduce symptoms of PMS

Vitamins and supplements

Calcium, vitamin D, vitamin E, B vitamins

Abnormal and Dysfunctional Uterine Bleeding is a sign of?

Can be sign of systemic diseaseRenal, liver, thyroid diseases; coagulopathies; thrombocytopenia; von Willebrand's disease (VWD); coagulation factor deficiencies

Carcinoma in situ

Cancer that has stayed in the place where it began and has not spread to neighboring tissues

VVC ....... is responsible for 85% to 90% of vaginal yeast infections. Other species of Candida, such as ...... and ......., can cause vulvovaginal symptoms and tend to be ...... to therapy.

Candida albicans C. glabrata C. tropicalis resistant

Cultural & racial background may have an effect on menopausal symptoms

Caucasian & Hispanic women reported the greatest number of psychosomatic symptoms

Etiology Biologic: Excessive amount of prostaglandins

Cause the uterus to contract abnormally & reduce uterine blood flow & oxygenation, thereby causing pain . Decrease in progesterone, estrogen stimulate prostaglandin secretion

Vulvovaginal candidiasis

Causes 25% of vaginitis cases. Usually caused by Candida albicans. Symptoms include pruritus, external dysuria, vaginal discharge.

metronidazole

Causes a disulfiram like reaction when alcohol is consumed.

Second outpatient regimen for PID

Cefoxitin and Probenecid, + doxycycline with/without metronidazole

First outpatient regimen for PID

Ceftriaxone + Doxy with/without metronidazole

Treatment for Gonorrhea

Cephalosporins for Gonorrhea PLUS Azithromycin as a double treatment for resistant gonorrhea (The azithromycin is also for Chlamydia as *co-infection is frequent*)

Cervical Dilation for D and E

Cervical Dilation: at 16wks, osmotic dilator w/Misoprostol prn

Cervical Dilation for Manual Vaccuum Aspiration or Electromechanical Suction

Cervical Dilation: at > 12 wks w/Misoprostol (pv, po)

Inevitable Abortion

Cervix is dilated and unable to stop process. Fetus will be aborted.

risk factors

Children who receive broad-spectrum antibiotics (eg, amoxicillin, cephalexin) that are likely to alter gastrointestinal (GI) and periurethral flora are at increased risk for UTI, because these drugs disturb the urinary tract's natural defense against colonization by pathogenic bacteria. Prolonged incubation of bacteria in bladder urine due to incomplete bladder emptying or infrequent voiding compromises an important bladder defense against infection

children with UTIs

Children with UTIs who have voiding symptoms or dysuria, little or no fever, and no systemic symptoms, have lower cystitis. After age 2 years, UTI manifesting as cystitis is common among girls.

Z pack

Chlamydia treatment. May cause GI upset. Okay for use in pregnancy.

What is the drug of choice for acute uncomplicated cystitis if ≥ 20% of local E. coli is resistant to SMX/TMP or if patients is allergic to sulfa?

Ciprofloxacin 250 mg BID x 3 days OR Ciprofloxacin ER 500 mg QD x 3 days OR Levofloxacin 250 mg QD x 3 days or Nitrofurantoin 100 mg BID x 5 days OR Fosfomycin single dose 3 g in 4 oz of water.

The first choice of medication for ovulation induction is usually

Clomiphene Citrate . Side effects: hot flashes, headaches, ovarian enlargement, multiple gestation, less frequently nausea and visual disturbances

CMMR-D syndrome has a high risk of malignancy in childhood

Colon, stomach, small intestine, leukemia, lymphoma, and brain tumors. The risk of multiple primaries also increased

Contraception

Combined OCs: Absolute contraindications include hx of DVT, CVA, CAD, DM, pregnancy, gyn bleeding, age > 35 with HTN or DM, LDL > 160, trigs >250. Remember ACHES (abd pain, chest pain, HA, eye problems, or severe leg pain) Progestin only: good choice for lactating women, women with CV risks. If pill is 3h late, r/o pregnancy rises Depo-provera: no effect on milk production, long return to ovulation

When is mastalgia common?

Common during pregnancy and lactation

Mifepristone MOA, usage, SE

Competitive inhibitor of progestins at progesterone receptors Used: Terminate pregnancy up to 49d SE: Heavy bleeding, GI effects, abdominal pain (association with fatal infection)

Oral HRT Estrogen Regimen

Conjugated Equine Estrogens (Premarin) 17B Estradiol (estrace) Advantages; lipid profile Disadvantage; clotting

What are the two main types of estrogen used in drugs?

Conjugated estrogens: - from urine of pregnant mares (Premarin) - synthetic form available as well (cenestin-plant based) Estradiol: - active form of endogenous estrogen - micronized oral form (Estrace) - transdermal form (Vivelle)

a woman over 35 years old presenting with uterine bleeding different from her normal; what must be considered and what must be done?

Consider endometrial hyperplasia and cancer; endometrial biopsy

Monophasic oral contraceptives

Contain estrogen and progestin consistent throughout the 21 day period of admin

Biphasic and Triphasic oral contracceptives

Contain estrogen and progestin with concentrations that resemble ratios that occuring during the normal cycle

Advice for Women Taking a Potentially Interacting Medication

Continue to take hormonal contraception even if spotting occurs. • Use backup contraceptive method for duration of time taking the interacting medication plus an additional 7 days

What is the recommended procedure when UTI treatment doesn't show response on the 3-day course?

Culture followed by treatment for 2 weeks.

PMM

Cyclic in the sense that symptoms are present during the postmenstrual (follicular) & early premenstrual (luteal) phases, but often increase in severity during premenstrual phase . Careful evaluation to determine whether an exacerabation of another disorder such as depression, anxiety, substance abuse, IBS, asthma, or chronic pelvic pain

Severe, debilitating menstrual & premenstrual symptom experiences that require professional or pharmacologic intervention

Cyclic pelvic pain & discomfort (CPPD) . Guideline recommend to use this term to differentiate normal cyclic changes associated with menstruation

prognosis 3

Cystitis may cause voiding symptoms and require antibiotics, but it is not associated with long-term, deleterious kidney damage. The voiding symptoms are usually transient, clearing within 24-48 hours of effective treatment

How is the lower urinary tract infection called?

Cystitis.

A 20-year-old woman requests information about emergency contraception using high-dose progestin after having unprotected sexual intercourse approximately 18 hours ago. Today is day 12 of her normally 27-29 day menstrual cycle. You respond that: A. The use of this type of emergency contraception reduces the risk of pregnancy by approximately 33% if taken at this point. B. All forms of emergency contraception must be used within 18 hours of unprotected intercourse to be effective. C. The likelihood of pregnancy is minimal and emergency contraception is not indicated. D. Progestin-based emergency contraceptive is likely helpful in reducing pregnancy risk when used as long as 120 hours post intercourse.

D

According to the US medical eligibility criteria for contraceptive use... ... which of the following clinical scenarios describes a Category 3(exercise caution) situation for the use of a combined oral contraceptive? A. Presence of factor V Leiden mutation B. Report of ectopic pregnancy 2 years ago C. Treatment 3 years ago for high grade squamous intraepithelial neoplasia (HGSIL) with loop electrical excision procedure (LEEP) D. History of gastric bypass surgery

D

quickest way to stop acute bleeding

D & C . Should be considered in a woman 35 years or older with symptoms of hypovolemia because pathology is more likely in this age group

What is the quickest way to stop acute bleeding in a person with AUB?

D&C

SE of estrogen

DES-- clear cell adenocarcinoma Increased risk of enodmetrial cancer, bleeding in post-menopausual, increase risk of thrombi

Urogenital Atrophy in Menopause

Declining estrogen causes drier, thinner and less elastic vaginal tissue. This causes burning, itching pain, and dyspareunia. Same changes occur in urinary tract, causing urgency, frequency and incontinence. Symptoms persist or worsen with aging. Occurs in 30% early menopausal years, and 50% later.

Oligomenorrhea

Decrease blood flow with interval >35 days

What effect does aging have on total bladder capacity?

Decreases

Estrogen overall effect on bone?

Decreases bone resportion, prevents bone loss

Lynch syndrome results from failure of mismatch repair genes

Defective DNA repair is MMR+

Gestational Diabetes Mellitus (GDM)

Diabetes that occurs during pregnancy. GDM mothers are at high risk for type 2 diabetes. GDM has high rates of reoccurrence (33% to 50% chance) in future pregnancies. Higher rates of birth defects (neural tube defects, heart, craniofacial), birth trauma (brachial plexus injury), and neonatal hypoglycemia. Risk factors include obesity, macrosomic infant (>9 lbs), and history of previous gestational diabetes. Plan Consider screening earlier than 24 weeks if at high risk for GDM (presence of risk factors). ■ Screen at 24 to 28 weeks of pregnancy (75-g 2-hour oral glucose tolerance test [OGTT]). ■ Screen earlier if risk factors are present (as above).

Diagnostic Criteria (Starts at 24 Weeks)

Diagnostic Criteria (Starts at 24 Weeks) ■ Fasting: equal or >92 mg/dL ■ 1 hour: equal or >180 mg/dL ■ 2 hours: equal or >153 mg/dL

Disparity by education

Disparity between least educated and most educated rates of unintended pregnancy continues to widen.

Most common type of collaborative reproduction

Donor's sperm

Why fluoroquinolones should be avoided in pregnant women with UTI?

Due to cartilage toxicity and arthopathies.

Hot Flash

Due to changes in hypothalamic thermoregulatory center. Estrogen deficiency increases amount of norepinephrine, which resets the thermoregulatory set point to a lower temperature. Hot flashes occur in women with surgical oophorectomy, patients treated with anti-estrogens, and men treated with anti-androgens.

Why tetracyclines should be avoided in pregnant women with UTI?

Due to teratogenic effects.

Sex related demographics

During the first few months of life, the incidence of UTI in boys exceeds that in girls. By the end of the first year and thereafter, first-time and recurrent UTI are most common in girls

Is subjectively identified acute discomfort that occurs in the abdominal area circumscribed by the pelvis & recurs in a pattern associated with mestrual cycle

Dysmenorrhea . 2 types of Dysmenorrhea: primary & secondary

Difficult or painful menses

Dysmenorrhea . Primary- multiple symptoms: abdominal cramps, backache, bodyaches, headache, other somatic discomforts. - no underlying pathology - more common than secondary . Secondary- involves underlying pathology that causes pain during menses

What are the signs & symptoms of cystitis?

Dysuria (painful urination), urgency, frequency, burning, nocturia, suprapubic heaviness, hematuria (blood in the urine).

What are the most common pathogens in acute uncomplicated pyelonephritis?

E. coli, Enterococci, P. mirabilis, K. pneumoniae, P. aeruginosa.

What are the most common pathogens in acute uncomplicated cystitis?

E. coli, S. saprophyticus and Enterococcus.

When do most abortions occur?

Eighty-eight percent of abortions occur in the first 12 weeks of pregnancy (2006)

medication summary 3

Empiric antibiotics should be chosen for coverage of the most common uropathogens; namely, E coli and Enterococcus, Proteus, and Klebsiella species. For suspected pyelonephritis, parenteral antibiotics are recommended

PMS & CPPD need to be distinguished from

Endocrine abnormalities & from conditions that are subject to PMM

less invasive procedure, as an alternative to hysterectomy

Endometrial ablation . Results in destruction of the endometrium using heated fluid either contained within a balloon or circulating freely within the uterine cavity . should not be performed on the woman who desires to maintain her fertility

abnormal uterine bleeding in a woman greater than 40 years old: can't miss?

Endometrial cancer

ERT (Estrogen replacement therapy) increases the risk for? What helps prevent this risk?

Endometrial cancer--- give ERT + progesterone (HRT) to prevent

Why is it important to treat BV?

Endometritis (infection of the uterus) Pelvic Inflammatory Disease Post-hysterectomy surgical infection Septic abortion Chorioamnionitis Preterm Labor and Delivery Abnormal pap smears

EPCAM

Epithelial Cell Adhesion Molecule gene: 1. inhibits MSH2. 2. Deletions in 3' end lead to hypermethylation of MSH2 promoter and silencing of gene. 3. risk of CRC however, risk of extra-colonic cancers may be lower-more studies necessary.

Examples of vaginal estrogens: Femring

Estradiol acetate vaginal ring (higher conc of progestins for systemic effects) replace every 90 days

Examples of vaginal estrogens: Estrace

Estradiol cream twice weekly (initially, two weeks straight to build lining, then drop to twice weekly) * should not have intercourse after application of the cream-avoid for that day

Examples of vaginal estrogens: Estring

Estradiol vaginal ring (lower conc for local effects) replace every 90 days

Occurs as a result of endometrium being stimulated by long- term chronic unopposed estrogen, such as in women experiencing chronic anovulation. E.g. PCOS

Estrogen breakthrough bleeding

T/F? Abortions performed in the first trimester pose risks to future pregnancies and documented mental health risks.

FALSE

T/F? Restrictive abortion laws are associated with lower abortion rates.

FALSE

amenorrhea in an older patient considered this lab test

FSH level *elevated initially in menopause; must have more than one result >30iu/ml

recommendations 1

Febrile infants aged 2-24 months with UTI should undergo renal and bladder ultrasonography.[5] In addition, the following pediatric patients should also undergo ultrasonography of the urinary tract after a febrile UTI

febrile infants younger than 3 months

Febrile infants younger than 3 months are an important subset of children who may present with fever without a localizing source. Workup of fever in these infants should always include evaluation for UTI.

Jarisch-Herxheimer symptoms and treatment

Fever, chills, tachycardia, tachypnea. Treatment is supportive.

Types of benign breast masses

Fibroadenomas cysts, lipomas fat necroses phyllodes tumors:the tumor cells grow in a leaf-like pattern hamartomas: non cancerous tumor benign focal malformation galactoceles: benign milk-filled cyst.

What type of breast changes common among women with mastalgia

Fibrocystic

What is Standard start?

First day of menses start -Start COC, patch, ring on first day of menses -No backup needed

What are the signs & symptoms of pyelonephritis?

Flank pain, abdominal pain, fever, nausea and vomiting, malaise.

The most important cluster for distiguishing women with low- symptom severity from those with PMS or premenstrual magnification

Fluid retention

What is the drug of choice to treat moderate ill outpatient suffering from acute uncomplicated pyelonephritis?

Fluoroquinolones PO x 5 - 7 days (Cipro 500mg BID or Cipro 1000mg ER QD or Levofloxacin 750mg QD or Ofloxacin 400mg BID.

What is the drug of choice to treat severe hospitalized patients suffering from acute uncomplicated pyelonephritis?

Fluoroquinolones, Ampicillin + Gentamycin, Zosyn (Pipperacillin + Tazobactam), Ceftriaxone (Rocephin) x 14 days. IV route is preferred. Most acute uncomplicated pyelonephritis require hospitalization.

Pelvic Infections Abx Coverage

For full spectrum: Pip/tazo or Ertapenem OR pair Flagyl with a cephalosporin

Levonorgestrel-containing IUS/IUD (LNg-IUS/IUD) Mechanism

Foreign body effect induced by presence of devise in intrauterine cavity results in a sterile inflammatory response that is toxic to sperm and ova, impairs implantation Local uterine changes enhanced by presence of progestin which causes thickening of cervical mucus, resulting in physical barrier to prevent sperm from entering uterus. Additional changes include LNG-induced endometrial thinning that discouraged implantation

BV = nonspecific vaginitis or....... . It is an alteration of normal vaginal bacterial flora that results in the loss of hydrogen peroxide-producing aerobic ...... and an overgrowth of predominantly ........ bacteria .

Gardnella vaginitis lactobacilli anaerobic

Duration of HRT

General Recommendation; Short term therapy during menopausal period (3-5 yrs) Don't treat past 60yo. Extending past 60yo is acceptable under supervision and clear expression of risk/benefit.

5 week Sono

Gestational sac and Yolk sac

What does lactobacillus produce?

H2O2, peroxidase, chloride, and bacteriodins (all keep bacterial levels low)

What diagnostic test MUST a woman with AUB have ordered?

HCG All women who present with AUB considered pregnant until proven otherwise.

Length of Medical Abortion

HEAVY BLEEDING 4-6 HRS (and mild for several wks)

HERS Study Findings

HRT initiated in older women with pre-existent heart disease has negative impact. HRT cannot be used as secondary intervention for heart disease.

initial treatment measures to consider in menopause (3)

HRT, exercise, calcium and vitamin D

labs to order with postmenopausal bleeding

HSG, CBC, TSH, prolactin, FSH

...

HSV suppressive therapy for those who are constantly breaking out.

PHYSIOLOGIC CHANGES DURING PREGNANCY

Heart The heart is displaced into a horizontal position by the larger uterus. It causes the heart to lie in horizontal position and to rotate to the left (increases the transverse diameter). Cardiac Output Increases by 30% to 50% and peaks at about 24 weeks. Plasma Volume Increases by 30% to 50% by the end of the third trimester. Heart Rate Increases during pregnancy by 15 to 20 beats per minute. Preload and Afterload Preload increases due to higher blood volume. The afterload goes down because of the decrease in peripheral vascular resistance that occurs during pregnancy. Blood Pressure The systolic and diastolic pressure starts to decrease in the first trimester and continues to the second trimester. Many mothers who are hypertensive before pregnancy can be off prescription antihypertensives at this time. By the third trimester, the BP gradually returns back to prepregnancy levels. Antihypertensives used for pregnant women are methyldopa (Aldomet) and labetolol (Normodyne) (beta blocker). Systolic Ejection Murmur (95%) Caused by the increase in blood volume and cardiac output later in pregnancy. Occurs during systole. Mammary Souffle Bilateral continuous murmur from increased blood fl ow in the mammary vessels. Vena Cava Compression by enlarged uterus decreases blood return to the brain, resulting in orthostatic hypotension. Advise women to lie on the left side and to change positions slowly. Lungs The gravid uterus pushes up the diaphragm as it gets larger. The diameter of the thorax is increased. Coagulation Factors Pregnancy is a hypercoagulable state (clotting factors go up) especially after labor. Physiologic Anemia of Pregnancy Physiologic anemia of pregnancy is most obvious during the second and third trimesters. Hemoglobin and hematocrit are decreased because of the hemodilution from increased plasma volume. The hemoglobin can decrease to as low as 9.5 to 10 g/dL. Thyroid Diffusely enlarged (size up to 15% larger). Higher metabolic activity. Free T3 (triiodothyronine) is mildly elevated. GI Tract Decreased peristalsis from progesterone effects (constipation, heartburn). Skin Pigmentation Pigmentary changes from increase in melanocyte-stimulating hormone from higher levels of estrogen. Causes the linea nigra (dark pigmented "line" that extends from mons pubis to the umbilicus located midline). Darkening of color of the nipples and areola. Chloasma (Melasma) Blotchy hyperpigmentation on forehead, cheeks, nose, upper lip seen in pregnant women and in some birth control pill users. Hyperpigmentation of the skin is usually permanent but becomes lighter over time. More common in darker skins (olive skins and darker). Renal System Kidneys increase in size during pregnancy. The ureters and renal pelvis become dilated (physiologic hydronephrosis). The glomerular filtration rate (GFR) is much higher in pregnancy due to higher cardiac output and renal blood fl ow. ENT Some women develop nasal congestion due to increased blood flow during pregnancy. Varicose Veins Varicose veins become more severe during pregnancy. Edema Mild edema of the lower extremities and the feet is normal and is most noticeable in the third trimester.

Anovulatory bleeding also tends to be heavy secondary to

High & sustained levels of unopposed estrogen that can result in endometrial hyperplasia . Endometrial hyperplasia can result in episodes of amenorrhea, menorrhagia, & intermenstrual bleeding

Results scored as MSI-H, MSI-L, or MSS

High: 30-40% or more unstable, low fewer than 40%, or MSS is stable. 1 of every 5 MSI-H CRC associated with LS. 1 of every 40 endometrial cancer associated with LS. MHI results may direct the next steps.

Menopausal Therapy

Hormone Replacement therapy. Initiated at or soon after menopause Many theories.

Risk factors for endometrial cancer

Hormones : birth control pills, or tamoxifen; the number of menstrual cycles (over a lifetime), pregnancy, obesity, certain ovarian tumors Anovulation polycystic ovarian syndrome Nullparity Obesity Family history

Hot Flashes and HRT

Hot flashes (moderate to severe) are #1 indication to try EPT/ET. Good time indicator (within 2 years of menses cessation) Estrogen decreases hot flashes by up to 90%. Paroxetine (SSRI) is non-hormonal alternative.

Symptoms of Menopause

Hot flashes (vasomotor changes) Sleep disturbances (lack of estrogen decreases serotonin) (hot flashes contribute) Osteoporosis Uro-genital atrophy Mood swings, depression, anxiety Memory problems

ultrasonography 2

However, a current recommendation is that urinary ultrasonography be omitted after a first febrile UTI in infants and children if they respond to treatment (afebrile within 72h), good follow-up is assured, and no voiding abnormality (no dribbling of urine) or abdominal mass is present

last resort for menorrhagia not resolved with other treatments

Hysterectomy

Noncontraceptive benefits of OC increase a decrease in all of the following:

IDA, PID, ovarian cancer

Immunohistochemistry

IHC = staining of pathology sample to visualize the protein of interest in a tumor and control samples. IHC for protein expression of the MLH1, MSH2, MSH6 and PMS2 genes. Loss of expression can direct germline testing.

Pelvic examination should focus on identifying any abnormalities of the internal genitalia such as enlargement, tenderness, masses, as well as evidence of gynecologic infections or STI's.

If Infection is suspected, a microscopic examination of vagina secretions chlamydia gonorrhea testing should be performed.

medication summary 2

If the urinalysis is positive for nitrites, the bacterium responsible for the infection is exceedingly likely to be sensitive to a third-generation cephalosporin.

What is the prophylaxis guidelines for acute uncomplicated cystitis?

If there are three or more episodes in one year, take 1 SMX/TMP SS (400mg/80mg) QD or SMX/TMP DS (800mg/160mg) post coitus.

Cefixime

If you use _____ to treat gonorrhea, you must follow up with the patient in 7 days to ensure success, as gonorrhea has increased resistance to the drug.

Post-Medical Abortion Contraception

Immediate (w/in 1 wk)

Rh-Incompatibility Disease

In Rh-negative mothers with Rh-positive fetuses, the maternal immune system develops antibodies against Rh-positive blood if not given RhoGAM (gamma globulin against Rh factor). Give RhoGAM for all pregnancies of Rh-negative mothers—even if they terminate in miscarriages, abortions, or tubal or ectopic pregnancies.

prognosis 4

In developed countries, kidney damage with long-term complications as a consequence of UTI per se is currently less common than it was in the early 20th century, when pyelonephritis was a frequent cause of hypertension and ESRD in young women. This change is probably a result of improved overall healthcare and close follow-up of children after an episode of pyelonephritis. In countries with high-quality healthcare, hypertension, impaired renal function, and ESRD are now most commonly encountered in infants with intrauterine renal damage.

Is the most widely used assisted reproductive technology (ART) procedure

In vitro fertilization . In this technique, the ovaries are hyperstimulated with medication and then several mature ova are surgically retrieved, placed in laboratory dish, and the mixed with spem . Fertilization takes place in vitro and, after fertilization, one or more embryos are transferred directly into the woman's uterus for implantation . Bypasses the fallopian tubes, thus, commonly attempted on women who have tubal blockage from structural problems or secondary to pelvic infection or scan tissue . Also used when the etiology of infertility remains unknown

Menopause and Heart Disease

Incidence of atherosclerosis increases in postmenopausal women as opposed to premenopausal women. More women die of heard disease than breast cancer.

occurs in dysmenorrhea causing pain

Increase or imbalance in prostaglandins

What effect does aging have on post-void residual urine?

Increases

What are the complications of amenorrhea?

Infertility and osteoporosis

Clomiphene MOA, usage, side effects

Inhibits the estrogen receptors at the anterior pituitary (loss of negative feedback)--> Increase FSH/LH release--> stimulate ovulation Use: Treats infertility due to anovulation (PCOS) SE: hot flashes, ovarian enlargement, multiple pregnancies, visual disturbances

similar

Inpatient and outpatient therapy has _____ outcomes, as long as the patient is relatively healthy (i.e. non pregnant, afebrile etc)

Bleeding or spotting between normal menses, flow usually light

Intermenstrual bleeding

Common causes of non-milky nipple discharge?

Intraductal papilloma/mammary duct ectasia Paget's disease Sexual stimulation infection or abscess

What is a complicated UTI?

It is generally a result from an abstruction or abnormalities (congenital, stones, indwelling catheter) or neurologic deficit that interferes with the normal flow of urine.

What is an uncomplicated UTI?

It is not associated with structural or neurological abnormalities that may interfere with the normal flow of urine.

What risks does SMX/TMP have during pregnancy?

It is pregnancy category C (no controlled studies in humans have been performed and animal studies have shown adverse events, OR studies in humans and animals are not available, give only if potential benefit outweighs the risk), but 2 weeks before EDC it is pregnancy category D (positive evidence of fetal risk is available, but the benefits may outweigh the risk if life threatening or serious disease) as it can cause hyperbilirubinemia and kernicterus in the 3rd trimester.

Juxtaposition

Juxtaposition is placing two elements or words side by side so the reader can compare them and contrast them.

Genetic Heterogeneity in LS

LS is associated with germline mutations in any one of at least 4 genes. MSH6 & MSH2 on chromosome 2p. MLH1 gene is on chromosome 3p. PMS2 gene is on chromosome 7p.

Causes of defective MMR leading to MSI

LS- germline MMR mutation and acquired mutation in working MMR gene. MLH 1 hypermethylation of one MMR gene and acquired mutation of other copy.

What is normal vaginal flora?

Lactobacillus (90-95%) Aerobic Bacteria: Gardeneralla vaginallis, staphylococus epidermidis, e-coli Anaerobic: prevotella and peptoseptolococus

effective therapy for menorrhagia caused by fibroids.

Levonorgestrel-releasing intrauterine system (LNG-IUS)

Mismatch repair and microsatellites

Linkage to 2p and 3p in LS families using microsatellite markers- stretches of DNA with repeating units of 2,3, or 4 nucleotides. Look for LOH at 2p in tumors- except with tumor suppressor genes. Instead,found NEW microsatellite alleles. Genome wide instability of replication/repair of repeated sequences = microsatellite instability.

Is there Evidence to support a follow up visit after 1st trimester?

Little evidence

prognosis 2

Long-term complications of pyelonephritis are hypertension, impaired kidney function, end-stage renal disease (ESRD), and complications of pregnancy (eg, UTI, pregnancy-related hypertension, low-birth-weight neonates). Long-term complications of UTI are caused by renal damage secondary to pyelonephritis.

Harvard Nurse's Study Findings

Longterm HRT confers cardioprotective effects. Longterm (>10yr) HRT confers increase in breast cancer.

What occurs during menopause?

Loss of 17 beta-estradiol from the ovaries (still have estrogen production from other tissues such as estrone from adipose)

Perimenopausal Therapy

Low dose contraceptives (patients still may become pregnant) Hormone regiments for menopause, but not potent enough to prevent pregnancy.

Recommended to initiate ET & EPT at

Lower than standard doses

Gonadotropin-releasing hormone agonists (GnRHa): not recommended for long-term use.

Lupron , synarel, zoladex may be used for short period of time while a woman is awaiting surgical treatment for her heavy bleeding

vaginitis

MCC of itching, discharge and odor. Common infection and recurrent cause of abnormal vaginal discharge in women of childbearing age.

Who Can Provide Manual Vacuum Aspiration (MVA) in the United States?

MDs MID-LEVEL PROVIDERS (PA's, CNMs, NPs) In Some states: Oregon, Montana, Vermont, New Hampshire and California (by law)

Support/Privacy for surg abortion

MEDICAL FACILITY/NEED SUPPORT PERSON

Vast majority of mutations, about 90%, are found in which 2 genes?

MLH1 & MSH2

Genes belong to DNA mismatch repair, i.e. MMR, family

MLH1, MSH2, MSHH6, & PMS2

Raloxifene MOA Used to treat? SE?

MOA: Agonist at bone, antagonist at uterus Tx: Osteoporosis SE: Increased risk of thromboembolic events

Tamoxifen MOA, clinical use, SE

MOA: Agonist at uterus, antagonist at breast tissue Used to treat/prevent estrogen sensitive breast cancer SE: Thromboembolic events, increased risk for endometrial cancer (5 year max)

Tumor Testing key features

MSI can be found in most LS associated tumors. Used for LS in colorectal and endometrial tumors routinely. Little info on MSI/IHC screening with other LS associated tumors. Normal in other types of tumors should be taken with caution as they do not rule out diagnosis of LS.

Mastalgia

Majority cyclic and occurs premenstrually; normal, physiologic condition caused by hormonal changes of menstrual cycle

Long-Term Consequences of Abortion

Majority of studies indicate abortion is not associated with any significant morbidities related to future fertility, future birth outcomes, cancer, or mental health.

Should be the last resort in many cases of abnormal uterine bleeding

Medical intervention, particularly, surgical

Asymptomatic Bacteriuria Treatment

Medications ■ Nitrofurantoin (Macrobid) every 12 hours (BID) for 5 days. ■ Amoxicillin (or Augmentin) every 12 hours (BID) for 3 to 7 days. ■ Cephalexin BID for 3 to 7 days ■ Fosfomycin 3 g for a single dose ■ Do not use sulfas (e.g., Bactrim) or nitrofurantoin near term due to the risk of hyperbilirubinemia. Causes hemolysis if the mother (or both mother and baby) has G6PDanemia.

Oral HRT Progestin Regimen

Medroxyprogesterone Acetate (MPA)(Provera); considered standard. Disadvantage; Opposes positive lipid effect of estrogen. Micronized progesterone (MP)(Prometrium); minimized hepatic first pass. Does not oppose lipid effects of estrogen. Combined with estrogen.

NSAID Therapy for menorrhagia

Mefanamic 500mg TID Ibuprofen 600mg TID Naproxen Sodium 550mg loading dose then 275 mg Q 6 hours

Irregular, heavy, prolonged menstrual bleeding

Menometrorrhagia

HRT is given to treat what symptoms?

Menopausal Sx 1. Genitourinary atrophy--> restore the epithelial lining 2. Stabilizes the vasomotor instability 3. Prevent bone loss associated w/menopause 4. CVD (must be given in therapeutic window to avoid - effects)

abnormally long or heavy menses; regular

Menorrhagia

Tx for Trichomonas Vaginitis

Metronidazole or Tinidazole (same MOA of Flagyl) -*Must treat male sexual partners also*

Classic PMS pattern or cyclic perimenstrual mood & physical discomforts include

Moderate to extremely severe for up to 2 weeks before the onset of menses . Symptoms disappear or become milder within 1 to 4 days of onset of bleeding

Women who have regular, ovulatory menstrual cycles often experience premenstrual symptoms such as bloating, fatigue, constipation & mood changes. This is collectively called

Molimina . A result of higher progesterone

For women who did not respond to Clomiphene Citrate

More potent ovulation induction with injectable exogenous gonadotropins

prognosis 1

Mortality related to UTI is exceedingly rare in otherwise healthy children in developed countries. Morbidity associated with pyelonephritis is characterized by systemic symptoms such as fever, abdominal pain, vomiting, and dehydration. Bacteremia and clinical sepsis may occur. Children with pyelonephritis also may have cystitis and may develop lobar inflammation of the kidney (lobar or focal nephronia) or renal abscess. Any inflammation of the renal parenchyma may lead to scar formation.

Fibroadenoma

Most common in < 25 years Order US to dx

Breastfeeding Mastitis (Lactational Mastitis)

Most common in the fi rst 2 months of breastfeeding. Skin fissures on the nipple(s) allow bacterial entry. Most common organism is Staphylococcus aureus (Gram-positive). Consider methicillin-resistant staph aureas (MRSA) bacterial infection (becoming more common). If severe or toxic, refer to ED or admit to the hospital. Prevention Frequent and complete emptying of breast and proper breastfeeding technique. Breast engorgement and poor technique increases risk of mastitis. Classic Case Patient who is breastfeeding complains of the sudden onset of a red, firm, and tender area (induration) on one breast. May also have fever/chills and malaise (flu-like symptoms). May have adenopathy on the axilla by the affected breast. Labs ■ Usually not needed. A clinical diagnosis. ■ CBC shows leukocytosis. Order C&S of milk if hospital-acquired, severe, or not responding to antibiotic treatment. Medications If Low Risk of MRSA ■ Dicloxacillin 500 mg PO QID or cephalexin (Kefl ex) 500 mg PO QID for 10 to 14 days. If High Risk of MRSA ■ Trimethoprim-sulfamethoxazole (Bactrim) 1 to 2 tablets PO BID for 10 to 14 days. ■ OR clindamycin 300 mg PO QID for 10 to 14 days. ■ Continue to breastfeed on affected breast during antibiotic treatment. If unable to breastfeed, pump milk from infected breast (and discard) to prevent engorgement. ■ If suspect breast abscess, order an ultrasound and refer for incision and drainage. ■ Ibuprofen for pain as needed. Cold compresses on indurated breast area. ■ Refer to lactation consultant if suspect poor breastfeeding technique.

What quinolones cannot be used to treat UTIs? Why?

Moxifloxacin and Gemifloxacin do not reach high levels in the urine.

"Clotrimazole" Consideration

NOT a PPI; actually an antifungal -Think *Clotramin* brand athlete's foot creams

What can the presence of stool during an incontinent episode indicate?

Neurogenic combined urinary and fecal incontinence

ovulatory-idiopathic menorrhagia

Nonsteroidal anti-inflammatory drugs (NSAIDs)

What progestin products are used?

Norethindrone: first generation Medroxyprogesterone; less androgenic (WHI) Dropsirenone: antiandrogenic & antialdosterone (incr K+, monitor K if on ACEI/ARB, incr risk hyperkalemia) Micronized progesterone

Goals for Treating AUB

Normalize bleeding. Correct any anemia. Prevent cancer. Restore quality of life

approach considerations 4

Obtain a complete blood count (CBC) and basic metabolic panel for children with a presumptive diagnosis of pyelonephritis. Perform blood cultures in febrile infants and older patients who are clinically ill, toxic, or severely febrile

age related demographics

Occurrences of first-time symptomatic UTI are highest in boys and girls during the first year of life and markedly decrease after that. The minimum cumulative incidence in boys and girls aged 2 years is slightly over 2%.

What effect does aging have on first urge to void?

Occurs at a lower capacity

Decreased, scanty flow, > 35 days

Oligomenorrhea

Hyperthyroidism can result in

Oligomenorrhea or amenorrhea

What diagnostic test should be ordered for a benign breast mass?

Order mammogram and ultrasound (less than 40 years old) ; biopsy required

Uses of estrogen

Ovarian failure, post-menopausal hormone replacement, menstrual abnormlaities

An abnormal flow rate is specific for the following types of incontinence:

Overflow incontinence

What is an elevated postvoid residual urine of 200ml indicative of?

Overflow incontinence

Occurs less often than abnormal bleeding due to anovulation

Ovulatory abnormal bleeding . Occurs most often in the postadolescent and during premenopausal years

Support/Privacy for med abortion

PRIVACY/AT HOME/NEED SUPPORT PERSON

PID treatment regimen A.

Parental (24 hours) cefotetan or cefoxitin + doxycycline. Continue Doxy for 14 days.

PID treatment regimen B.

Parenteral (24 hours) clindamycin plus gentamicin + clindamycin or doxy for 14 days.

Lymphatic drainage

Patient supine; physician at the foot of the table. 2. Place the palms of both hands over the metatarsal heads on the plantar surface of the feet and dorsiflexesthem to their physiologic barriers. 3. Apply a low velocity, moderate amplitude springing force in a cephaladdirection. The repetition rate of the springing is varied until the maximal abdominal visceral motion is observed. 4. Continue for 1-2 minutes, to patient's tolerance, or until the desired fluid response is obtained

best guide in diagnosis or differential diagnosis

Patterns and severity of symptoms

Refers to the period from about 7-10 days before menstrual flow begins until the 1st or 2nd day of menstrual flow

Perimenstrual

Are psychological symptoms that escalate & subside around the time of menstruation & subsides after menses

Perimenstrual mood discomforts . Symptoms include depression, irritability, tension, anxiety, anger, tearfulness

Are painful physical symptoms other than those commonly associated with CPP that escalate around the time of menstruation & subside after menses begins

Perimenstrual physical discomforts . Fatigue . Fluid retention . Headache . Joint pains . Breast tenderness . Change in energy & appetite

Regular, frequent menses, <24 days

Polymenorrhea

Ovulatory abnormal bleeding cycles are regular & tend to be cyclic, although bleeding patterns are often abnormal. May include

Polymenorrhea, oligomenorrhea, midcycle spotting,& menorrhagia

Refers to the first five years or so following menopause when hormonal fluctuations often continue to occur

Post menopausal period

When does ovulatory AUB occur?

Postadolescent years and during premenopausal years

Preeclampsia (Pregnancy-Induced Hypertension)

Preeclampsia (Pregnancy-Induced Hypertension) Most cases of preeclampsia occur in the late third trimester (34 weeks of gestation or later). It can occur up to 4 weeks after childbirth (postpartum period). Mild cases of preeclampsia may not have symptoms such as headaches, blurred vision, or right upper quadrant abdominal pain. Will cause multiorgan damage such as to the brain (stroke), kidneys (acute renal failure), lungs (pulmonary edema), liver (hepatic rupture), DIC, and fetal and/or maternal death. Older name is "toxemia." The exact etiology of preeclampsia is unknown. If seizuresdevelop, will be diagnosed with eclampsia. Risk factors: primigravida, multipara, older than 35 years of age, obesity, prior history of preeclampsia, hypertension, or kidney disease. Diagnostic Criteria Classic triad: Hypertension, proteinuria, and edema that occur after 20 weeks of gestation. Take at least two separate readings (at least 6 hours apart). ■ Systolic BP: ≥140 mm Hg or ■ Diastolic BP: ≥90 mm Hg ■ Proteinuria: >0.3 g protein in a 24-hour urine specimen. Proteinuria ranges from trace to 1+ to 4+ (severe cases). ■ Rapid weight gain of from 2 to 5 lbs per week. The edema is most obvious in the face, around the eyes, and on the hands. Treatment Plan Refer to obstetrician for management. The only defi nitive "cure" for preeclampsia and eclampsia is delivery of the placenta/fetus.

All women of childbearing age with AUB is considered

Pregnant until proven otherwise

Converting from Premarin dosing to Vivelle or Estrace:

Premarin 0.625mg:Vivelle 50 mcg/day Premarin 0.625 mg: Estrace 1 mg

the main medication used for estrogen replacement therapy, the dosage range, the most common initial dose?

Premarin, 0.3 mg-1.25 mg tablets, 0.625 mg and titrate

Exacerbation of an ongoing mental health problem or physical disorder

Premenstrual magnification (PMM) . Women experience moderately severe symptoms postmenses and more severe symptoms premenses . Conditions that may worsen during premenstrual or menstrual phase include: eating disorders, bipolar, major depression/ anxiety, personality disorders, IBS, asthma, allergies

Indicates cyclical recurrence of distressing physical, mood & behavioral experiences that often affects interpersonal relationships & personal health & function

Premenstrual syndrome (PMS) . Can be perceived as an absence of symptoms or low severity symptoms after menses, usually day 6 through 10 of cycle followed by an escalation of symptoms premenstrual during the 7 days preceding the next menses that subside again @ the onset of menses . pattern begins 3 days prior and ends 4 days after flow ceases

Bethesda Guidelines state MSI-H histology

Presnce of tumor infiltrating lymphocytes, Chron's-like lymphocytic reaction, mucinous/signet ring, or medullary growth pattern.

Metronidazole or metro gel or clindamycin

Primary Bacterial vaginosis

Diagnosed by exclusion when painful menstruation occurs in the absence of pathology

Primary dysmenorrhea . Increased endometrial prostaglandin is believed to be the cause

Tx for Bacterial Vanginitis/osis + Secondary Tx

Primary: Metronidazole or Tinidazole (same MOA of Flagyl) *Secondary: clindamycin*

Occurs when progesterone - estrogen ratio becomes elevated. Seen in women using progestin- only pills or other forms of progestin- only contraception

Progesterone breakthrough bleeding

treat chronic heavy bleeding due to anovulation.

Progestogens

Options counseling- Active Listening

Provide information, give support, assist in evaluating options -Establish rapport -Use open ended questions: How do you feel about being pregnant? Do you know what your choices are? -Assess support systems: partners, parents -Establish timetable for decision making

Conclusion from Guttmacher study

Public health campaigns should dispel myths, address magical thinking, and call attention to the general problem of low health literacy

Fulfestrant

Pure anti-estrogen used to replace Tamoxifen after 5 years

How is the upper urinary tract infection called?

Pyelonephritis.

Outcomes of WHI and Quality of life and Dementia

QOL: - estrogen+progestin improved vasomotor sx's; did NOT improve QOL-related health measures Dementia: - estrogen+progestin did NOT improve cognition in women over 65; INCR risk of dementia in women over 65

Jarisch-Herxheimer

Reaction which occurs secondary to spirochete lysis and pro inflammatory cascade. Can occur as early a 2 hours after PCN administration and usually resolve with 24 hours.

What is the first-line treatment of Mastalgia?

Reassurance

Effects of WHI Study

Recommended dosages of HRT were significantly lowered. Lowest amount of drug to control symptoms is now recommended.

USPSTF GUIDELINES for Breast Cancer Screening

Recommends that primary care providers screen women who have family members with breast, ovarian, tubal, or peritoneal cancer with one of several screening tools designed to identify a family history that may be associated with an increased risk for potentially harmful mutations in breast cancer susceptibility genes (BRCA1 or BRCA2). Women with positive screening results should receive genetic counseling and, if indicated after counseling, BRCA testing Grade B recommendation The USPSTF recommends against routine genetic counseling or BRCA testing for women whose family history is not associated with an increased risk for potentially harmful mutations in the BRCA1 or BRCA2 genes Grade: D Recommendation

Complicated VVC

Recurrent symptoms Severe symptoms Non-albicans Candida Immunocompromised

Collaborative reproduction

Refers to the involvement of a person who will not be raising the child, such as a sperm or egg donor or surrogate mother . Chosen when infertility treatment or ART has proved unsuccessful or is not possible because of individual factors

Polymenorrhea

Regular, frequent periods with intervals< 24 days

What is the drug of choice for acute uncomplicated cystitis?

SMX/TMP DS (800mg/160mg) tab BID x 3 days.

Follow-up eval for med abortion

SONO F/U, no exam

What non-hormonal options are there for hot flashes?

SSRIs and SNRIs: relieve sx's of vasomotor instability - AVOID with breast cancer pts on tamoxifen (may decr formation of tamoxifen's active metabolite) Gabapentin: mech unknown Clonidine: considered less effective

FDA approved medications for vasomotor symptoms (4)

SSRIs, SSNRIs, gabapentin, clonidine

cephalosporins

Safe in pregnancy

____________leading cause of cancer deaths in women

Second

Involves an underlying pathology acting directly or indirectly on the pelvic anatomy to cause pain symptoms during menstruation

Secondary dysmenorrhea . Associated with dyspareunia, dysuria, abnormal uterine bleeding, or infertility . The pain associated with either primary or secondary is the same; thus, the mere presence of pain does not indicate which type of dysmenorrhea is experienced

HELLP (Hemolysis, Elevated Liver Enzymes, and Low Platelets) Syndrome

Serious but rare complication of preeclampsia. Classic patient is a multipara older than 25 years who is in the third trimester of pregnancy. Presence of the signs and symptoms of preeclampsia that is accompanied by right upper quadrant (or midepigastric) abdominal pain with nausea/vomiting and malaise. Symptoms can present suddenly. Lab abnormalities are elevation of the alanine aminotranferease (AST), aspartate aminotransferase (ALT), total bilirubin, lactate dehydrogenase (LDH) with decreased number of platelets (<100,000 cells/microl) and hemoglobin and hematocrit.

For how long pregnant women must be treated for UTI? Why?

Seven days. If not, can lead to premature birth or pyelonephritis.

Risk Factors involved w/BV?

Sexual activity: multiple partners, new partner in last 30 days, lesbian sexual activity frequent douching antibiotic use low lactobacilus and socioeconomic status

Alleviation of Menopausal Symptoms by HRT

Sleep; increased estrogen prevents sleep disturbance. CV; Cardioprotective via improved lipid profile. UG; Reverses atrophy of vulva, vagina, urethra, and trigone of bladder. Osteoporosis; Estrogen decreases resorption of bone, and has been shown to decrease # of hip fractures. Must begin within 2-3 years of onset for full efficacy. Vasomotor; Estrogen reestablishes feedback on hypothalamic control of norepinephrine secretion, leading to decreased hot flashes.

What can cause stress incontinence in men?

Sphincter damage that occurs after prostate surgery

Uncomplicated VVC

Sporadic or infrequent Mild to moderate sympt Candida albicans Immunocompetent

medication summary 1

Start antibiotics after urinalysis and culture are obtained in patients with UTI. A 10-day course of antibiotics is recommended, even for an uncomplicated infection. Typical short-course treatments should be reserved for nontoxic-appearing adolescent females with UTI.

approach considerations 3

Studies report that the specific components of the dipstick test and complete urinalysis cannot detect positive culture findings in all children who have positive urine culture findings and unexplained fever or voiding symptoms. The conclusion is that urine should be obtained for culture, and initial treatment should be given to those children with fever and a presumptive diagnosis of a UTI, regardless of the result of urinary dipstick testing or urinalysis

Guttmacher Study

Study participants reported thinking they were at LOW RISK for pregnancy because 1) perceived invulnerability to pregnancy Magical Thinking: "always had good luck"; " I just really don't want children, so thought that would prevent me from getting pregnant 2) 1/3 thought they or partners were sterile 3) contraceptive use would protect them (majority were using incorrectly or inconsistently)

Which endocrine glands are stressed during pregnancy?

THYROID (BC Metabolism is for two people) PANCREAS (Glucose needed by two people)

Which age group has the highest PERCENT of unintended pregnancies?

Teens

80% of of women experiencing dysmenorrhea are in their

Teens & early 20s

RPR

Test for syphilis

2 categories

The 2 broad clinical categories of UTI are pyelonephritis (upper UTI) and cystitis (lower UTI).

recommendations 2

The American Academy of Pediatrics recommends that all infants and young children (aged 2mo to 2y) with a first UTI undergo urinary tract ultrasonography and VCUG. These tests should be acquired promptly if patients fail to show expected clinical response within 2 days of treatment

The most common times for a woman to experience irregular menses are at

The beginning & the end of her reproductive life cycle: postmenarche & perimenopause . Affected by normal life- cycle transitions that occur during the 1st 2 years after menarche & 3 years prior to menopause - irregular bleeding at this time may be a reflection of normal functioning

Ovulation detection: women can detect ovulation by monitoring and recording their basal body temperature each day upon awakening

The biphasic cycle is indicated by temperature recordings consistently < 98°F during the follicular phase and consistently higher than 98°F during the Luteal phase. Usually there is a slight drop in temperature just prior to ovulation, and a surge in temperature with ovulation . Recorded for at least 3 months = 3 menstrual cycles

Artificial insemination

The clinician places the semen into the woman's cervix. With IUI, the semen placed directly into the uterus, bypassing the cervix . This approach is particularly warranted when the problem with the cervical mucus is the cause of the infertility

approach considerations 1

The diagnosis of UTI is based on the quantitative culture of a properly collected urine specimen. A midstream, clean-catch specimen may be obtained from children who have urinary control. In the infant or child unable to void on request, the specimen for culture should be obtained by means of suprapubic aspiration or urethral catheterization

pyelonephritis

The febrile infant or child with clinically significant bacteriuria and no other site of infection to explain the fever, even in the absence of systemic symptoms, has pyelonephritis (ie, upper UTI). Most episodes of UTI during the first year of life are pyelonephritis

reccomendations 1

The following types of pediatric patients should undergo VCUG after a first febrile UTI: • Those in whom treatment fails after 48-72 hours • Patients with an abnormal voiding pattern (dribbling of urine) • Infants and children in whom good follow-up is not assured • Those with an abdominal mass • Infants and children with recurrence of a febrile UTI

most common causative organism

The most common causative organisms are bowel flora, typically gram-negative rods. Escherichia coli is the most commonly isolated organism from pediatric patients with UTIs. However, any organism that gains access to the urinary tract system may cause infection, including fungi (Candida species) and viruses

TNM system

The size of the breast tumor (T) and if it has grown into nearby areas Whether the cancer has reached nearby lymph nodes (N) Whether the cancer has metastasized (spread to other parts of the body) (M)

PMS and the DDx

The symptoms of PMS can be similar to or overlap with other conditions, including: Perimenopause Depression or anxiety Chronic fatigue syndrome Thyroid disease

Screening should assess for pelvic pain at or around

The time of menstruation

Breast cancer staging 3

The tumor is larger than 2 inches . It has spread to 4 to 9 axillary lymph nodes, or it has enlarged the internal mammary lymph nodes (N2). The cancer hasn't spread to distant sites (M0).

Breast cancer staging 2

The tumor is less than 2 inches or less across (or is not found) (T0 or T1) with micrometastases in 1 to 3 axillary lymph nodes (the cancer in the underarm lymph nodes is greater than 0.2mm across and/or more than 200 cells but is not larger than 2 mm)(N1mi). The cancer has not spread to distant sites (M0).

Breast cancer staging 1

The tumor less across than one inch (T1) and has not spread to lymph nodes (N0) or distant sites (M0).

PID treatment

Therapy should provide empiric, broad spectrum coverage of likely pathogens.

urinary analgesics

These agents are used to relieve burning, spasticity, and pain during voiding due to urinary tract infection. Phenazopyridine exerts local, topical anesthetic or analgesic action on urinary mucosa. It is used for symptomatic relief of pain, burning, urgency, frequency, and other discomforts arising from irritation of the lower urinary tract mucosa caused by infection, trauma, surgery, endoscopic procedures, passage of sound, or catheters. Analgesic action may reduce or eliminate the need for systemic analgesics

MOA of selective estrogen receptor modulators (SERMs)

These are drugs that can act as agonist on one tissue and antagonist at other tissues (offers the benefits with the least harmful side effects)

Presumptive Signs of Pregnancy

These are the softest and least objective signs. Can be caused by many other conditions besides pregnancy. ■ Amenorrhea. ■ Nausea/vomiting (most common in fi rst trimester in the morning, usually disappears by the second trimester). ■ Breast changes (swollen and tender). ■ Fatigue. ■ Urinary frequency. ■ Slight increase in body temperature.

medications

These data led to the recommendation that children with a febrile UTI should receive oral treatment with a second- or third-generation cephalosporin, amoxicillin clavulanate, or with sulfamethoxazole-trimethoprim (SMZ-TMP)

Breast cancer staging 0

This is ductal carcinoma in situ (DCIS), a pre-cancer of the breast. Many consider DCIS the earliest form of breast cancer. In DCIS, cancer cells are still within a duct and have not invaded deeper into the surrounding fatty breast tissue. Lobular carcinoma in situ (LCIS) sometimes also is classified as stage 0 breast cancer, but most oncologists believe it is not a true cancer or pre-cancer. Paget disease of the nipple (without an underlying tumor mass) is also stage 0. In all cases the cancer has not spread to lymph nodes or distant sites.

Bacterial vaginosis (not STI), trichomoniasis, vulvovaginal candidiasis (not STI)

Three infections most frequently associated with discharge

Vulvovaginitis Tx Secondary to Candida albicans

Topical "-conazole" creams/ointments (e.g. *nystatin*); *Oral fluconazole*

reduces menstrual bleeding.

Tranexamic acid (antifibrinolytic agent)

Perimenopause

Transitional years between reproductive and nonreproductive years. Tends to begin in 30s. Decreasing fertility

Current Indications for HRT

Treat symptoms of menopause; treat hot flashes and urogenital atrophy. Not sued as primary or secondary prevention of heart disease. Not first line for osteoporosis. Use lowest possible dose. Treat as soon as possible.

Ceftriaxone

Treatment for gonococcal conjunctivitis and disseminated gonococcal infection

Valacyclovir or Acyclovir

Treatment for initial HSV episode

Benzathine penicillin

Treatment for syphilis

Follow-up eval for surgical abortion

UPT F/U, no exam

• Emergency contraceptive pills (ECPs)

Ulipristal acetate (UPA) as a single dose (30 mg) Levonorgestrel (LNG) in 1-2 dose

Clinician Values Clarification

Understanding your own biases and professional responsibilities: -Ethical and legal mandate to provide "comprehensive care" -If cannot have "healthy detachment": refer to another provider

Time for a Manual Vaccuum Aspiration or Electromechanical Suction:

Up to 14 weeks

Frequency of prenatal visits

Up to 28 wks: q4wk 28-36 wks: q2wk >36 wks: qweek

Laboratory Testing

Urinalysis (Dipstick) Obtain mid-stream urine before gynecologic exam (minimizes contamination from vaginal discharge). Check protein, leukocytes, nitrite, blood, glucose. ■ Protein: trace and from 1+ to 4+ is abnormal. ■ If 20 weeks of gestation or more, rule out preeclampsia. ■ If proteinuria present, order 24-hour urine for protein and creatinine clearance. Liver Function Tests ALT, AST, bilirubin, and gamma glutamyl transpeptidase (GGT) remain the same except for alkaline phosphatase. Alkaline Phosphatase Alkaline phosphatase is expected to increase during pregnancy due to the growth of the fetal bones. Values higher in multiple gestation pregnancies. White Blood Cells The white blood cell (WBC) count is elevated throughout pregnancy especially during the third trimester. Leukocytosis with neutrophilia is "normal" during pregnancy (if it is not accompanied by signs of infection). The WBC may climb as high as 16 mm3. Hemoglobin and Hematocrit Both values go down during pregnancy due to hemodilution. Physiologic anemia of pregnancy is most obvious during the second to third trimesters. The hemoglobin value may be as low as 9.5 g/dL and the hematocrit value may go down to 28 g/dL (by the third trimester). Erythrocyte Sedimentation Rate Increases during pregnancy. Thyroid Function Tests Total T3 and total T4 are higher during pregnancy due to increase levels of thyroid-binding globulin (TBG). The TSH, free T3, and free T4 results remain unchanged. Serum Alpha-Fetoprotein (AFP) Manufactured by the liver of the fetus and mother. Majority of maternal AFP comes from the fetus (liver, fetal yolk sac, gastrointestinal tract). Low AFP Results ■ Mature maternal age is the most common risk factor for Down's syndrome (35-year-old or older female has a 1:200 chance). ■ If AFP is low, order the Triple Screen test (AFP, hCG, and estriol) or the Quadruple Screen test (AFP, hCG, estriol, inhibin-A) to evaluate for Down's syndrome (trisonomy 21). High AFP ■ Rule out neural tube defects or multiple gestation. Most common reason for a high AFP is pregnancy dating error. ■ If AFP is high, order the Triple Screen or the Quad Screen test and sonogram to rule out neural tube abnormalities. ■ Prevention of neural tube defects: folic acid 400 mcg per day (found in leafy green vegetables, fortifi ed cereals). Advise to take prenatal vitamins when planning to become pregnant to reduce risk. Triple Screen Test The Triple Test combines the AFP, beta hCG, and estriol serum level values. The hormone level results are used in a formula to figure out the risk for a Down's syndrome infant. Diagnostic test for genetic anomalies is chromosome testing. Quadruple Screen Test Combination of the Triple Screen hormones plus inhibin-A (hormone release by the placenta). The Triple or Quadruple Screen tests are more sensitive than the AFP alone (but higher rate of false positives). Gold standard test for genetic disorders is testing of fetal chromosomes/DNA. Screening for Genetic Disorders ■ Jewish-descent (Tay-Sachs disease) Tay-Sachs disease is a rare inherited disorder that progressively destroys nerve cells (neurons) in the brain and spinal cord. ■ Whites/Caucasians screen (cystic fibrosis) ■ African Americans (sickle cell anemia) ■ Tay-Sachs disease is a fatal neurological disease with no known cure. It is more common among Eastern Europeans of Jewish-descent (Ashkenazi Jews). Amniocentesis and Chorionic Villus Sampling (CVS) Chorionic villus sampling can be done earlier (10-12 weeks) than the amniocentesis (15-18 weeks). Specimens contain fetal cells. Fetal chromosomes/DNA are tested for abnormalities. Human Chorionic Gonadotropin (Beta hCG) Manufactured by the chorion by day 8 (develops into the placenta). High-quality urine home pregnancy tests (e.g., "First Response," EPT) can pick up pregnancy as early as the first missed period (2 weeks from conception). Higher levels with twins/multiple fetus. Doubling Time An important indicator of the viability of a pregnancy. Useful only in the first trimester; thereafter, it loses its predictive value (do not use after week 12). ■ Normal finding: Doubles every 48 hours during the fi rst 12 weeks (first trimester) in a normal pregnancy. ■ Ectopic pregnancy: Lower values than normal. Values increase slowly and do not double as expected. ■ Inevitable abortion: Values start decreasing rapidly. No doubling. Cervix is dilated. Vaginal Cultures ■ Group B strep (GBS) at 35 to 37 weeks. Swab vaginal introitus and rectum (insert up to anal sphincter) for C&S. If positive, treat with penicillin G 2.5 million units intravascularly (IV). ■ If penicillin allergic: use clindamycin or erythromycin instead. Sexually Transmitted Diseases (STDs) ■ Screen for HBsAg, anti-HCV, HIV, gonorrhea, chlamydia, syphilis, herpes virus type 1 and type 2. Titers ■ Check rubella titer. Check varicella titers (if no proof of infection).

Voiding Cystourethrography

VCUG is indicated if renal and bladder ultrasonography reveals hydronephrosis, scarring, or other findings that suggest either high-grade VUR or obstructive uropathy. VCUG should be performed if there is a recurrence of a febrile UTI, even if previous ultrasonographic examination findings were unremarkable

Threatened Abortion

Vaginal bleeding occurs but cervical os remains closed. Most of these cases will result in an ongoing pregnancy.

WHO Evacuation recommendation

WHO recommends evacuation 1st trimester

What is the jump start method?

Well suited if unprotected intercourse since LMP - Prescribe oral hormonal emergency contraception - Start COC, patch, ring that day - Use backup for 7 days

Diagnosis of Trichomonas

Wet prep will show trichomonads which are small motile cells w/flagella that will shake in place

When should indwelling catheters be changed?

When significant sludge or debris has accumulated and is causing reduced urinary catheter drainage by partially obstructing the catheter lumen

don't

When treating chlamydia, you ______ necessarily have to also treat for gonorrhea. Rely on the lab results.

Liver & renal diseases result in an inability to adequately clear estrogen,

Which allows biologically active, free- floating estrogen to increase . This build up impacts the HPOA, with resultant anovulation

what is common when stopping HRT use?

Withdrawal bleeding

Economic disparity gap

Women w/incomes below poverty level: >5x the unintended pregnancy rate of financially well-off women

Primary, Preventive Reproductive Health Care: ONE KEY QUESTION

Would you like to become pregnant in the next year?

Is acute uncomplicated cystitis treated differently in females?

Yes, generally it is a 3-day therapy. Longer term therapy needed for UTI's in men, pregnancy, elderly people with diabetes and children.

A diagnosis of infertility is not made until

a couple has attempted pregnancy for at least 12 months; thus evaluation is not initiated until this amount of time has elapsed

Ode

a form of lyric poetry — expressing emotion — usually addressed to someone or something, or it represents the poet's musings on that person or thing

Doing general examination it is important to note weight & BMI, acne, hirsutism, or alopecia that could indicate

a hyperandrogenic disorder

Personification

a metaphor where an animal, idea, or thing is described as if it were a person. "And this same flower that smiles today, Tomorrow will be dying." Robert Herrick

Thyroid abnormalities, such as enlargement, nodule, tenderness; and nipple discharge or visual changes that could indicate

a pituitary mass.

Pun

a play on words based on words having similar pronunciation or spelling but different meanings. "Bravery runs in my family." AR Ammons

Extended metaphor

a point-by-point presentation of one thing as though it were another. "Hope is the thing with feathers That perches in the soul, And sings the tune--without the words, And never stops at all." Emily Dickinson

Inflammatory carcinoma

a rare and very aggressive cancer which block the lymph vessela in the skin of the breast.

during a pelvic exam, you ask Mrs. K to Valsalva (strain). While doing this, a pouching is seen on the anterior wall of the vagina. This is indicative of: a. a cystocele b. a rectocele c. an enterocele d. a uterine prolapse

a. a cystocele

unilateral galactorrhea may be present with: a. an intraductal papilloma b. a woman who is lactating c. a ruptured breast implant d. pregnancy

a. an intraductal papilloma

in a premenopausal woman, the biggest heart attack risk factor is: a. cigarette smoking b. family hx c. sedentary lifestyle d. obesity

a. cigarette smoking

bone effects of decreased estrogen levels

accelerated resorption

Apostrophe

addressing an inanimate object as though it could answer. A direct address to something abstract, a thing, an animal, or an imaginary or absent person. "Leave me, O Heart."

Nonemergent AUB due to anovulation managed by

administration of COC

Risk of breast cancer increases

after 3 to 5 years of EPT

hot flash triggers (4)

alcohol, spicy food, facial heat, hot baths

perimenopausal FSH levels may vary, so only draw FSH levels if woman has (2)

amenorrhea or oligomenorrhea (and) menopausal symptoms

A breast examination can rule out the presence of galactorrhea which may indicate

an elevated prolactin level

metaphor

an implied comparison between two things that are not alike but have something important in common. One of the things is the writer's actual subject. "All the world's a stage/And all the men and women merely players." As You Like It. Personification and simile are both kinds of metaphors.

irregular periods and polymenorrhea indicate

anovulation

Factors that predispose women to the development of symptomatic VVC include ..., ...., .....

antibiotic use , pregnancy , and diabetes . Pregnancy and diabetes are both associated with a qualitative decrease in cell-mediated immunity

Dosing for Combipatch

apply twice weekly Estradiol 0.05mg/day Norethindrone 0.14mg and 0.25mg/day

Distribution of body fat also changes adipose tissue tends to accumulate at the hips and thighs in younger women known as Pear-shaped

as women age adipose tissue is distributed and begins to accumulate at the waist - Apple- shaped

which type of cyst fo the female reproductive system usually results in pain, redness, a perineal mass, and dyspareunia? a. ovarian cyst b. Bartholin's cyst c. Gardner's cyst d. Nabothian cyst

b. Bartholin's cyst

which blood tumor marker is highly specific to epithelial ovarian cancer? a. PSA b. CA 125 c. CA 15-3 d. CA 19-9

b. CA 125

Jane, age 27, is c/o of lower abdominal pain. After doing some lab studies, you find leukocytosis, an elevated erythrocyte sed rate and an elevated C-reactive protein level. Which is the most appropriate dx? a. ovarian cyst b. PID c. tubal pregnancy d. diverticulitis

b. PID

which type of breast cancer involves infiltration of the nipple epithelium and has an initial sx of itching or burning of the nipple? a. ductal cancer b. Paget's disease c. mammaryd uct ectasia d. fibroadenoma

b. Paget's disease

a 17 y/o female pat requests to start Depo-provera injections as her method of birth control. She discloses that she has had four sexual partners in the past year. Her LMP was 12 days ago and she had unprotected intercourse 3 days ago. The appropriate management for this pt would be to: a. administer the injection today b. advise her to use another method for now and return with her next menses c. give the injection after a negative pregnancy test and tell her to use condoms for the next 7 days d. give the injection and tell her to use a barrier method for 7 days

b. advise her to use another method for now and return with her next menses

dysfunctional uterine bleeding is usually associated with: a. pregnancy b. anovulation c. genital tumor d. inflammation

b. anovulation

emotional support is best given to the clt with a sexually transmitted infection by: a. offering many alternatives b. authentic active listening c. assuring the clt that everything will be okay d. emphasizing the duration of the disease

b. authentic active listening

You have just finished a Pap smear on Sally, age 39. during the wet mount, you see cells with bacteria adherent to the cell wall giving it a stippled, granular appearance. What do you suspect? a. candidiasis b. bacterial vaginosis c. trichomoniasis d. cervicitis

b. bacterial vaginosis

the Mobiluncus species is responsible for which sexually transmitted infection? a. condylomata acuminata b. bacterial vaginosis c. human papillomavirus d. lymphogranuloma venereum

b. bacterial vaginosis

J. has severe pain monthly with her fibrocystic breast disease. Which medication do you suggest she try? a. micronized estradiol (estrace) b. danazol (danocrine) c. paroxetine (paxil) d. venlafaxine (effexor)

b. danazol (danocrine)

the majority of breast carcinomas are found in which anatomical site in the breast? a. around the areola b. in the upper outer quadrant c. in the lower half of the breast d. toward the sternum

b. in the upper outer quadrant

2 presentations of osteoporosis

backache; spontaneous fracture or collapse of a vertebra

What is most common type of vaginitis?

bacterial vaginitis

What are the different types of vaginal infections?

bacterial vaginitis, vaginitis candidiasis "yeast infection", trichomonas, atrophic vaginitis

Which estrogen circulates before then after menopause?

before menopause: - estradiol after menopause: - estrone

the average age of menopause int he US is: a. 45 yrs b. 48 yrs c. 50 yrs d. 53 yrs

c. 50 yrs

L., age 43 comes to your office in tears, stating that last night she had unprotected sex and forgot to take her birth control pill. she wants to know about the morning -after pill. You tell her: a. if your period does not start at the schedule time, come back to see me b. I'll go ahead and order the estrogen-only postcoital contraception pill c. I'll go ahead and order the Yuzpe regimen d. I'll refer you to a gynecologist

c. I'll go ahead and order the Yuzpe regimen

first line tx for polycystic ovary syndrome is: a. a bilateral oophorectomy b. oral testosterone therapy c. a combination of diet modification, wt loss, and stress management d. a laparoscopy with a bilateral wedge resection

c. a combination of diet modification, wt loss, and stress management

which of the following drugs may diminish the effectiveness of oral contraceptives? a. beta blockers b. oral anticoagulants c. antibiotics d. oral hypoglycemis

c. antibiotics

human papillomavirus may lead to: a. pelvic inflammatory disease b. molluscum contagiosum c. cervical dysplasia d. genital herpes

c. cervical dysplasia

What are normal yeast vaginal flora?

candida albicans, glabrata, tropicalis

2nd most effective treatment for BV is ____________ 2% cream x 7 days OR 300mg 2xday x 7days

clindamycin

Chronic anovulatory AUB can be managed with?

cyclic progestogen therapy

Mrs. W. would like to schedule an appt to bring her 18 y/o daughter in to see you for her first gyn exam and Pap smear. the most appropriate reply would include: a. I would be happy to see your daughter and complete her gyn exam and Pap smear b. your daughter does not need a gyn exam and Pap at this time c. Your daughter only needs a gyn exam and Pap smear when she becomes sexually active d. I would be happy to see your daughter and complete her gyn exam; however, she does not need a Pap smear until the age of 21

d. I would be happy to see your daughter and complete her gyn exam; however, she does not need a Pap smear until the age of 21

Jane wants to use some form of birth control, but because she is getting married next year,s he wants to be able to stop the birth control method after the wedding and have her fertility restored almost immediately. Which method do you recommend for her? a. birth control pills b. vaginal ring c. depot-medroxyprogesterone acetate (DMPA) d. Lea's shield

d. Lea's shield

a vagina Ph of 4.2 is an expected finding in: a. a healthy prepubertal age girl b. a woman with trichomoniasis vaginalis c. a postmenopausal woman with atropic vaginitis d. a healthy woman of reproductive age

d. a healthy woman of reproductive age

what is the positionof the uterus when the cervix is on the anterior vaginal wall? a. midposition b. retroverted c. retroflexed d. anteverted

d. anteverted

the most common type of vaginal infection is: a. candidiasis b. trichomoniasis c. gonorrhea d. bacterial vaginosis

d. bacterial vaginosis

J. is nursing her 8 wk old baby and states that he is very irritable and sleeps poorly. What medication or substance do you ask her if she is taking or using? a. cimetidine b. ergotamine c. nicotine d. caffeine

d. caffeine

Hyperbole

deliberate exaggeration for emphasis or to produce a comic or ironic effect. Overstatement to make a point. "And I will love you still, my dear/when all the seas gang(go) dry." Robert Burns

What is yeast infection associated with?

diabetes, hygiene, immunosuppression, HIV/Aids, pregnancy, and antibiotic use

simile

direct comparison using as or like "Life, like a dome of many-colored glass/Stains the white radiance of Eternity." Percy Bysshe Shelley, "Adonis"

when do symptoms occurs in both PMS and PMDD

during the LUTEAL phase of menstrual cycle**** (ON TEST)

uterine bleeding that is different from the patient's normal and unrelated to organic cause

dysfunctional uterine bleeding

Contraindication for COC (Combined OC) use?

factor V Leiden mutation

Litotes

figure of speech consisting of an understatement in which an affirmative is expressed by negating its opposite. Remember Beowulf:)

Hot flashes are most frequent in the

first 5 to 7 years following menopause that can last for many more years in some women . Characterized by a measurable increase in skin temperature and conductance that is followed by a decrease in core body temperature . Some women experience a prodrome prior to a hot flash. They feel cold following hot flash owing to the reduction in core temperature

first and second line treatments with dysmenorrhea

first = NSAIDS; second = OCPs

Diabetes is more difficult to manage for women during perimenopause due to

fluctuations in hormone concentrations . Insulin resistance increases with reduced levels of estrogen, causing higher glucose levels . Progesterone changes have a converse effect on glucose, causing lower levels due to the increase in insulin sensitivity that accompanies falling progesterone concentrations

menopause involves the initial rise in this hormone

follicle-stimulating hormone

measures to take for abnormal uterine bleeding during puberty

give Fergon; NSAIDS; Premarin until bleeding stops; check von Willebrand factor

If the pregnancy is DESIRED

give: -supportive care -preconception counseling -referral for prenatal care be: -non-judgemental -non-directive -follow-up

Metrorrhagia:

heavy bleeding but irregular

Menorrhagia

heavy bleeding but regular

Menometrorrhagia

heavy prolong period and irregular

Illicit drugs have been shown to inhibit ovulation due to

hyperprolactinemia which results in amenorrhea

MRI can be done with secondary amenorrhea to rule out these 2 things

hypothalamic or pituitary lesions

4 causes of vaginitis

i. Bacterial vaginosis (50%) ii. Vulvovaginal candidiasis (30%) iii. Trichomoniasis (5-20%) iv. Postmenopausal atrophic vaginitis

Common symptoms of vaginitis

itching or burning, discharge, odor, dysuria, or pain

Lifestyle changes that can be made for hot flashes:

keep room cool dress in layers avoid hot/spicy foods take cool bath/shower exercise/lose weight decr caffeine/stop smoking

Aromatase Inhibitors (ALS)

lower the amount of estrogen in the body by stopping certain hormones from turning into estrogen. If estrogen levels are low enough, the tumor cannot grow

Median blood loss per month? And normal upper limit per month in a normal adult menstrual cycle?

median = 30 mL per month; Upper limit = 60-80 mL per month

Progestogen therapy for chronic anovulation (Drug name and dosage)

medroxyprogesterone acetate (Provera) 10 mg daily for 10 days Norethindrone 5mg 2x/day x 10 days Oral micronized progesterone (Prometrium) 200mg/day x 10 days

pt who need a 10-14 day abx course

o Children < 2 years old o Male adolescents o Febrile UTI o Known urinary tract anomalies o Recurrent UTI o Rx prophylactic abx until imaging obtained

pt who need a 5-7 day abx course

o Children >2 years old o Afebrile o No known urinary tract anomalies o No previous UTIs o Low risk of complications

Invasive breast cancer

occurs when cancer cells from inside the milk ducts or lobules break out into nearby breast tissue.

New example of vaginal estrogen: Osphena

ospemifeme: SERM specific for vagina, approved for dyspareunia due to vulvar and vaginal atrophy - not to be used with breast cancer or osteoporosis - can incr hot flashes

T-scores for osteopenia, osteoporosis, and severe osteoporosis in postmenopausal women

osteopenia = 1 to -2.5; osteoporosis = < -2.5; severe osteoporosis = <-2.5 with a fracture

Reduction of ____ is a benefit of estrogen replacement therapy, but estrogen should not be used to treat it.

osteoporosis

The basic simple diagnostic procedures that should be done with an initial evaluation include

ovulation detection and semen analysis

Clinical signs of Atrophic Vaginitis

pale, thin vaginal mucosa, smooth vaginal wall, excoriations, synechiae (scarring)

intermenstrual or post coital spotting, evaluate for

pathologic causes

Pregnant women with BV are at risk for

premature rupture of the membranes , preterm labor and delivery, chorioamnionitis, and postcesarean endometritis .

Pregnant women with trichomonas vaginitis are at increased risk for ...

premature rupture of the membranes and preterm delivery.

Understatement

presenting something as less important, awful, good, or whatever than it really is, usually for satire or humor. "The grave's a fine and private place, But one, I think do there embrace." Andrew Marvell, "To His Coy Mistress."

2 major advantages of hormone replacement therapy

prevention of bone loss and relief of menopausal symptoms

Does estrogen or progestin lead to endometrial atrophy/transformation

progestin

menorrhagia

prolonged (7 days) or excessive (80 mL) uterine bleeding;*regular intervals*(greater than 24 pads per day)

menometrorrhagia

prolonged uterine bleeding at *irregular* intervals

Anaphora

repeating an initial word or word to add emphasis. "'Whose herds with milk, whose fields with bread, Whose flocks supply him with attire." Alexander Pope

Vaginal Candidias (Yeast) statistics

second most common, represents 30% or 33% of vaginal infections

How do you diagnose using wet prep?

smear vaginal discharge onto slide, add saline or KOH, and view under 40X microscope

management of abnormal bleeding in perimenopausal smokers and nonsmoker

smokers = cyclic progestin to provide monthly withdrawal bleeding Nonsmokers = hormonal contraceptives for cycle control

early menopause is common in women who have a history of (5)

smoking, short cycles, nulliparity, type 1 diabetes, family history

How do you diagnose BV?

speculum exam: gray, white discharge on vaginal walls pH>4.5 Whiff test: apply 10% KOH which liberates amine cells; fishy odor Wet prep: shows clue cells

What does Estrogen therapy do for AUB?

stimulates endometrial proliferation; resolves bleeding.

Urinalysis

suggestive, not diagnostic 1. pH, blood, ketones, glucose, nitrites, leukocyte esterase, specific gravity 2. Findings: a. (+) nitrite & leukocyte esterase = likely UTI b. (+) nitrites only= probable UTI c. (+) leukocyte esterase only= nonspecific ***do not use UA to substitute for cx on UTI diagnosis -can ID febrile children who should receive abx while cx results are pending

Hot flashes occur concurrently with a

surge in LH levels

Angelique

tablets-take daily Estradiol 0.5mg or 1mg Drosperinone 0.25mg or 0.5mg

Dosing for Prempro

take daily CEE 0.3 mg, 0.45mg, 0.625mg Medroxyprogesterone 1.5mg, 2.5mg, 5mg

Women gain an average of 5 pounds at midlife. This increase is partly due to

the decrease in muscle mass that occurs with age and slows down the rate at which calories are burned and partly due to a decrease in activity that often accompanies midlife

What causes overflow incontinence?

the inability to empty because of an obstruction or loss of function (drugs/neuropathy).

Euphemism

the substitution of an inoffensive term for one considered offensively explicit. Using an indirect word or phrase for a more direct one. She passed away.

if patient having hot flashes, must rule out (3)

thyroid, tumors, autoimmune disorders

Dosing for Vivelle:

transdermal estradiol 14, 25, 37.5, 50, 60, 75 & 100 mcg/day

What is the most type of chronic urinary catheter?

transurethral

What is the role of progestogens in AUB?

treat chronic heavy bleeding due to anovulation.

24yo woman with green-yellow vaginal dc, vaginal erythema, petechial hemorrhages on cervix, numerous WBC, motile organisms on examination:

trichomoniasis. Tx with Flagyl

TV The parasite, which exists only in form......, is an anaerobe that has the ability to generate hydrogen to combine with oxygen to create an anaerobic environment. It often accompanies ....., which can be diagnosed in as many as 60% of patients with trichomonas vaginitis.

trophozoite BV

Noncyclic mastalgia

unilateral, localized pain that is sharp or burning; occurs frequently in women 40 years and older

Management

•Concomitant therapy may be necessary. •Acute hemorrhage: physician referral; medical management in hospital. •Estrogen therapy: stimulates endometrial proliferation; resolves bleeding.

Positive Signs of Pregnancy

■ Palpation of fetus by health provider. ■ Ultrasound and visualization of fetus. ■ Fetal heart tones (FHT) auscultated by health provider). - 10 to 12 weeks by Doppler/Doptone - 20 weeks by fetoscope/stethoscope

Preschoolers sx of UTI

(2 - 6 years) o MC to have classic sx o Dysuria, frequency, urgency o ABD/back pain (lower = bladder / upper = kidneys) o Strong smelling urine o New onset of incontinence - uncommon in kids (sexual abuse) or enuresis (nighttime incontinence) o May have GI sx: V/D, fever

School-aged children sx of UTI

(6 - 12 years) o MC to have classic sx same as preschoolers o Less likely to have GI sx though (younger peds have GI sx) V, fever

What happens to the musculoskeletal system during pregnancy?

(POSTURE) 1. Backward tilt of the torso 2. Relaxation of the ligaments which support the joints of the spinal column and pelvis: waddling gait (Dental) 1. Dental inflammation 2. Periodontal health can impact on perinatal health

What happens to the skin during pregnancy?

(Striae gravidarum: stretch marks) (Pigment changes) 1. Linea nigra: black line running from the umbilicus to the symphysis pubis 2. Chloasma: "mask of pregnancy", irregular spots or blotches of brown color on the face 3. Vascular markings: spider veins 4. Pigmentation of the breast

Discuss the physiological adaptions to the expectant mother's respiratory system during pregnancy. Why do women get SOB? How does the body compensate for this increased SOB?

*ADAPTS TO INCREASED O2 REQUIREMENTS 1. Diaphragm displaced upward in later pregnancy, thus 'shortness of breath' 2. Widening of the thoracic cage compensates to increase oxygen consumption

Women's Health Initiative: Estrogen-only arm

- 10739 women with hysterectomy - conjugated equine estrogens or placebo: * CHD-no change *breast cancer-no change *incr stroke *decr hip fx

Women's Health Initiative (study done by NIH)

- 16608 postmenopausal women aged 50-79 with intact uterus were included - mean age 63 yrs - active group received prempro-conjugated equine estrogens: 0.625 mg/d & medroxyprogesterone acetate 2.5 mg/d - study terminated early due to breast cancer risk

If 10,000 more women took estrogen & progesterone for a year, there would be:

- 7 more CHD events - 8 more breast cancers when used more than 5 years - 8 more PE, ischemic strokes - 6 fewer colorectal cancers - 5 fewer hip fx's

Metronidazole/Tinidazole MOA

*Inhibits nucleic acid synthesis* by disrupting the DNA of microbial cells -A drug of choice for *anaerobic Gram (-)*

PMS vs prementstrual exacerbation of mood or anxiety disorder

*PMS = -no history of depression or anxiety -symptoms in luteal phase of menstrual cycle *Depression/anxiety -prior episodes of major depression -history of gen anxiety -may meet criteria for luteal phase disorder ****Key diff = PMS symptoms come and go in distinct pattern, month after month

Diuretics

*Spironolactone -Aldosterone antagonist -Has been shown to be of benefit with bloating and water retention -Reduction of somatic and affective complaints -100mg/dl in the morning during 14-day luteal phase

metrorrhagia

*irregular* but frequent uterine bleeding between regular periods

What are the Healthy People 2020 goals for unintended pregnancy?

- decrease unintended pregnancy by 10% -decrease <18 mos pregnancy spacing

Take home points

- do not use for prevention - if urogenital sx's, use topical - if mod-severe vasomotor sx's, use oral/transdermal - always use lowest dose possible for shortest amt time possible (less than 5 years) - evaluate annually - consider tapering therapy to avoid hot flashes (instead of abruptly stopping)

Role of progestins in therapy:

- endometrial protection in woman with intact uterus - must be taken 12-14 days each month (not needed for the entire month)

Combination regimens:

- estrogen given each day - progestin is added for 12-14 days: * can be combine therapy (estro+progest): more BTB assoc early on b/c giving her back her hormones - if bleeding very heavy like a period-send to PCP (possibility endometrial cancer) * can be sequential (when giving progest, there is no estrogen)

Hot flashes

- hot flashes or vasomotor sx's occur in 80% women - at night, can result in night sweats - involves feeling of heat around chest & face, becoming generalized --> usually lasts a few minutes --> may be followed by chills, shivering, anxiety-->may have many episodes per day - can last 4-5 years post-menopause; 9% will have them past age 70 (variable across cultures)

History of HRT:

- in 1980s/90s: women took HRT for years (Ca and estrogen) - goal was to prevent osteoporosis & CHD - observational studies had shown a decr risk of fx & improved lipid panels with estrogen after menopause - thought that estrogen deficiency might incr risk for dementia

Vaginal Estrogens:

- indicated for tx atrophic vaginitis - available in creams, tablets, rings - low systemic absorption - progestin NOT NEEDED in pt with intact uterus - however Femring does achieve systemic concentrations of progestin & is indicated for mod-severe vasomotor sx's

What are side effects of progestin?

- irritability - depression - headache - premenstrual sx's - mood swings, bloating, fluid retention, sleep disturbance

Vaginal Symptoms

- lining of vagina and urethra is estrogen-dependent (has estrogen receptors) - changes when estrogen levels change - epithelium thins, leading to dryness, itching, dyspareunia, urinary infections, incontinence (overactive & stress incontinence) - 47% women 3 yrs past menopause will experience dryness - treatment options: vaginal lubricants, vaginal estrogen, oral estrogen

Cause of Menopause:

- loss of ovarian function & subsequent hormonal deficiency - causes: * normal process of aging * surgery (total abdominal hysterectomy, bilateral oophorectomy-pushes into early menopause-no more estradiol) * chemotherapy * pelvic irradiation - postmenopausal ovary no longer produces estradiol or progesterone

What are side effects of estrogen?

- nausea - headache - breast tenderness - vaginal bleeding - gall bladder disease - ACHES applies to HRT and OCPs

Perimenopause: when does it start and what are the manifestations

- occurs after primary reproductive years but before menopause (~45) - begins about 4 yrs before final menstrual period - manifestations: irreg periods, hormonal fluctuations (don't measure hormones this time), hot flashes, sleep disturbances, vaginal dryness * if taking OCP, may not notices some of these changes

What are some things to think about regarding dosage forms of estrogen?

- oral estrogen undergoes first pass-need higher dose to have intended effect (can incr HLD & TG's) - transdermal doses lower than oral & avoid first pass - transdermal estrogen does NOT impact TG or HDL and may have less risk DVT/PE

What to think about when giving estrogen to a woman?

- pts with intact uterus must ALSO take progesterone pdt to prevent uterine hyperplasia and cancer** - estrogen is given continuously to avoid hot flashes - HRT most effective for tx vasomotor and vaginal sx's **do not use HRT for prevention**

CVD and menopause

- risk of CHD incr after menopause - LDL incr after menopause by a mean of 6% - NCEP recognizes menopause as a risk factor for CHD - women tend to gain fat mass and lose muscle mass - treatment of estrogen does NOT decr risk

Estrogen therapy is individualized, what should be assessed before starting?

- severity of menopausal sx's - CHD risk - DVT/PE risk - breast cancer risk * choose lowest effective dose for shortest duration needed for effective sx control (women with hysterectomy need highest dose)

timeline for Follow-up culture in patients with recurrent UTI

- to gain better control of infection 1st:Once monthly for 3 months then: Once every 3 months for 1 year finally: Yearly for 2-3 years

Definition of PMS

-"the cyclic occurrence of symptoms that are of sufficient severity to interfere with some aspects of life and that appear with consistent and predictable relationship to the menses."

Dilation and Evacuation- late

-20-24 wks -Cervical Dilation: osmotic dilators w/ misoprostol prn -Amniotic infusion of Digoxin, forceps, curette, vacuum x 2 -Paracervical block +General anesthesia -Post Op Abx -Contraception: Immediate (to w/in 5d) -2-3 visits

Women report a variety of reasons for contraceptive nonuse

-40% problems accessing or using methods -19% infrequent sex -18% do not care if pregnancy occurs -7% underestimate pregnancy risk -other

Diagnostic criteria for PMDD

-At least five premenstrual symptoms -Seriously interfere with patients daily life *At least one of the following: Depressed mood Marked anxiety Marked affective lability Marked irritability

Dietary suggestions

-Complex carbohydrates, such as whole-grain breads and cereals packed with fiber -fiber can keep blood sugar which may reduce mood swings and food cravings -whole-grain products also contain the PMS-fighting B vitamins thiamine and riboflavin -vegetarian diets may help increase SHBG which inactivate estrogen thus improving PMS

POP

-Does not consistently suppress ovulation. It does thicken endocervical mucus. Advantages include failure 1-4% (less effective than COC), but nausea rate lower (d/t lack of estrogen), ok for lactating women and does not alter quality of quant of breast milk. -Disadvantage: bleeding irregularity.

Yasmin

-Drospirenone found in Yaz, is an analog of aldosterone antagonist and has potassium-sparing qualities. Use with caution in hepatic/renal dysfunction or with concomitant use of ACE, ARB, salt substitute, or K sparing diuretic.

OMT

-Dysfunctions causing intensified congestion in the lesser pelvis can be reduced causing less pain -Treatment of somatic dysfunctions of the thoraco-lumbar spine T10 - L2 have an affect -This may also positively influence pain and the symptoms of PMS.

Stages of labor

-Early: may last 2-3 days, mild-to-mod contractions last 30-45 seconds and are 5-20 min apart (starting and stopping). Dilates to about 3 cm, membranes intact. -First stage of active labor: starts when cervix is 3-4cm and is complete when fully dilated. Go to hospital when contractions occur q5min and last up to 1 min. -Second stage is the actual birth. -Third: delivery of placenta.

What are the four principle components necessary for continence?

-Effective functioning of the urinary tract -Adequate cognition -Motivation -Environment

Menopause continued

-Endometrial ca with unopposed strogen use is considerable. Increased breast ca with HT use. -Estrogen reduces some dementia. -Do not give estrogen in unexplained vag bleeding, liver dx, thrombotic dx, endometrial/breast ca. -SERM: tamoxifen locks out estrogens effects in breasts. Useful in prevention of breast ca, small increase in endometrial ca. Raloxifene (Evista) has activity of CV system and bone but none in breast, uterine, or brain. Lowers cholesterol and prevents osteoporosis.

Selective seritonin reuptake inhibitors

-FDA-Approved SSRIs for Premenstrusal Dysphoric Disorder-initial drug for severe sx 1) Fluoxetine (Sarafem) = during the luteal phase provides significant improvement in 50% of patients 2) Sertraline (Zoloft) 3) Paroxetine (Paxil CR)

How to rule out an ectopic pregnancy

-HIGH RISK: HX of ectopic, tubal surgery, in utero DES exposure -ask: "Have you had any bleeding, unilateral abdominal or pelvic pain, shoulder pain, dizziness/fainting?" - Exam indicates adnexal mass If yes, needs stat BhCG, 36-48 hours apart to determine viability v. pregnancy loss v. ectopic -BhCG: doubles q 48-72 hrs -Discriminatory zone: 1500-2000 , gest sac can be seen on sono (by 35d LMP) If BhCG=1500-2000 and no sac on sono: R/O ECTOPIC

Direct doming of diaphragm

-Indication: Decreased respiratory excursion on screening exam -Diagnosis :assessment of decreased diaphragm motion during exhalation -Stand at the side of the supine patient facing the head -Place both hands on the anterolateral aspect of the inferior border of the ribs with the thumbs placed medial to the costal margins directly inferior to the xiphoid process -Have the patient inhale and exhale deeply. During exhalation press the thumbs gently toward the table following the motion of the diaphragm -Hold the position during inhalation -Repeat through several respiratory cycles until the diaphragm moves fully(domes) at the end of exhalation

Pelvis diaphragm release

-Indication: lymphatic congestion in the pelvis and lower extremity and visceral disease ie dysmenorrhea, constipation -Diagnosis: pelvic diaphragm has descended and is restricted in moving superiorly, Inhalation somatic dysfunction. -Sit at the side of the supine patient facing the patient's head. Place the fingers of your medial hand into the ischiorectal fossa on the medial side of the ischial tuberosity.Ask the patient to inhale then exhale fully. -With exhalation, encourage diaphragmatic excursion by pushing the fingers superior and slightly anterior. -Maintain this position. Ask the patient to inhale. Be sure to hold ground, not allowing the pelvic diaphragm to descend. -Ask patient to exhale and continue to follow the diaphragm superiorly. -Treat through 4-5 cycles until the pelvic diaphragm releases.

Trichomonas is associated with?

-preterm labor, LBW, and premature rupture of membranes -inc. risk of HIV transmission -asymptomatic in men so reinfection in women from man is common

Supine thoracic inlet release

-Indications: Any condition involving lymphatic congestion -Diagnosis: fascial restriction in the supraclavicular fossa -Procedure: Physician sits on the side of the supine patient facing the head. The patients arm is abducted to 90 , externally rotated and flexed at the elbow to 90. Place the patients elbow on the knee, support the arm with one hand and place the other hand in the supraclavicular fossa. -Then an inferior /anterior pressure is applied to the clavicle while moving the patient's arm from external to internal rotation. -This pressure is maintained as the arm is brought back into external rotation. -This can be repeated until release is palpated.

What autonomic activity causes urination?

-Inhibition of sympathetic -Stimulation of parasympathetic

Tx of TV ....... is the drug of choice for treatment of vaginal trichomoniasis. Both a single-dose (2 g orally) and a multidose (500 mg twice daily for 7 days) regimen are highly effective and have cure rates of about 95%. The ...... should also be treated. ....... ..., although highly effective for the treatment of BV, should not be used for the treatment of vaginal trichomoniasis.

-Metronidazole -sexual partner -Metronidazole gel

HPV

-Most likely to cause condyloma acuminatum: 6/11 -Associated with cervical cancer: 16/18 Tx with podofilox, imiquimod, trichloroacetic acid or cryotherapy.

Emergency Contraception

-Multiple methods: COCs, POPs, CU-IUDs. -Oral tx will not interrupt established pregnancy or increase risk of early pregnancy loss. -CU-IUD (ParaGard T) can be inserted within 5 days as form of emergency contraception. Risk of upper reproductive tract infections. Menstrual bleeding should be expected within 3-4 weeks, if not- pregnancy test!

Age distribution

-PMS = peaks at age 36 then slowly declines -PMDD = increased at age 35 because more people start coming in more for this (so just inc reporting); spike at age 45 b/c perimenopausal state

RFs

-Seen at any age in menstruating females-treatment frequently sought after age 30 -Diagnosed in women with higher levels of stress -No cultural or socioeconomic association

When can insertion of a pessary be useful?

-Severe stress incontinence in a non-surgical candidate

Surgical Abortion- Contraindications

-Uncontrolled DM, HTN -Current Respiratory Compromise ->300 lbs (rec hospital) -Severe anemia (HCT <29) ->14wks: Hx of c-sec or myomectomy AND placenta previa or overlying scar (must be done in hospital) - >24 wks

UI types

-Urge: detrusor overactivity, strong sensation cannot be suppressed. Tx- bladder stretching, teaching -Stress: weakness of pelvic floor muscles, Kegels help. -Urethral obstruction: dribbling post-void, most common in men with BPH.

Epidemiology

-Various reports place the prevalence of PMS at about 80% of women affected at one time or another -Severe symptoms (PMDD) occurs in about 3% - 8% of women between the ages of 18 and 48

The diagnosis of cervicitis

-a purulent endocervical discharge, generally yellow or green in color and referred to as "mucopus " -"mucopus " =increased number of neutrophils -The presence of intracellular gram-negative diplococci = gonococcal endocervicitis , If negative for gonococci= chlamydial cervicitis -nuclei acid amplification tests (prefer) - 50% is unknown

Risks for preeclampsia

-age <16 years, >40 years -collagen vascular dx -first pregnancy with new partner -pregestational DM, -HTN -Multiple gestation pregnancy.

Diagnosis

-asymptomatic. - associated with a profuse, purulent, malodorous vaginal discharge that may be accompanied by vulvar pruritus. -a patchy vaginal erythema and colpitis macularis ("strawberry" cervix) (high concentration of organism) - pH higher than 5.0 -Microscopy of the secretions=motile trichomonads and increased numbers of leukocytes -Clue cells may be present because of the common association with BV. -The whiff test may be positive.

UTI in 38 wk pregnant woman. Prescribe:

-augmentin -Tx with 3-7 day course when asymptomatic. Cat B: amoxicillin, cephalexin, fepodoxime, cefixime, Augmentin and nitrofurantoin. *Avoid Nitrofurantoin after 36th wk d/t potential risk for hemolysis if G6PD-deficient. -When symptomatic tx for 7 days, then do monthly screening urine cx.

PID

-can be caused by many organisms, 60% of the time STIs. Abd pain, abnormal vaginal dc, dyspareunia, fever, GI upset and abnormal vaginal bleeding. Tx option differ according to presentation. When ill, pregnant need admission. Ceftriaxone 250mg IM once follow by doxycycline 100mg BID x 2 weeks is highly effective. If severe pencillin allergy, try Ofloxacin 400mg BID with metronidazole 500mg BID x 2 weeks.

Adoption

-rare, <1% -open v closed adoptions -laws vary by state -public or private agencies

Physiologic changes during pregnancy

-color and texture change in cervix, becoming cyanotic (Chadwick's sign) and less firm (Goodell's sign). -physiologic glucosuria and proteinuria occur due in part to increase in GFR and a resulting inability of renal tubules to reabsorb glucose and protein.

Abortion

-common: 42% -medication- 9 weeks -surgical (up to 24 weeks) -laws vary by state -medicaid v private providers

Midol

-contains acetaminophen + caffeine

Candida are ..... fungi existing as ...., which are responsible for transmission and symptomatic colonization, and as mycelia, which result from blastospore germination and enhance colonization and facilitate tissue invasion.

-dimorphic -blastospores - -

women with BV are at increased risk for p......... .... ... , .... ..... , ..... .... .... after hysterectomy ,and abnormal cervical cytology .

-elvic inflammatory disease (PID) postabortal PID postoperative cuff infections

Symptoms of BV?

-heavy grayish disharge -fishy odor -itching (pruritis) and irritation

What causes urge incontinence?

-irritability of the trigone of the bladder -causes involuntary bladder contractions

What conditions cause urge incontinence?

-local/surrounding infection (UTI, etc) -Motor urge incontinence

Statistics of Trichomonas

-miscroscopic parasite -50 to 75% of prostitutes -5 million cases in U.S./yr -5 to 15% of women in gynecology clinics -32% of women in STD clinics

Common symptoms of PMS

-mood = depression, sadness, irritability, anxiety, tearful, anger, lonely, food cravings MC =fatigue, irritability, labile mood with sadness or anger, depression -cognitive = mood swings, diff concentrating, dec efficiency, confusion, forgetfulness, angry -physical = breast tenderness or swelling, wt gain, abd bloating, nausea, acne, hot flushes, constipation, palpitations MC = breast tenderness and abd bloating

What is detrusor hyperreflexia

-motor urge incontinence -An uninhibited bladder caused by stoke, dementia, or parkinson's

COC

-noncontraceptive benefits include lower rates of benign breast tumors, dysmenorrhea, decreased IDA, endometrial, ovarian and breast cancers. -BTB can be minimized by taking within same 4-hr period every day. Not harmful and doesn't indicate lesser contraceptive benefit. -Nausea common in first few months. -Causes reduction in therapeutic levels of antiepileptics such as phenytoin, carbamazepine, phenobarbital, primidone. -Depo-provera works and may cause reduction in seizures.

How is vaginal homeostasis maintained?

-normal estrogen levels w/high glycogen content -lactobacillus uses glycogen to produce lactic acid which keeps pH level low -bacteria don't grow well in low pH

UI

-obtain urinalysis and urine culture -Urge incont. is most common in elderly. Behavioral tx and pharm tx may be needed. Terodiline is a selective muscarinic receptor antagonist that blocks bladder receptors and limits bladder contraction. Decreases micturation and incont episodes. Oxybutynin is a nonselective muscarine receptor antagonist that blocks receptors in bladder and oral cavity- AE include dry mouth and constipation.

Characteristics of Atrophic Vaginitis

-occurs in etrogen depleted state (menopause) -ph > 5 -non-lactobacillus bacteria count -associated with vaginal atrophy -more often in smokers

HVLA

-of T10-L2 -Subject lies on their back, practitioner places a towel between subjects elbows and practitioners epigastric area, practitioner supports the subjects‟ head and neck and places their hand as a fulcrum under the subjects lower thoracic/upper lumbar vertebra (T10-L2). -Practitioner localizes fulcrum focus by transferring minimal body weight over targeted segment, subject is asked to inhale and exhale -At end of exhalation the practitioner applied a HVLA thrust to achieve joint cavitation.

Diagnosis of V.C.?

-pH < 5 -satelite lesions: erythema -wet prep -excoriations from scratching -curdish discharge in vaginal vault

First prenatal visit

-pap smear, CBC, rubella titer, blood type, antibody screen, VDRL/RPR, HIV, GC/chlamydia, PPD, Hg electrophoresis (consider), HBsAg, UA, urine C&S

Sacral rocking

-parasymp inhibition -Patient prone, physician stands at bedside -Palms over sacrum longitudinally, one hand over the other -Lean over sacrum, and apply pressure anteriorly over base, and rock sacrum anteriorly at the base -Apply pressure anteriorly over the apex of the sacrum, rocking the sacrum posteriorly at the base -Continue with slow gentle rocking alternating with first anterior pressure over the base and then over the apex -Continue for 30 seconds to a minute or until tissue texture change signals release of somatic dysfunction -Recheck sacral motion

Because of the sexually transmitted nature of trichomonas vaginitis, women with this infection ....... ...... .... ...., particularly Neisseria gonorrhoeae and Chlamydia trachomatis. ......... for syphilis and human immunodeficiency virus (HIV) infection should also be considered

-should be tested for other sexually transmitted diseases (STDs) -Serologic testing

What are the cardinal symptoms of overflow incontinence?

-straining to urinate -decreased flow rate -size of stream -dribbling -hesitancy

Trichomonas Vaginitis (TV) Trichomonas vaginitis is caused by .......... The transmission rate is .....; 70% of men contract the disease after a single exposure to an infected woman.

-the sexually transmitted, flagellated parasite, Trichomonas vaginalis -high

Symptoms of V.C. (yeast) infection?

-thick white discharge "cottage cheese" -erythema -dysuria -itching, burning -vulver edema (swelling)

Etiology of PMS and PMDD

-unclear -related to: -ovarian steroid fluctuation -dec serotonin (CNS) -genetic predisposition -social expectations

Supportive therapy

-understanding symptoms of PMS -relaxation training -lifestyle and nutrition guidelines *encourage them to inc activity level -exercise, esp aerobic varieties, inc endorphins, which improves mood and reduces fatigue *Avoid: -salt (inc bloating) -caffeine (inc irritability) -sugar (makes cravings worse) -alcohol (afefcts mood and NRG)

What labs should be run for primary care workup of urinary incontinence?

-urinalysis -urine culture

The ectocervical epithelium can become inflamed by the same micro-organisms that are responsible for ...... In fact, the ectocervical squamous epithelium is an extension of and is continuous with the ......... Trichomonas, candida, and HSV can cause inflammation of the ectocervix. Conversely, ....... and ...... infect only the glandular epithelium .

-vaginitis -vaginal epithelium -N. gonorrhoeae, C. trachomatis

Dx of VVC

-vulvar pruritus associated with a vaginal discharge=cottage cheese. - discharge= vary from watery to homogeneously thick -Vaginal soreness, dyspareunia, vulvar burning, and irritation -External dysuria ("splash" dysuria) during micturition=exposure of the inflamed vulvar and vestibular epithelium to urine. -erythema and edema of the labia and vulvar skin -Discrete pustulopapular peripheral lesions - vagina may be erythematous with an adherent, whitish discharge -normal pH (<4.5). -budding yeast forms or mycelia, appear in as many as 80% of cases. increase in the number of inflammatory cells in severe cases. -The whiff test is negative.

What changes related to aging cause stress incontinence?

-weakness of the pelvic floor musculature -Thinning and atrophy of the perineal and vaginal tissues

For diagnosis of PMDD

. 5 or more symptoms, including affective & physical symptoms, are present during the week before menses & are absent in the follicular phase . 1 of the symptoms is iritability, depressed mood, anxiety or affective lability . Symptoms markedly interfere with occupational or social functioning . Symptoms are not due to an exacerbation of another disorder . Preceding criteria have been confirmed by prospective daily rating over at least menstrual cycles

Hypothyroidism can cause either in

. Amenorrhea- if hypothyroidism is chronic . Menorrhagia- if hypothyroidism is recent

Differential diagnoses that have symptoms similar to perimenopause & menopause

. Anemia- fatigue and cognitive change . Pregnancy- irregular bleeding . Arrythmias- fatigue and palpitations . Diabetes- fatigue and hot flashes or heat intolerance . Hyperthyroidism- sleep disturbance, insomnia, nervousness, irritability, heat intolerance . Hypothyroidism- fatigue, dry skin, cognitive problems

Lifestyle causes

. Athletic women are at risk for oligomenorrhea & amenorrhea, as are women who have anorexia & eating disorders . It is not the fact of exercise itself that causes the amenorrhea but rather the specific type of exercise . The pathophysiology of exercise- induced amenorrhea is complex and is probably due to the combination of low body fat and diminished secretion of GnRH

Candidates for endometrial ablation

. Cancer has been ruled out . No previous myomectomy . Non distorted uterine cavity . Past childbearing . Refractory to medical therapy

Outflow tract causes of AUB

. Congenital or structural abnormalities:no vaginal orifice, absent uterus or cervix . Rare

Genital tract etiologies of AUB

. Endometrial or cervical CA- uterine bleeding that occurs in women who are postmenopausal is always abnormal & needs to r/o endometrial CA . STIs: chlamydia, gonorrhea, endometritis . Fibroids, adenomyosis, cervical polyps . Trauma to genital tract: tampon, sexual assault

HT protocols and formulation

. Estrogen therapy has been prescribed exclusively for women who have had a hysterectomy - 1975 report suggested that unopposed estrogen increases risk for endoemetrial hyperplasia & cancer . Estrogen-progestogen therapy - can be taken sequentially or continuously. Side effect: withdrawal bleed . Estrogens . Progestogens . Estrogen-androgen therapy - theorized to improve loss of libido

This condition is generally caused by 3 hormonal imbalances

. Estrogen withdrawal . Estrogen breakthrough . Progesterone breakthrough

Anovulatory woman is always

. In the follicular phase of the ovarian cycle . In the proliferative phase of endometrial cycle . There is no luteal or secretory phase because there is no ovulation or cycle

Vaginal lubricants and moisturizers

. KY . Caution against oil-based products such as petroleum jelly- exception is vitamin E oil, provides relief for vaginal dryness without interfering with condom or diaphragm function and rarely irritates tissues

Contraindications to endometrial ablation

. Known or suspected uterine cancer . Uterine hyperplasia . Thin myometrium . Intrauterine device . Pregnancy . Previous classical cesarean birth . Pelvic, uterine, cervical, vaginal infections . Uterus sounds less than 4 cm or uterus sounds outside device parameters . Disorders of mullerian fusion/ absorption

NSAID therapy for menorrhagia

. Mefenamic acid 500 mg tid . Ibuprofen 600 mg tid . Naproxen 550 mg loading dose then 275 mg q 6 hrs

Laboratory assessments

. No specific tests for CPPD, PMS, or PMM . Blood tests to identify anemia, thyroid disorders, DM, hypoglycemia . Ovarian hormone testing is necessary unless premature menopause (before age 40) is suspected

Hysterosalpingogram

. Performed 2 to 5 days after menstruation and to avoid interference from menstrual tissue and disruption of potential fertilization and implantation . In a normal HSG, the contrast travels unobstructed through the uterus and into the fallopian tubes . Thus this test indicates whether the fallopian tubes are patent or whether a structural abnormality is present in the uterus or tubes

Women reaching midlife

. Perimenopausal years: Ages 35 to 50 . Menopause ages 50 to 60 years

Distinguishing perimenstrual symptoms & discomfort patterns across 3 menstrual cycle phases:

. Premenstrual phase: up to 14 days prior to the onset of menses . Menstrual phase: days during menses . Postmenstrual phase: after menses & before ovulation

Prevention of infertility

. Prevent STI's - to seek care early to prevent PID and subsequent infertility . Certain contraceptive methods may protect future fertility by decreasing the risk of PID, ectopic pregnancies, endometriosis . Some environmental factors may prevent infertility- more definitive studies needed

Pharmacologic Symptom Management

. SSRI - Prozac , Zoloft may be taken daily or intermittently during the premenstrual phase - for PMDD, PMS . Anxiolytic - Alprazolam not recommended by ACOG. Short- term use only, should not exceed >8 weeks without medical/ psychiatric eval . Hormonal - micronized progesterone . Duretics - spironolactone for bloating . Chinese medicine . Dietary supplements - calcium, vitamin D, B6 for PMS . Herbal/Botanic - VITEX/ chaste tree berry for PMS - evening primrose oil for PMS - Sam-e for depression

Four processes that women experience during menopause were identified: ("integrating a new me" theoretical model)

. Turning into me . Facing a paradox of feelings: both a (+)& (-) feelings . Contrasting impressions: conflicting information . Making adjustments: changes

Transvaginal ultrasound and hysteroscopy

. Ultrasound can help identify uterine factors associated with infertility such as fibroids and endometrial polyps . Hysteroscopy can be used for definitive diagnosis and treatment of intrauterine conditions causing infertility

Intrauterine insemination (IUI)

. insemination procedures begin with a man masturbating to collect the semen, which is then processed in a laboratory to remove the seminal plasma, creating a more highly concentrated specimen of motile sperm

Factors that may affect the age of menopause have been studied such as

. parity, . age at menarche, . obesity, . height, and . oral contraceptive use . Ethnicity

pediatric UTIs Prevention

1. Avoid unneeded use of abx for URIs and OM -abx alter GI and periurethral flora >> dec natural defenses against pathogenic agents 2. tx to prevent dysfunction -Esp. assoc. w/ posturing >> urethrovesical reflux and UTI recurrence 3. Consider circumcision of male neonates 4. Consider prophylactic abx to avoid scarring 5. Good hygiene: Front-to-back wiping - Avoid of bubble baths and chemical irritants - Avoid of tight clothing - Avoid of correction of constipation

when to use Fluoroquinolones

1. effective for E. coli and saprophyticus -Ciprofloxacin -Rare resistance, but increasing 2. treat complicated UTI and pyelonephritis in children -Safety still under investigation 3. Pseudomonas aeruginosa and Multi drug resistance gram negative bacteria 4. Never first line treatment

Indications for Hospitilization

1. Bacteremia or Sepsis 2. Signs of urinary tract obstruction 3. Signs of significant underlying disease -Toxic, dehydrated: "am I going to worry about this kid" 4. Unable to tolerate adequate PO fluids or medications 5. Failure to respond to outpatient treatment 6. All infants younger than 3 months with febrile UTI (presumed UTI) 7. Immunocompromised patient 8. Lack of adequate outpatient follow-up

four ways to collect a urine sample

1. Bag sample- stick bag over penis or female genitalia to collect sample 2. Clean catch midstream (CCMS): -if pt is old enough to understand and cooperate -contamination d/t improper procedure. Most preferred method 3. Urethral catheterization • Most performed method when accurate cx needed 4. Suprapubic aspiration- needle into suprapubic abdomen into bladder to get a sample- must have a palpable bladder to do this • INVASIVE, but most reliable sample • GOLD STANDARD -

First Line Treatment for Children and Adolescents:

1. Begin with 2nd or 3rd gen. cephalosporins -MC- Cefixime -Cefproxil, cefotaxime, ceftriaxone, cefpodoxime - no resistance present 2. OR an aminoglycoside -Gentamicin, Amikacin: some resistance present 3. Does not cover enterococcus -Catherized patients, urinary tract instrumentation, anatomical abnormality -Add amoxicillin or ampicillin in these cases

Indications for dx evaluation

1. Boys of any age with a first UTI 2. Girls < 3 years with a first UTI 3. Children of any age with a febrile UTI 4. Children with recurrent UTIs, not previously imaged 5. First UTI in pt with a FH: -Renal disease, Abnormal voiding patterns -Poor growth, HTN and -Urinary tract abnormalities

Concerns with HRT

1. Cancer-- breast cancer (contraindicated if high risk for breast cancer such as family hx) endometrial cancer can be prevented if given progesterone 2. CVD-- given within the therapeutic window to avoid CVD risks, contraindicated in women who have hx of hypercoaguability due to estrogen effects in increasing blood clotting factors 3. Cholelithiasis: 2-3x fold increase due to the lowering cholesterol ability

pediatric UTIs Complications

1. Dehydration: MC complication 2. Vesicoureteral reflux dt Cystitis and may worsen preexisting reflux 3. Pyelonephritis dt reflux -2° to untx simple cystitis 4. Renal Scarring: occurs in 5-10% of kids w/ sx UTI and fever 5. Lobar inflammation or renal abscess 6. Generalized Bacteremia or sepsis dt UTI -1-3 month olds-30% sepsis ->3 months- 5% sepsis 7. HTN in more severe cases 8. Renal insufficiency or ESRD in more severe cases 9. Renal failure in severe cases

What changes occur in the breasts during pregnancy?

1. Distinct increased nipple sensation/tenderness in some women 2. Larger, firmer, more tender 3. Increased pigmentation, especially areola 4. Veins beneath the skin become more prominent 5. Discharge: around 4-6 months, thin, yellowish = colostrum

What are the PROBABLE (Objective) signs and symptoms of pregnancy?

1. Enlargement of abdomen 2. Changes in size, shape, consistency of uterus 3. Fetal outline distinguished by abdominal palpation 4. Braxton hicks contractions (Irregular Contractions early in pregnancy) 5. Positive pregnancy test 6. + Goodell's sign, softening of cervix 7. + Hegar's sign, softening of isthmus of cervix 8. + Chadwick's sign, deepened color of cervix

Dx of BV

1. Homogeneous vaginal discharge (color and amount may vary) 2. Amine (fishy) odor when potassium hydroxide (KOH) solution is added to vaginal secretions (commonly called the "whiff test") 3. Presence of clue cells (greater than 20%) on microscopy* 4. Vaginal pH greater than 4.5 (should be 3.8-4.2)

Discuss the adaptions to the cardiovascular system of the expectant mother.

1. Increased cardiac output by 30-50% in 1st 12 weeks 2. Increased heart rate (10-15 bpm) from 14-20 weeks 3. Decreased BP due to arteriole vasodilation from decreased sensitivity to angiotensins 4. Increased rise in diaphragm displaces heart into more transverse position: apex moves up and to the left 5. Systolic murmur due to increased flow rate and decreased blood viscosity 6. May hear S3 after 20 weeks 7. Premature beats and palpitations: due to increased steroid hormones 8. Supine hypotension syndrome

What happens to the urinary system during pregnancy?

1. Increased frequency, most pronounced in 1st and 3rd trimesters 2. Pelvic pressure from growing uterus 3. Increased GFR (decreased serum values), only partly compensated for by increased tubular reabsorption 4. Ureters and pelvis dilate (R>L) due to hormonal influences and pressure of uterus on kidneys (> risk for pyelonephritis related to delayed emptying time) 5. UTI risk

etiology of pediatric UTIs

1. MC, E. coli - 90% of 1st episodes and 75% of recurrences d/t bowel (also MC in adults) 2. Klebsiella, Proteus, Enterococcus- UT anomaly 3. Staphylococcus saprophyticus -#1 in ADOLESCENT FEMALES (sexually active)

Result of the absence of MLH1 and PMS2 possible interpretations

1. MLH1 germline mutation or 2. hypermethylation of MLH1 promoter, sporadic tumor. 3. PMS2 germline mutation-unlikely. Explanations/comments: MLH1 and PMS2 form a heterodimer, PMS2 staining often absent as result of MLH1 mutation.

Results of MLH1 on IHC possible interpretations

1. MLH1 germline mutation or 2. hypermethylation of MLH1 promoter, sporadic tumor. Explanations/comments: epigenetic silencing of MLH1 gene.

Result of the absence of PMS2 possible interpretations

1. MLH1 mutation. 2. PMS2 mutation. explanations / comments: MLH1mutation identified with loss of PMS2 only; MLH1 has dimer partners other than PMS2, germline mutations in PMS2 would not necessarily cause loss of MLH1 staining.

Result of the absence of MSH6 only possible interpretations

1. MSH6 germline mutation. 2. MSH2 germline mutations. Explanations / comments: MSH2 has dimer partners other than MSH6 germline mutations in MSH6 tend to cause loss of staining for MSH6 only.

pediatric UTIs Prognosis

1. Most cases are simple, uncomplicated UTIs >> readily tx OP abx w/ out sequelae - Fever resolves in 24 hours -asx in 48 hours 2. Appropriate treatment, follow-up, and imaging prevent long-term complications in more severe or chronic infections 3. Non-severe VUR typically resolves w/out permanent damage

Result of the absence of MSH2 possible interpretations

1. strong likelihood of MSH2 germline mutations. 2. EPCAM mutations. Explanations / comments: EPCAM mutation inhibits MSH2 leads to promoter hypermethylation.

clinical pearls associated with cultures and contamination

1. Multiple organisms suggest contamination or urinary tract abnormalities (very common in kids) -Order another cx d/t error in collection, unless already on abx 2. Contamination w/ perineal flora may mask an existing UTI 3. MCC of bacteriuria w/o UTI - CONTAMINATION of Lactobacillus and epithelial cells 4. Use judgment when interpreting clean catch midstream samples with any growth -Low SG with 60,000 - 80,000 CFU/mL may be significant even though it isn't 100,000 -Special Consideration: Well hydrated kids, Frequent voiding, Recent abx use >>dilute urine

pathophysiology of pediatric UTIs.

1. Neonates = hematogenous spread to bladder 2. Older pediatric pts: Ascending infection from perineal contaminants to bladder -Pyelonephritis and bacteremia can be d/t further ascending infection 3. Prolonged incubation in bladder>> infrequent/ incomplete voiding 4. Vesicoutereral reflux: congenital abn of ureter or abn dt UTI 5. UT anomalies = obstruction 6. Neurologic abnormalities 7. Recent broad spectrum abx (Amoxil - PCN, Keflex) 8. Constipation >> shifts angle of bladder

Uncomplicated UTI

1. No medical problems 2. No anatomic problems 3. No or physiologic problems

Highest Percentage of Abortions by Race

1. Non-Hispanic white 2. (non-hispanic black) 3. (hispanic)

Renal Ultrasound

1. Non-invasive and role is limited -Depicts kidney size and shape 2. Shows duplication and dilation of the ureters 3. Shows gross anatomic abnormalities 4. Unreliable in detecting renal scarring or VUR -No ability to show parenchymal function

Treatments for mastalgia?

1. Nonpharmacologic, complementary, alternative therapies. 2.Supportive bra 3.modify dose or route of HT 4. different contraceptive method or delivery system 5. topical use of NSAID;.

protocol for Follow-up urine culture

1. Not needed if cultured organism susceptible to prescribed abx -With good response 2. Repeat culture after 48 hours if organism is not susceptible 3. Perform culture after 48 hours if no culture initially performed

What happens to the GI system during pregnancy and why?

1. PROGESTERONE slows gut motility 2. Nausea and vomiting: first trimester, or longer... 3. Decreased tone and mobility ~ constipation (Progesterone is Cause) 4. Increased secretion of hCG & altered carbohydrate metabolism 5. Gallbladder: distended, thickening of bile, slight hypercholesterolemia from increased progesterone; gallstones not uncommon *These all happen bc there is a greater demand for nutrients

PE for a patient with a suspected UTI.

1. Palpable bladder, Suprapubic, Abdominal, CVA tenderness (CVA think pylonephritis) 2. HTN: Renal parenchyma disease - polycystic kidney -Hydronephrosis: distention and dilation of the renal pelvis and calyces - water on kidneys d/t a build-up of urine 3. Watch the patient void and look for: -Dribbling, poor stream, straining to void 4. Examine external genitalia: Irritation, Pinworms, vaginitis, trauma, sexual abuse

Result of all proteins exhibit normal expression possible interpretations

1. Possible missense mutation in any gene. 2. No LS. Explanations / comments: normal IHC results do not exclude LS; missense mutation could result intact, but non-function MMR protein. Refer to MSI results and/or FH.

epidemiology of pediatric UTIs by age and gender.

1. Prevalence varies by age: source of infection in 6-8% of febrile infants < 3mo 2. MC in FEMALES -EXCEPT in 1st 6 months: M>F -Uncircumcised > circumcised 3. Highest in sexually active adolescent females 4. <11 yo: F=3-5% risk / M=1% risk ***Peds PEARL: FUO in infant think Meningitis and UTI = DDx (hematogenous spread to bladder)

3 Most Common Imaging Techniques:

1. Renal Ultrasound: 2. Voiding cystourethrogram (VCUG): 3. Nuclear cortical scanning Gold standard: Renal US and VCUG for all children 2mon-2yr

Voiding cystourethrogram (VCUG)

1. Shows urethral and bladder anatomy -catheter w/ dye>>images voiding 2. Detects degree of Vesicoureteral reflux (grade I-V) 3. Can be scheduled as soon as child is asx

why all pediatric urine specimens should be sent to the lab

1. UA is not dx. It is used only as a screening tool that may suggest UTI. 2. Only a urine culture is diagnostic 3. 20% of negative UAs are really (+)

medical-legal issues associated with pediatric UTIs

1. UTIs and pyelonephritis must be considered in younger pediatric patients with fever and/or non-specific symptoms 2. Obtain UA and culture in all febrile males <6 mon. and in all febrile females <24 mon. unless another apparent source of infection 3. Pregnancy must be considered in adolescent girls who present with UTI sx, before tx w/ abx

Treatment in the Newborn:

1. Uncomplicated UTI: Gentamicin and Ampicillin for 10-14 days IV 2. Older infants can be switched to oral tx later in course (afebrile 24-48hrs) check for resistance to both drugs>> is an issue

criteria for diagnosing a UTI in the pediatric population.

1. Urine culture- GOLD STANDARD 2. Urinalysis (UA)- 3. Microscopy

genitourinary tract abnormalities detected by imaging

1. Vesicoureteral reflux 2. Obstructive uropathy

What happens to the blood volume and its composition during pregnancy?

1. Volume expands by 40-50% above non-pregnant (approx 1500ml: 1000ml plasma and 450ml RBC) 2. RBC mass increases by 20-30% 3. State of hemodilution 4. (Anemic if Hemoglobin less than 11g/dl and Hematocrit 32% or less) 5. Clotting factors change (eg. Factor VII and VIII increase)

Microscopy

1. WBC > 5/hpf and/or bacteria are strongly suggestive of UTI 2. Bacteria in urine correlates highly to cultures from suprapubic aspiration 3. RBCs, WBC casts, and epithelial cells (didn't clean properly) - any irritation to trigone causes shedding of RBCs

What happens to metabolism during pregnancy?

1. Weight gain 2. Water retained 3. Nutrients metabolism meets increased demand for protein and fat

Who is a combined oral contraceptive (estrogen/progestin) candidate? MEC Category, Yes or No 1. 22-year-old who smokes 1 PPD 2. 29-year-old with PID 3. 45-year-old with recurrent tension type headache - Non-migrainous headache - Age>40 years 4. A 36-year-old woman with well controlled primary hypertension who is taking an ARB and HCTZ - HTN with adequate control - HTN with poor control

1. YES 2. YES 3. YES 4. NO

Result of the absence of MSH2 and MSH6 possible interpretations

1. strong likelihood of MSH2 germline mutations. 2. MSH6 mutaion. 3. EPCAM mutations. Explanations / comments: MSH2 and MSH6 form a heterodimer, MSH6 staining is often absent as a result of MSH2 mutation.

the treatment plans for pediatric UTIs

1. treat early and aggressively to avoid renal damage. 2. All ill-appearing patients with febrile UTIs should be treated with parenteral abx. -Abx should cover E. coli, Enterococcus, Proteus, and Klebsiella -Suspected pyelonephritis should be tx with a combination of parenteral antibiotics

why early recognition and prompt therapy for pediatric UTIs are important?

1. very common in peds (MC GU dx in childhood) 2. prevent complications -Early: pyelonephritis (bladder infection into the kidneys), urosepsis (systemic infect. that starts in the kidneys) -Late: renal scarring lost of nephrons (≥ 72hr ) >>renal failure

Statistics regarding bacterial vaginitis (BV)

10-26% of visits to the OB/GYN 24-37% of visits to STD clinics 50% of women are asymptomatic 15% of sexually active women Less common in non-sexually active women More common in African/American population

Prenatal weeks

16-20 wks: triple marker/ screen US 24-28 wks: 1hr glucose, if Rh negative, Type and screen 28-32 wks: Hg, STI testing, RhoGAM as indicated 32-36 wks: Fetal presentation, kick count (>4 in 1 hour, >10 in 2 hour) 35-37 wks: Group B streptococcus culture (Rectal and vaginal) 40-42 wks: vaginal examination to assess cervical ripeness, fetal station 41+ wks: nonstress test, biophysical profile

leiomyomas are found: a. within the uterine wall b. on the vaginal wall c. within the cervix d. on the fallopian tube

a. within the uterine wall

Light therapy

A link exists between PMS & seasonal affective disorder that typically occurs in the fall & winter months when levels of natural light declines . Exposure to full spectrum light including UV light decrease moodiness & depressive symptoms of PMS

Severe Preeclampsia

A primigravida who is in the late third trimester of pregnancy (more than 34 weeks) complains of a sudden onset of severe recurrent headaches, visual abnormalities (blurred vision, scotomas), and pitting edema. Edema easily seen on the face/eyes and fingers. Sudden rapid weight gain within 1 to 2 days (greater than 2 lbs/week). New onset of right upper quadrant abdominal pain. Blood pressure more than 140/90 mmHg. Urine protein 1+ or higher. Sudden decrease in urine output (oliguria). Nausea and vomiting is a worrisome sign (encephalopathy). If seizures occur, it is reclassified as eclampsia. Earliest time period that preeclampsia/eclampsia can occur is at 20 weeks of gestation (and up to 4 weeks postpartum). Only known "cure" is delivery of fetus/baby.

ACOG reproductive coercion statement

ACOG issued statement re reproductive coercion based on a national domestic violence hotline report that ¼ callers reported reproductive coercion

What is the drug of choice for complicated UTIs?

AMP + Gentamycin or Zosyn (Pipperacillin + Tazobactam) or Timentin (Ticarcillin + Clavulanic Acid) or Doripenem or Imipenem or Meropenem x 2-3 weeks.

Dosing for Estrace:

estradiol (micronized) 0.5, 1, 1.5, 2 mg

to hormonal testing measures with secondary amenorrhea

estradiol level and progesterone challenge

analgesics

Acetaminophen is a nonopioid, systemic analgesic used for moderate voiding discomfort caused by urinary tract infection. Ibuprofen is a nonsteroidal anti-inflammatory agent that is used to provide symptomatic relief of dysuria.

MOA of Oral contraceptives

Estrogen and progestin feedback to the HT and ant. pituitary to decrease LH/FSH therefore no ovulation occurs Progestins also increase thickness of cervical mucus=acts as a barrier Progestins also decrease endometrial proliferation= endometrium is less suitable for implantation

If findings from the pelvic exam are normal, AUB can be a sign of

Systemic disease . Causes AUB by altering the HPOA or the uterine endometrium, which is the target of HPOA . Abnormal bleeding patterns with systemic etiologies typically include: amenorrhea, oligomenorrhea, & menorrhagia

An all- encompassing diagnosis referring to any uterine bleeding that is irregular in amount, frequency, duration, or timing

Abnormal uterine bleeding . Can be a normal physiologic event such as irregular bleeding that accompanies menarche or perimenopause . Can also signal pathologic life- threatening conditions such as ectopic pregnancy or endoemtrial cancer

Urinary Tract Infections

Acute cystitis can occur alone (simple urinary tract infections [UTI]) or it may be complicated by acute pyelonephritis. The most common organism is Escherichia coli (75% to 95%). The signs and symptoms includes dysuria, frequency, urgency, and nocturia. Higher risk preterm birth and low birth weight.

Teratogens

Agents that can cause structural abnormalities during pregnancy. ■ Alcohol: fetal alcohol syndrome ■ Aminoglycosides: deafness ■ Cigarettes: intrauterine growth retardation (IUGR), prematurity ■ Cocaine: CVAs, mental retardation, abruptio placentae ■ Isotretinoin (Accutane): central nervous system (CNS)/craniofacial/ear/cardiovascular defects ■ Lithium: cardiac defects (Ebstein's anomalies) Chronic hyperglycemia during pregnancy (poorly controlled diabetes or gestational diabetes mellitus [GDM]) is a teratogenic state. It increases the risk of neural tube defects and craniofacial defects.

Evidence from histologic & molecular studies suggest that anovulatory bleeding is a result of

An increased density of abnormal vessels that have a fragile structure prone to focal rupture followed by the release of lysosomes from surrounding epithelial & stromal cells & migratory leukocytes & macrophages

Does estrogen or progestin lead to endometrial proliferation

estrogen

Post-Surgical Abortion Contraception

Contraception: Immediate

What will wet prep of V.C. show?

budding yeast and hyphae

thelarche is the first sign of puberty in most girls. It usually occurs at about what age? a. 8 yrs b. 10 yrs c. 11 yrs d. 13 yrs

c. 11 yrs

Estrogen- Estrogen receptor mechanism

Estrogen diffuses into the cell and binds to an intracellular estrogen receptor. The Est-Est-Rec complex dimerizes and binds to an estrogen response element on the DNA which allows for upregulation/downregulation of transcription

What is the relationship between estrogen and plasma lipids?

Estrogen increases HDL and decreases LDL

The use of which of the following antibiotics is most likely to reduce combined oral contraceptive effectiveness? A. Amoxicillin B. Ciprofloxacin C. Doxycycline D. Rifampin

D

HRT and Osteoporosis

Estrogen replacement prevents, rather than reverses bone loss. Decreases # of osteoclasts. Reduces osteoporotic fracture, including hip fracture. Must be initiated early to achieve full effect.

Observational Studies and Estrogen

Estrogen users experience: - 50% reduction in CHD - 50% reduction in vertebral fractures - 25% reduction in hip fractures

Attributable to decrease in estrogen levels, cause often iatrogenic, such as cessation of estrogen therapy. Women with sustained high estrogen levels tend to experience heavy bleeding . PCOS, obese, perimenopausal

Estrogen withdrawal bleeding

Types of estrogens

Ethinyl estradiol, DES, mestranol

What herbal & OTC therapies are there for hot flashes?

Evening primrose oil: considered ineffect. Phytoestrogens: considered ineffect. - act as SERM & may incr risk breast cancer & decr efficacy of tamoxifen Black Cohosh: considered ineffect. - may have SERM effects - may cause liver dysfunction

cardiovascular effects of decreased estrogen levels

increased HDL, decreased LDL, vascular endothelium changes

Progestin only MOA, tx

Decreases endometrial proliferation, increases vascularization of endometrium (thickens cervical mucus) Tx: endometrial cancer, abnormal uterine bleeding, contraception

Preexisting/Chronic Hypertension

Defi ned as the presence of an elevated BP (>140/90) before the 20th week of gestation. Do not confuse this condition with preeclampsia. May be on a prescription. If on an ACEI/ARB, discontinue ASAP and monitor BP closely. Most pregnant women with preexisting hypertension can usually get off their BP meds (temporarily) during the fi rst to second trimester due to the lowering of blood pressure during pregnancy (less peripheral vascular resistance).

Size and Date Discrepancy

Defined as a difference of 2 cm (or more) in uterine size from the number of weeks of gestation. If present, order an ultrasound for further evaluation. For example, 30-week gestation with 28 cm uterus; the uterus is smaller than expected, the fetus is not growing normally. Fetus may have IUGR or other problem.

Mismatch repair failure leads to microsatellite instability

MSI: somatic expansion or contraction of microsatellite repetitive DNA elements.

Fundal Heights: 16 Weeks (Fourth Month)

■ Uterine fundus between symphysis pubis and the umbilicus. *Quickening also starts.

Estrogen and Progestin Therapy

EPT Most common Progestin added because estrogen alone is linked with endometrial cancer. Indicated for women with intact uteri

What are the two estrogen receptors

ER-alpha and ER-beta

Solo Estrogen Therapy

ET Indicated for women with hysterectomies

Women Health Initiative Findings

ET; no increased risk of heart disease, no increased risk of breast cancer.

Symptom monitoring essentially consists of

Educating women in self- diagnosis . Listing & rating feelings, symptoms & rating feelings

Category D Drugs

Evidence of fetal risk. Benefi ts should outweigh the risk of using the drug. ■ Angiotensin-converting enzyme inhibitors (ACEI) and angiotensin receptor blockers (ARBs) - Causes fetal renal abnormalities, renal failure, and hypotension. - Captopril (Capoten) and losartan (Hyzaar). - Category C in fi rst trimester. - Category D in second and third trimester. ■ Fluoroquinolones - Affects fetal cartilage development. A rare side effect is Achilles tendon rupture in athletes. Contraindicated in pregnant or lactating women and children younger than 18 years. - Ciprofloxacin (Cipro) - Ofloxacin (Floxin) - Levofloxacin (Levaquin) ■ Tetracyclines - Stains growing tooth enamel. - Tetracycline, minocin (Minocycline). - Avoid in the third trimester ■ NSAIDs - Block prostaglandins and may cause premature labor. Avoid using especially in the last 2 weeks of pregnancy. ■ Sulfa drugs - Risk of hyperbilirubinemia (neonatal jaundice or kernicterus). Sulfa drugs displace bilirubin from albumin. High levels of unconjugated bilirubin will cross the blood-brain barrier and cause brain damage (mental retardation, seizures, deafness, etc.)

Copper-containing IUD (Cu-IUD) Mechanism

Foreign body effect induced by presence of devise in intrauterine cavity results in a sterile inflammatory response that is toxic via a number of mechanisms to sperm and ova, impairs implantation Local uterine changes enhanced by presence of copper.

Pharmacologic treatment options for osteoporosis in postmenopausal women

Fosamax, Actonel, Boniva, calcitonin

What Is a Manual Vacuum Aspirator?

Has locking valve •Is portable and reusable •Vacuum is equivalent to electric pump •Efficacy is same as electric vacuum (98%-99%) •Has semi-flexible plastic cannula

MOA of "-azoles"

Inhibits synthesis of *ergosterol* from *lanosterol* via blocking *14-α demethylase* leading to poor cell wall structure

What can be done to relieve burning symptoms in UTIs?

May add the urinary analgesic phenazopyridine for 2 days (only not to mask the antibiotic effects).

What are the sides effects of phenazopyridine?

May cause hemolytic anemia in patient s with G6PD deficiency. It may also cause red-orange coloring of the urine and other body fluids (contact lens and clothes can be stained).

Most frequently observed pattern with ovulatory AUB

Menorrhagia . Also commonly associated with pelvic pathology such as uterine fibroids, adenomyosis, endometrial polyps . Also frequently associated with bleeding dyscrasias- 20% of adolescents

Irregular, Uterine bleeding that is excessive in flow or duration or both

Metrorrhagia

Types of testing for LS

Microsatellite instability analysis, i.e. MSI. Immunohistochemistry, i.e. IHC. MLH1 promoter hypermethylation / BRAF assay is used to prove tumor is sporadic

Classic dysmenorrhea pattern includes

Multiple pain symptoms that are rated as moderate or severe for 1 day premenstrually & 3 to 5 menstrual days . Completely subside after menses but might include cyclic headaches, bloating

Limitations of PMS2 testing

Multiple pseudo-genes which are pieces of PMS2 genes on other chromosomes with homologous sequences. Possibility mutations-large deletions- missed

What are some other common bacteria?

Mycoplasma Hominis Ureaplasma Urealyticum Mobiluncus sp. Bacteroides Ureolyticus

risk reduction after circumcision

Neonatal circumcision decreases the risk of UTI by about 90% in male infants during the first year of life. The risk of UTI in a circumcised infant is about 1 in 1000 during the first year, whereas an uncircumcised male infant has a 1 in 100 risk of developing a UTI. Overall, the rate of UTIs in circumcised boys has been estimated at 0.2-0.4%, with the rate in uncircumcised boys being 5-20 times higher than in circumcised boys.

Complete Menopause

No period for 1 year. Typically occurs between 45 and 55. Diagnosed via high FSH levels.

Secondary amenorrhea

No period for 6 months in a woman who have menstruated previously (at least 3 previous cycle)

Red Flags for Nipple Discharge

Non-milky discharge spontaneous unilateral From single duct Clear or bloody in color, likely associated with cancer

ultrasonography 1

Nonetheless, the American Academy of Pediatrics Clinical Practice Guidelines recommend routine ultrasonography after a first febrile UTI in children aged 2-24 months.

Abruptio Placentae

Pregnant woman who is in the late third trimester of pregnancy complains of sudden onset of vaginal bleeding accompanied by a contracted uterus that feels hard (hypertonic) and is very painful. Associated with a sudden onset of dark red-colored vaginal bleeding. Up to 20% of women do not have vaginal bleeding (blood is trapped between placenta and uterine wall). If mild, blood is reabsorbed and affected area reimplants. Severe cases cause hemorrhage; fetus must be delivered to save mother's life. Higher risk in females with history of hypertension, preeclampsia/eclampsia, cocaine use, or history of abruptio placentae.

Asymptomatic Bacteriuria

Pregnant women with asymptomatic bacteria are always treated because they are at high risk for acute pyelonephritis (20%). Diagnosis is based on a mid-stream urine C&S results. UTIs increase the risk of preterm birth, low birth weight, and perinatal mortality. Treatment Plan Short-course antibiotic therapy. Increase fluids. Pre- and posttreatment urine C&S. ■ Urine dipstick: WBCs (leukocyte esterase), positive/negative nitrite, sometimes RBCs. ■ Send mid-stream urine for urinalysis and urine C&S. ■ Document resolution of infection by ordering posttreatment urine C&S 1 week after completing antibiotic therapy. ■ If suspect pyelonephritis, refer to ED or a physician.

Placenta Abruptio (Placental Abruption)

Premature partial to complete separation of a normally implanted placenta from the uterine bed. Rupture of the maternal blood vessels from the decidua basalis. Bleeding ranges from mild to hemorrhage. Controllable risk factors are smoking, cocaine use, hypertension, and seatbelt use. Classic Sudden onset of vaginal bleeding (mild to hemorrhage) with abdominal and/or back pain. Painful uterine contractions. Uterus is rigid (hypertonic) and very tender. Treatment Plan ■ Refer to the ED. ■ Initial ED labs are CBC, PT/PTT, blood type, crossmatch, Rh factor, and so on. ■ Abdominal ultrasound. Blood transfusion. ■ If mild contractions, give magnesium sulfate (MgSO4) IV. Strict bedrest. ■ Deliver fetus by C-section if mother's life is threatened. Give steroids if fetus is viable.

A subset of women with severe PMS with an emphasis on mood symptoms

Premenstrual dysphoric disorder (PMDD) . Previous term is late luteal phase dysphoric disorder . A more severe form, a research diagnosis . Treatment: Prozac

Category X Drugs

Proven fetal risks outweighs the benefi ts. ■ Accutane (isotretinoin, a vitamin A derivative) - Used for severe cystic and nodular acne recalcitrant to treatment. Highly teratogenic. ■ Methotrexate (anti-metabolite) - Some types of autoimmune diseases (psoriasis, rheumatoid arthritis) and certain cancers. ■ Proscar (anti-androgen) - Used for BPH and prostate cancer. ■ Misoprostol (prostaglandin analogue) - Used as one of the drugs in medical abortions (a component of the "abortion pill"). ■ Evista (selective estrogen receptor blocker) - Used for treating osteoporosis. ■ All hormonal drugs (natural or synthetic) are category X in pregnancy—all forms of estrogens, progesterone, testosterone, and mefiristone (RU-86). ■ Any drug that blocks hormone synthesis or binding. ■ Depo Lupron is used for infertility, hormone-dependent cancers, and endometriosis.

What is drug therapy for urge incontinence directed at?

Relaxing smooth muscle of the base of the bladder or blocking the involuntary bladder contractions

recommendations 3

Renal ultrasonography should also be considered for any child with a first febrile UTI in whom good follow-up cannot be ensured

cystitis 1

Symptomatic relief for dysuria consists of increasing fluid intake to enhance urine dilution and output, acetaminophen, and nonsteroidal anti-inflammatory drugs. If voiding symptoms are severe and persistent, add phenazopyridine hydrochloride (Pyridium). Do not administer phenazopyridine hydrochloride for longer than 48 hours, because of the risk of methemoglobinemia, hemolytic anemia, and other adverse reactions

T/F? Almost 90% of US counties lack an abortion provider.

TRUE

T/F? Almost 90% of abortions in the US occur in the 1st 12 wks of pregnancy.

TRUE

T/F? More than 50% percent of women who have abortions used a contraceptive method during the month they became pregnant

TRUE

T/F? The majority of minors seeking an abortion report at least one parent knew about it.

TRUE

2 labs to order with secondary amenorrhea to rule out thyroid problems and hyperprolactinemia

TSH and prolactin level

What is the recommendation when dispensing phenazopyridine?

Take with food or following food to minimize stomach upset.

imaging

The American Academy of Pediatrics recommends that all infants and young children (aged 2mo to 2y) with a first UTI undergo urinary tract ultrasonography and voiding cystourethrography (VCUG). These tests should be acquired promptly if patients fail to show expected clinical response within 2 days of treatment

Antibiotics for Pregnant Women

The antibiotics below are category B. ■ Penicillins - Amoxicillin (Amoxil), penicillin, dicloxacillin. ■ Cephalosporins - First generation: cephalexin (Keflex), cefadroxil (Duricef). - Second generation: cefuroxime axetil (Ceftin), cefaclor (Ceclor), cefprozil (Cefzil). - Third generation: ceftriaxone (Rocephin) injections, cefdinir (Omnicef), cefixime (Suprax). - Fourth generation: cefepime (Maxipime) injection/IV (used mainly in hospitals). ■ Macrolides - Erythromycins - Erythromycin ethylsuccinate (E-mycin). Erythromycin estolate (EES) can cause hepatoxity in pregnant women and its use in pregnancy is not recommended. - Azithromycin (Zithromax), clarithromycin (Biaxin) - Clarithromycin (Biaxin) is the only macrolide that is a category C. Avoid use during pregnancy. Consult with physician before use and discuss risk versus benefits. ■ Nitrofurantoin - Nitrofurantoin (Furadantin, MacroBid) - Do not use with glucose-6-phospate dehydrogenase defi ciency (G6PD anemia)since it will cause hemolysis (anemia, jaundice, dark urine).

Breast cancer staging 4

The cancer can be any size (any T) and may or may not have spread to nearby lymph nodes (any N). It has spread to distant organs or to lymph nodes far from the breast (M1). The most common sites of spread are the bones, liver, brain, or lungs.

Mrs. L. would like to be fitted for a diaphragm. She has been on numerous hormones int he past and does not like the side effects. It is important to remember that when properly fitting a pt for a diaphragm, it should: a. allow a fingertip between it and the pubic arch b. be small enough to allow for vaginal expansion c. lie snugly over the pubic arch and under the cervix d. provide firm tension against the vaginal walls

a. allow a fingertip between it and the pubic arch

when premenstrual syndrome sxs do not respond to dietary and nonmedical therapies, which of the following drugs might you try? a. antidepressants b. antihistamines c. corticosteroids d. anticholinergics

a. antidepressants

Gestational Diabetes Mellitus (GDM) Treatment

Treatment Plan ■ First-line treatment is lifestyle (follow a meal plan and scheduled physical activity). - Eat 3 meals per day plus 2 to 3 snacks. Limit carbohydrates. - Exercise 30 minutes per day at least 5 days a week (total of 2 hours per week). ■ Low impact exercises such as walking and swimming preferred. - Frequent home glucose monitoring (4 to 6 times per day) - Insulin injections if unable to control blood glucose by diet and exercise. - No oral antidiabetics drugs are approved for use in pregnancy by the FDA. Follow Up (GDM) Test for GDM 6 to 12 weeks postpartum and at least every 3 years afterward (future). *Metformin and glyburide are being used (out of indication) to treat GDM by some health care providers. The ADA and FDA do not endorse the use of any oral antidiabetic drugs for pregnant women. Note: Gestational diabetes is now also known as overt diabetes.

Ceftriaxone + Z pack or Doxy, or cefixime + Z pack or Doxy

Treatment for uncomplicated gonococcal infection.

Butoconazole or fluconazole or Tioconazole

Treatment for vulvovaginal candidiasis.

the most common reason for surgery for macromastia is: a. back and shoulder pain b. breast cancer c. fibrocystic breast disease d. inability to breastfeed

a. back and shoulder pain

Metronidazole or Tinidazole

Trichomoniasis treatment

What is one drug that can be used in urge incontinence?

Tricyclic agents that possess moderate anticholinergic properties

T/F: "Available scientific and pharmacokinetic data do not support the hypothesis that antibiotics (with the exception of rifampin) lower the contraceptive efficacy of oral contraceptives

True

T/F: After single coital act, women who do not use contraception have 7.2% chance of becoming pregnant

True

T/F: For contraceptive methods other than IUDs, the benefits of starting to use a contraceptive method likely exceed any risk, even in situations in which the healthcare provider is uncertain whether the woman is pregnant."

True

T/F: Issues of lower clinical efficacy noted with use of hormonal EC in obesity, in particular LNG-containing options. Not noted with EC use of a CU-IUD in obesity

True

T/F: Long-acting Reversible Contraception (LARC) "Should be considered as first-line choices for both nulliparous and parous women including teens"

True

T/F: Ulipristal acetate (UPA) has been shown to be more effective than the LNG formulation between 3 and 5 days after unprotected intercourse.

True

True or False, Ability to self diagnose vaginal infection is frequently wrong

True

True/False: Ulipristal approved for use up to 5 d (120 h) post unprotected coitus LNG approved up to 3 d (72 h) post unprotected coitus

True

A dominant symptom cluster in terms of explaining variance in premenstrual symptoms

Turmoil

Naegele's Rule

Used to estimate date of delivery (EDD) during the fi rst trimester. Assumes regular 28- to 30-day menstrual cycle. Not as useful for irregular menstrual cycles. Procedure: Subtract 3 months from the month of the last menstrual period (LMP). Then add 7 days to the day of LMP.

Doxycycline

Used when Z pack is contraindicated when treating Chlamydia. Don't use in pregnancy.

safe and effective option for women who desire to retain fertility

Uterine artery embolization . All candidates for these procedures need to be evaluated by an OB-GYN prior to the procedure and provide informed consent indicating an understanding about the rare but possibly deadly complications of these procedure, i.e. Pulmonary embolism

which of the following drugs may have their effects enhanced when used in combination with an oral contraceptive? a. beta blockers b. oral anticoagulants c. antacids d. anti-convulsants

a. beta blockers

Complete Abortion

Vaginal bleeding with cramping. Placenta and fetus are expelled completely. Cervical os will close and bleeding stops.

Incomplete Abortion (Abortion With Retained Products of Conception)

Vaginal bleeding with cramping. Placental products remain in the uterus. Cervical os remains dilated and bleeding persists; pieces of tissue may be seen at the cervical os. Foul-smelling vaginal discharge (bacterial vaginosis). Treatment: Dilation with curettage (D and C) and antibiotics.

2 forms of estrogen therapy other than oral?

Vaginal estrogen cream, transdermal estrogen patch

Treatment of Urogenital Atrophy in Menopause

Vaginal preparations preferred if this is the sole complaint.

Vaginal HRT Estrogen Regimen

Vaginal rings, creams, pills. Advantages; local effects Disadvantages; local effects.

J. age 42 presents for her well-woman exam and you notice "dimpling" on her left breast. Your initial reaction is that is may possible be: a. breast cancer b. fibrocystic breast disease c. Paget's disease d. striae from recent dieting

a. breast cancer

Paget's disease

a rare type of cancer involving the skin of the nipple and the areola area

Paradox

a statement that seems contradictory but actually is not "I, a child, very old." Walt Whitman

Oxymoron

a type of paradox made up of two seemingly contradictory words: "Out of the murderous innocence of the sea." WB Yeats. Jumbo shrimp

which glands are posterior on each side of the vaginal orifice and open onto the sides of the vestibule int he groove between the labia minora and hymen? a. Bartholin's glands b. Skene's glands c. paraurethral glands d. cystocele

a. Bartholin's glands

what is the most common virus to be transmitted in utero? a. cytomegalovirus b. rubella c. varicella d. toxoplasmosis

a. cytomegalovirus

when a woman complains of dyspareunia in the lower back during orgasm, you should consider: a. endometriosis b. cystitis c. vaginitis d. causes r/t pelvic inflammatory disease

a. endometriosis

Susan says that her previous doctor never discussed why he took her off HRT. You share with her some of the results of the women's Health Initiative (WHI). which statement is true regarding the study? a. estrogen plus progestin increased the risk of a cardiac event in apparently healthy women b. persons on HRT are at a higher risk of colorectal cancer c. postmenopausal hormones do not actually prevent fxs of the hip d. estrogen alone is associated with a greater risk of breast cancer than a combination of estrogen plus progestin

a. estrogen plus progestin increased the risk of a cardiac event in apparently healthy women

of the sxs listed below, the most commonly expressed sx of women with premenstrual syndrome is: a. fatigue b. depression c. breast tenderness d. swelling of the extremities

a. fatigue

if a woman is using the basal body temperature (BBT) method of birth control and does not want to become pregnant, when would you tell her to avoid unprotected intercourse? a. from the beginning of the menstrual cycle until the BBT has been elevated for 3 days b. whenever the BBT is elevated c. whenever the BBT is lowered d. fromt he end of the menstrual cycle until the BBT has been low for 5 days

a. from the beginning of the menstrual cycle until the BBT has been elevated for 3 days

Sally has a dx of a chlamydia vaginal infection. You believe that is is questionable whether she will fill the prescription that you write or take it for 7 days as order. What would you do? a. give azithromycin (zithromax) 1g PO now b. emphasize the importance of the drug and tell her the consequences of not taking it. c. send out the public health nurse to follow up on whether she takes the drug for 7 days d. assume that Sally is an adult and will follow your instructions

a. give azithromycin (zithromax) 1g PO now

the best method to dx uterine polyps is a: a. hysteroscopy b. dilation and curettage c. colposcopy d. laparoscopy

a. hysteroscopy

a 27 y/o female presents to your office for a Mirena (levonorgestrel intrauterine system) insertion. She reports that her menses started 3 days ago and is normal. How soon after insertion will she be able to safely rely on it for contraception? a. immediately b. after 48 hrs c. in one wk d. in one month

a. immediately

Sally, age, 21 is taking an oral contraceptive. She c/o acne. How should you adjust the estrogen in the oral contraceptive? a. increase the estrogen content b. decrease the estrogen content c. delete the estrogen content d. no adjustment should be made to the estrogen content

a. increase the estrogen content

Mrs. G., called the office to make an appt for her annual Pap smear. She needs to be given the following instructions prior to her appt: a. insert nothing in the vagina for 24 hrs before the exam b. douching enhances visualization of the cervix and should be done before the appt c. an infection or menstrual period is no reason to cancel the appt d. the procedure is completely painless

a. insert nothing in the vagina for 24 hrs before the exam

a tx used to improve the chance of pregnancy in an infertile woman who has minimal or mild endometriosis is: a. laparoscopic resection or ablation of the lesions b. dilation and curettage c. the use of gonadotropin-releasing hormone analogues d. the use of birth control pills for 3 months and then abruptly stopping

a. laparoscopic resection or ablation of the lesions

L., age 38 states that she has not had a pelvic exam in 5 yrs because she does not like having the digital rectal exam. How do you respond? a. let's schedule an exam now because women no longer need a rectal exam with routine pelvic exams b. ok, we'll do a pelvic, and I'll just put refused on the chart to cover my liability c. we really need to don one because the rectal exam has been shown to pick up many abnormalities such as rectal polyps d. I'll try to be quick with the rectal exam and get it over with

a. let's schedule an exam now because women no longer need a rectal exam with routine pelvic exams

a sexually active woman should be aware that genital herpes simplex virus: a. may be transmitted to a partner or newborn even in the absence of lesions because of viral shedding b. is suppressed during menstruation, physical or emotional stress, immunosuppression, sexual intercourse, and pregnancy c. recurrences usually last the same length of time as the initial outbreak d. requires the use of condoms only during outbreaks

a. may be transmitted to a partner or newborn even in the absence of lesions because of viral shedding

M., age 42, is having dysfunctional uterine bleeding (DUB) and cannot take oral contraceptives due to a hx of a blood clot (emboli). Management includes which medication? a. medroxyprogestone b. ethinyl estradiol c. conjugated estrogen d. piroxicam

a. medroxyprogestone

which of the following statements do you use when instructing women about their fertile period (when they are most likely to become pregnant)? a. ovulation occurs on the 14th day, plus or minus 2 days, before the next menses b. sperm are viable for 24 hrs c. the ovum is viable for 6 hrs d. the ovaries always release one ovum per month

a. ovulation occurs on the 14th day, plus or minus 2 days, before the next menses

which of the following is an indication for a colposcopy? a. pap smear showing dysplasia b. recurrent STIs c. HIV infection d. hx of leiomyomas

a. pap smear showing dysplasia

which type of incontinence has an associated sx of recurrent cystitis? a. stress incontinence b. urge incontinence c. overflow incontinence d. function incontinence

a. stress incontinence

toxic shock syndrome (TSS) may be causes by which of the following? a. tampon contamination with staphylococcus aureus b. a short vaginal canal c. the use of superabsorbent tampons d. a UTI involving the bladder and kidneys

a. tampon contamination with staphylococcus aureus

C., says that her healthcare provider wants to do a colposcopy. She ask you what this is. You tell her that a colposcopy: a. visualizes the cervical, vaginal, or vulvar epithelium under magnification to identify abnormal areas that may require a biopsy b. involves removal of one or more areas of the endometrium by means of a curette or small aspiration device without cervical dilation c. allows visual exam of the uterine cavity with a small fiber-optic endoscope passed through the cervix d. allows visualization of the abdominal and pelvic cavity through a small fiber-optic endoscope passed through a subumbilical incision

a. visualizes the cervical, vaginal, or vulvar epithelium under magnification to identify abnormal areas that may require a biopsy

B., age 24 c/o urgency, frequency, and dysuria. Your dipstick test shows no hematuria, and her urine culture shows no growth. What is your next action? a. you suspect a sexually transmitted infection, so you obtain a culture of the urethra, do a potassium hydroxide wet prep, and obtain another urine culture b. you suspect urethra irritation, so you tell her to take showers, not bubble baths, and wear white, dry underwear and loose-fitting clothing c. you suspect a UTI not visible yet on culture, so you start her on Bactrim DS d. you suspect that the vulva is irritated. You tell her to take a relaxing shower and dry the area well and come back in 1 wk if there is no improvement

a. you suspect a sexually transmitted infection, so you obtain a culture of the urethra, do a potassium hydroxide wet prep, and obtain another urine culture

secondary amenorrhea definition

absence of 3 consecutive cycles after regular periods have been established (is not pregnant, breast-feeding, or menopausal)

primary amenorrhea

absence of menarche by age 16 without secondary sex characteristics or by age ____ with secondary sex characteristics

Primary amenorrhea

absence of menses by 14 with delay in maturation of secondary sexual characteristics or absence of menses by age 16 with evidence of secondary sexual characteristics

Menopause is defined as the point in time in which there has been a cessation of menstruation for

at least 12 consecutive months

Tx of VVC

azole, fluconazole + hydrocortizone cream Clotrimazol Miconazole Nystatin

Jenn, a 25 y/o female pt, c/o dysuria. In taking a thorough hx to formulate a dx, it is most important to ask: a. do you have painful intercourse? b. do you have an associated vaginal discharge or irritation? c. do you also have a problem with defecation? d. do you have stress incontinence?

b. do you have an associated vaginal discharge or irritation?

which form of estrogen is secreted in the greatest amount by the ovaries during the reproductive yrs and considered most potent? a. estrone (E1) b. estradiol (E2) c. estriol (E3) d. the potency and secretion of all of the above are in equal amts

b. estradiol (E2)

S., age 29, c/o premenstrual syndrome. She state she was told that changing her diet might help in managing some of the sxs. What changes in her diet do you recommend? a. increase her intake of protein b. increase her intake of complex carbohydrates c. increase her intake of sale and salty foods d. decrease her intake of fatty foods

b. increase her intake of complex carbohydrates

a dancer from an adult club down the street comes in for a renewal of ther birth control pill prescription. She says that everything is fine. On exam, you find grayish-white vaginal discharge, greenish cervical discharge, and cervical motion tenderness. Which of the following differential dx is most UNlikely? a. gonorrhea b. interstitial cystitis c. bacterial vaginosis d. chlamydia

b. interstitial cystitis

when a woman has extreme spasticity, which position should she assume for a Pap smear? a. OB stirrups position b. knee-chest position while prone c. v-shaped position without stirrups d. side-lying position

b. knee-chest position while prone

Lactobacilli in the vagina: a. decreases glycogen metabolism b. maintains an acid pH range c. increases the development of WBCs d. maintains an alkaline pH range

b. maintains an acid pH range

which of the following drugs may have their effects diminished when used in combination with an oral contraceptive? a. corticosteroids b. oral anticoagulants c. antibiotics d. anticonvulsants

b. oral anticoagulants

Mrs. H. inquires about why she needs progestereone in addition to her estrogen for hormone replacement. Women who have an intact uterus need to add progesterone to their prescribed estrogen because progestin: a. assists in relieving the typical hot flashes ofmenopause b. reduces the incidence of endometrial hyperplasia and cancer c. decreases the risk of osteoporosis d. controls mood swings

b. reduces the incidence of endometrial hyperplasia and cancer

Dolly, is to undergo a cone-needle biopsy for a suspicious breast mass. This procedure includes: a. 21 or 22 gauge needle that is used to aspirate cells from the lesion for analysis b. removal of a large core of tissue from the lesion for histological eval utilizing a large-gauge cutting needle c.removal of a wedge of tissue for exam d. removal of the entire lesion

b. removal of a large core of tissue from the lesion for histological eval utilizing a large-gauge cutting needle

a 21 y/o woman comes to your ofice and reports a hx of genital warts. In reference to the HPV vaccination (Gardasil or Cervarix), she should be educated that: a. she is not in the correct age grp and is not a candidate for the vaccination b. she should receive the HPV vaccination c. she already has been exposed to HPV; therefore, she is not a candidate for the vaccine d. there is a vaccine coming out shortly specifically for those who have been exposed. She should wait

b. she should receive the HPV vaccination

Mary, age 59, presents with depression. According to the diagnostic criteria for a major depressive d/o, which of the following criterion needs to be met? a. the depression must have a specific cause, like alcohol, drugs, medication s/e, or physical illness b. the sxs must be severe enough to upset the cl'ts daily routine or to impact his/ or her work or interfere with relationships c. the depression may be a normal reaction to the death of a loved one d. there must be some type of sleep disturbance

b. the sxs must be severe enough to upset the cl'ts daily routine or to impact his/ or her work or interfere with relationships

which of the following is a sexually transmitted infection? a. candida vaginitis b. trichomonal vaginitis c. atrophic vaginitis d. lactobacilli vaginitis

b. trichomonal vaginitis

Melody, age 50, desires hormone replacement therapy (HRT) for her hot flashes, which she can't stand. You have discussed the pros and cons and given her some alternative suggestions. Her mother had a hx of osteoporosis. You have decided to initiate therapy for one year. She asks you if she also needs to take calcium or vitamin D for prevention of osteoporosis. How do you respond? a. research has shown that HRT alone is sufficient to protect against osteoporosis b. yes, calcium intake should be increased to 1200mg/day along with 600mg of vitamin D to decrease bone turnover and increase intestinal absorption c. if you decide to take calcium and vitamin D, you can stop the HRT d. if you are getting sufficient exercise, you don't need to take calcium and vitamin D

b. yes, calcium intake should be increased to 1200mg/day along with 600mg of vitamin D to decrease bone turnover and increase intestinal absorption

Tamoxifen

blocks estrogen receptors on breast cancer cells. Estrogen is still present in normal levels, but the breast cancer cells cannot get enough of it to grow.

postmenopausal women at 65 must be tested for ____.

bone density; *osteopenia*; calcium and vitamin D

The gold standard for diagnosing osteopenia or osteoporosis is

bone mineral density measurement by Dual energy x-ray absorptiometry . A technique that is used to Evaluate central BMD at the spine and hip

Ann, age 19, is going to begin taking birth control pills. She asks you if she is safe immediately. How do you respond? a. yes, you should not get pregnant once you start the pill. However, it doesn't protect you from STIs b. for the first month, you need to be on a backup birth-control method. However, it doesn't protect you from STIs c. a second birth control method needs to be used during intercourse for the first 7 days while taking the pill. However, it doesn't protect you from STIs d. until you have your second period (cycle) with the pill, you are not considered safe

c. a second birth control method needs to be used during intercourse for the first 7 days while taking the pill. However, it doesn't protect you from STIs

S. has been dx with fibrocystic breast disease. Which of the following may exacerbate the condition? a. daily dose of aspirin b. spicy foods c. chocolate d. wearing tight bras

c. chocolate

an occurrence of genital herpes is: a. cured with acyclovir b. best managed with trichloroacetic acid 80%-90% applied directly to the lesion c. expected to be completely resolved within 21 days (for the primary lesion) d. not a factor in continuing with intercourse

c. expected to be completely resolved within 21 days (for the primary lesion)

Belle is breastfeeding her 3 month old infant with no supplementation./ She says she has heard that she cannot get pregnant during this time. What do you tell her? a. it is highly likely that you may become pregnant, so you should use another method of birth control b. yes, you are safe for as long as you breastfeed c. for the first 6 months, if you breastfeed and have very little supplementation, your chances are less than 2% that you'l get pregnant d. you are more at risk for getting pregnant now because of your fluctuating hormone levels

c. for the first 6 months, if you breastfeed and have very little supplementation, your chances are less than 2% that you'l get pregnant

women who take oral contraceptives are less likely to experience: a. human papillomavirus infection b. migraine headace c. iron -deficiency anemia d. herpes simplex virus

c. iron -deficiency anemia

a 17 y/o female presents to your office with the c/o lower abdominal pain since her period ended 2 days ago. She has a new sexual partner in the past 3 months and does not use condms. On PE, you find that she has cervical motion tenderness. You are concerned that she may have PID. To meet the CDC minimum criteria for empiric tx of PID, she must also have: a. an oral temp greater than 101F and mucopurulent cervicitis b. a positive test for cervical infection and an adnexal mass c. lower abdominal tenderness and adnexal tenderness d. mucopurulent cervicitis and an elevated WBC

c. lower abdominal tenderness and adnexal tendernes

a 26 y/o female comes to your office to discuss birth control options. Her hx includes migraine headaches with aura while on combination oral contraceptives in the past. She does not want to become pregnant. Which of the following birth control options would be the best choice for her? a. combined hormonal contraceptive pills b. ortho evra patch c. mirena IUD d. vaginal Nuvaring

c. mirena IUD

which of the following drugs given to nursing mothers may cause a reduction in the milk supply? a. antihistamines b. antithyroid medication c. oral contraceptives d. laxatives

c. oral contraceptives

small-quantity incontinence with nearly continous dribbling is symptomatic of which kind of incontinence? a. stress incontinence b. urge incontinence c. overflow incontinence d. functional incontinence

c. overflow incontinence

J., has a seizure disorder and wants to get pregnant. What is the drug of choice for her during pregnancy a. valproate (depakene) b. trimethadione (tridione) c. phenobarbital (luminal) d. phenytoin (dilantin)

c. phenobarbital (luminal)

which of the following signs and/or sxs of a genital herpes infection usually occurs first? a. painful or pruritic vesicles b. dysuria c. prodromal tingling or pruritus of the genital region d. white, curdlike plaques on a red base in the vagina

c. prodromal tingling or pruritus of the genital region

which of the following lifestyle factors is associated with an increased risk for breast cancer? a. being underweight b. having one to two drinks of alcohol per day c. smoking d. eating a low-fat diet

c. smoking

on PE, J., has pubic hair that spreads over her mons pubis with a slt lateral spread. In addition, her breast development shows breast enlargement with secondary mound formation by the developing areola. Which Tanner best describes J.'s development? a. stage II b. stage III c. stage IV d. stage V

c. stage IV

M., age 40 has been given a dx of rheumatoid arthritis. She asks you whether she should continue taking her birth control pills. You tell her: a. to check with her rheumatologist b. to stop c. to continue d. the dose will have to be altered

c. to continue

candidiasis is more common in: a. teenage girls b. women on low fat diets c. women with diabetes d. women with frequent UTIs

c. women with diabetes

Women are at a significantly increased risk for developing heart disease following menopause. Some of these increased risk is due to

changes in cholesterol levels that are found in postmenopausal women . The LDL and VLDL levels increase, and LDL oxidation is enhanced. HDL levels may decrease somewhat . Women who experience premature menopause may have even greater risk especially if they smoke

Bethesda Guidelines indicates Lynch-associated tumors include

colorectal, endometrial, stomach, ovarian, pancreas, biliary tract, small intestine, brain tumors/glioblastoma, sebaceous gland adenomas, keratochanthomas, hepatobiliary, transitional cell carcinoma of the renal pelvis or ureter.

primary hormone replacement therapy

combination estrogen/progesterone if patient has uterus

Dosing for Premarin:

conjugated estrogens 0.3, 0.45, 0.625, 0.9, 1.25 mg * start low (0.3/0.45mg) then titrate up if no relief from sx's

if pregnancy is UNDESIRED

council patient about her options: -referral for counseling -referral for adoption -referral for abortion be: -non-judgemental -non-directive -follow-up

Chronic anovulatory AUB managed with

cyclic progestogen therapy

how long can the vaginal contraceptive ring (Nuvaring) be out of the vagina before an additional form of contraception is necessary? a. 3o min b. one hr c. 2 hrs d. 3 hrs

d. 3 hrs

which of the following terms describes the bluish or purplish discoloration of the vulva, vagina, and a portion of the cervix that occurs in pregnancy? a. Goodell's sign b. Hegar's sign c. Piskacek's sign d. Chadwick's sign

d. Chadwick's sign

in a pt dx with cervical gonococcal infection, you would also suspect a co-infection with: a. candidiasis b. syphilis c. trichomoniasis d. chlamydia

d. chlamydia

S., states that she has heard that douching effectively washes out the sperm after intercourse and that she has been using this as a method of birth control. Which of the following statements about douching is true? a. douching prevents sperm from entering the uterus b. douching should be used at least once a month after menses if not used after intercourse c. douching is a reliable contraceptive d. douching may increase the risk of ectopic pregnancy

d. douching may increase the risk of ectopic pregnancy

herpes implex virus can be potentially acquired through maternal transmission. This is least likely to occur: a. before labor b. during delivery c. postnatally d. during the neonatal period

d. during the neonatal period

Reiter's syndrome is a complication of: a. bacterial vaginosis b. syphilis c. chlamydia d. gonorrhea

d. gonorrhea

which of the following conditions is a contraindication to using the copper IUD? a. hx of ectopic pregnancy b. nulliparity c. tx cervical dysplasia d. heart disease

d. heart disease

there are many causes of amenorrhea. In ballet dancers or marathon runners, which anatomical structure is the probable cause? a. outflow tract b. ovary c. anterior pituitary d. hypothalamus

d. hypothalamus

Jane, age 60 asks, you about taking alendronate (Fosamax). What do you tell her about using the medication? a. if you decide to take it, stick with a lower dose of 5mg because the side effects are much worse with a 10mg dose b. fosamax works better in younger women, so you should start this now rather than wait until you're 70 c. you should take a daily dose because the wkly dose is not as effective d. in addition to its efficacy in the tx of osteoporosis in postmenopausal women, it is also useful for the prevention of osteoporosis

d. in addition to its efficacy in the tx of osteoporosis in postmenopausal women, it is also useful for the prevention of osteoporosis

infertility is best defined as the: a. inability to conceive with multiple sex partners b. inability to conceive for 9 months of unprotected intercourse when both partners are younger than 30 yrs of age c. state of voluntary childlessness d. inability to conceive after 1 full yr of unprotected intercourse

d. inability to conceive after 1 full yr of unprotected intercourse

the Jameses are thinking about going for infertility counseling because they have been married for 5 yrs and have been unable to conceive. They ask you whether the man or the woman is usually the cause of the infertility. What do you tell them about the etiology of infertility? a. in most cases, infertility is related to a female factor b. in most cases, infertility is related to a male factor c. in the majority of cases, the etiology cannot be identified d. male and female infertility rates are almost the same in the majority of cases

d. male and female infertility rates are almost the same in the majority of cases

menses at irregular intervals with excessive flow and duration is defined as: a. oligomenorrhea b. polymenorrhea c. menorrhagia d. metrorrhagia

d. metrorrhagia

D., is a 23 y/o pt dx with dyspareunia. Which of the following is NOT a cause for this condition? a. vulvovaginitis b. an incompletely stretch hymen c. vaginismus d. multiple pregnancies

d. multiple pregnancies

endometrial cancer, hirsutism, acne, breast cancer, increased risk of diabetes,infertility, menstrual bleeding problems, and an increased risk of cardiovascular disease are clinical consequences of: a. mastalgia b. menorrhagia c. endometriosis d. persistent anovulation

d. persistent anovulation

the most likely cause of amenorrhea is: a. an anatomical deviation b. a genetic factor c. an endocrine abnormality d. pregnancy

d. pregnancy

characteristics of polycystic ovary syndrome include: a. hirsutism, thinness, hypoinsulinemia b. menopausal onset, vitilgo, hyperinsulemia c. alopecia, thinness, abdominal cramping d. premenarchial onset, obesity, hyperinsulinemia

d. premenarchial onset, obesity, hyperinsulinemia

Gean, a 33 y/o female pt, c/o external vaginal irritation after adding new fabric softener to her laundry. You have dx her with reactive vaginitis. The tx of choice for this condition includes: a. metronidazole (flagyl) 500mg BID x 7 days b. conjugated vaginal estrogen cream externally every day for 1 wk c. rewashing undergarments without fabric softener and applying petroleum jelly to the affected area d. rewashing undergarments without fabric softener and applying corticosteroids to the affected area

d. rewashing undergarments without fabric softener and applying corticosteroids to the affected area

which of the following ovarian tumors or cysts has the potential for malignancy? a. follicle cysts b. Brenner's tumor c. fibroma d. secondary ovarian tumors

d. secondary ovarian tumors

what is the most common cause of mastitis in breastfeeding women? a. escherichia coli b. streptococcus c. mycobacterium tuberculosis d. staphylococcus aureus

d. staphylococcus aureus

emergency contraception refers to: a. an induced abortion inan emergency room b. quickly starting on birth control pills in anticipation of sexual intercourse c. having a medroxyrogesterone injection in the ER q12 wks d. taking emergency contraceptive pills (ECPs)

d. taking emergency contraceptive pills (ECPs)

Procidentia is a: a. cystocele b. rectocele c. vaginal fistula d. third-degree uterine prolapse

d. third-degree uterine prolapse

J., age 26, who has genital herpes, ask if her partner has to use a condom during sexual intercourse even if she does not have a visible lesion. How do you respond? a. yes, we are not sure if its still transmitted when the lesions are not visible, so it is better to be on the safe side b. no, you are not contagious when the lesions are not visible c. no, use of spermicidal agent is all that is required d. yes, shedding of the herpes simplex virus from mucocutaneous surfaces int he absence of visible lesions is a primary mode of transmission

d. yes, shedding of the herpes simplex virus from mucocutaneous surfaces int he absence of visible lesions is a primary mode of transmission

menopausal change to the breast (2)

decrease in breast size and change in texture

women may present with these 2 UTI symptoms but they may be from menopause

dysuria and frequency

Hormonal causes of early, mid, and late spotting

early = estrogen not increasing fast enough Mid = estrogen drop off at ovulation Late = progesterone deficiency

Semen analysis: Most male factor infertility is detected with semen analysis. It is important that semen analysis be performed as diagnostic procedures.

early in the infertility evaluation so that male factor infertility can be diagnosed before the woman undergoes extensive, invasive diagnostic procedures . Abstinence of 2 to 3 days prior to sample collection . No more than 1 hour should elapse between collection & microscopic examination of the semen . Keep @ room or body temperature during transport . Usually includes at least 2 semen analysis

can't miss diagnosis with postmenopausal bleeding

endometrial carcinoma

if patient is given estrogen alone and she has a uterus she is at risk for (2)

endometrial hyperplasia and endometrial cancer

pelvic ultrasound findings to exclude endometrial cancer

endometrial stripe less than or equal to 4 mm in diameter

5 differential diagnosis with secondary dysmenorrhea

endometriosis, adenomyosis, fibroids, cervical stenosis, adhesions from PID

The cause of cervical inflammation (cervicitis) depends on the ...... affected.

epithelium

ultrasound may be useful in abnormal pre-menopausal bleeding to evaluate ___, and to diagnose these 2 things

evaluate endometrial thickness; diagnosis intrauterine or ectopic pregnancy or adnexal masses

how fast does bHCG double?

every 48-72 hours

Germline genetic testing

evidence of deficient mismatch repair at the level of the genome

most common cause of postmenopausal bleeding

exogenous hormones

• Endometrial alterations w/ ulipristal use could impact embryo ________.

implantation

An alternative and more accurate method of ovulation detection is available

in the form of over-the-counter urine test for LH . The presence of a surge in LH in the first morning urine indicates ovulation is likely occurring. LH testing may become cost prohibitive if used for an extended period, however

with changes in FSH and estrogen levels can be expected in menopause

increase in FSH and decrease in estrogen

acute cystitis

infection involved up to the bladder (and urethra) - Lower UTI

pyelonephritis

infection upto the kidneys (urethra, bladder, and ureters>> upper UTI =in peds if there is evidence of UTI and fever( or ill appearing)

What is vaginitis?

inflammation or infection of vagina

oligomenorrhea

infrequent, *irregular* bleeding, cycles greater than 35 days apart

The reliable indicator of heavy bleeding

is the passage of clots accompanied by heavy bleeding

Hallmark of VWD is

menorrhagia

In women with BV who are undergoing surgical abortion or hysterectomy, perioperative treatment with ...... eliminates this increased risk.

metronidazole

most effective treatment for BV is __________.75 cream for 5 days or 500mg tabs for 7 days. (ok in pregnancy, preferred systemic)

metronidazole

Risk factors of trichomonas

multiple sex partners, gonorrhea and chlamydia, previous STIs,, untreated sex partner

When is nipple discharge usually cancer?

nipple discharge accompanied by mass or abnormal mammogram; woman older than 50

figurative language/figures of speech

nonliteral language that explains or considers one thing by comparing it to something else quite different. Figurative language should evoke strong images.

It is imperative that all ....... be treated with a similar antibiotic regimen. (cervicitis)

sexual partners

2 things to remember when writing a prescription for hormone replacement therapy

short duration (6-12 months), smallest dose to control symptoms

physical exam findings of ovaries in postmenopausal women

should not be palpable

Studies performed on pathology specimens of the tumor

show evidence of mismatch repair at the level of the tumor

How do you diagnose vaginitis?

signs/symptoms, culture, and wet prep

symptoms of menopause (7)

sleep disturbances, hot flashes, vaginal dryness, atrophy of vaginal and vulvar epithelium; Headache, nervousness, and depression

As women women age, their resorption rate slowly begins to exceed that of bone formation, resulting in a

slow decline of bone mass. Because of the loss of estrogen, the rate of bone loss in the 1st year is especially rapid

Dosing for Premphase

take daily CEE 0.625 mg days 1-28 medroxyprogesterone 5mg days 15-28

most common *structural* lesion causing abnormal bleeding and peri-menopausal women

uterine fibroid tumors; still must consider endometrial hyperplasia or cancer

the most common reason for hysterectomy in the United States

uterine fibroids

decreased estrogen causes atrophy of (3)

vagina ( dyspareunia), urethra (dysuria), cervix

contraindications for hormone replacement therapy (5)

vaginal bleeding; thromboembolic events; breast, ovarian or, uterine cancer ( known estrogen dependent neoplasms); chronic liver failure; pregnancy.

Characteristics of clue cells

vaginal epithelial cells with many bacteria attached grainy appearance specific to BV

All of the following are relative contraindication in copper-containing IUD:

valvular heart dx, multiple sexual encounters, lack of follow-up care

Symptoms associated with menopause and perimenopause: The symptoms women report most frequently are

vasomotor in nature including hot flashes/ flushes and sweats

HRT may be used to treat moderate-to-severe ___.

vasomotor symptoms

3 physical exams to perform in secondary amenorrhea

visual fields, cranial nerves, breast exam and attempt to express milk from nipple

Is frequently the cause of lifelong abnormal patterns in women

von Willebrand disease (VWD) . Genetically inherited bleeding disorder

first-line treatment for uterine fibroids

watchful waiting-most fibroids will decrease in size after menopause

Symptoms of Atrophic Vaginitis

watery, dirty grayish discharge, vaginal narrowing, dyspareunia, buring/itching, dysuria

treatment for increased blood loss

weight loss, NSAIDS, OCPs-if estrogen is contraindicated use progestin only.

Synecdoche

when part of something is used to represent the whole. "They were called legs or grunts." The Things They Carried Memory hint: his neck is part of him

Metonomy

when something is represented by another thing closely associated with it. The White House or Oval Office for the President

Symptoms of Trichomonas

yellow/green discharge, strawberry cervix, fishy odor, dysuria, erythema, dyspareunia (painful sex), severe itching

Fundal Heights: From 20 to 35 Weeks of Gestation

■ Fundal height in cm = number of weeks gestation. For example, a 24-week gestation fetus should have fundal height between 23 and 25 cm.

Who is a Cu-IUD (ParaGard®) or LNG-IUS (Mirena®) candidate (category 1 or 2)? Yes or no? Perhaps? • 45-year-old nulliparous woman • 33-year-old who smokes 2 PPD • 25-year-old with seizure disorder • 33-year-old woman with HIV (excludes AIDS-defining illness)

• 45-year-old nulliparous woman: YES • 33-year-old who smokes 2 PPD: YES • 25-year-old with seizure disorder: YES • 33-year-old woman with HIV (excludes AIDS-defining illness)

diagnosis and managment of UTI in febrile infants and children aged 2-24 months

• After 7-14 days of antimicrobial treatment; close clinical follow-up monitoring should be maintained to permit prompt diagnosis and treatment of recurrent infections • Ultrasonography of the kidneys and bladder should be performed to detect anatomic abnormalities • Because data do not support the use of antimicrobial prophylaxis to prevent febrile, recurrent UTI in infants with no VUR or with grade I to IV VUR, a VCUG is not recommended routinely after the first UTI • VCUG is indicated if renal and bladder ultrasonography reveal hydronephrosis, scarring, or other findings that suggest either high-grade VUR or obstructive uropathy. • VCUG should be performed if there is a recurrence of a febrile UTI, even if previous ultrasonographic examination was unremarkable.

Factors Contributing to the Decline in the Number of Abortion Providers

• Anti-choice harassment and violence • Social stigma/marginalization • Professional isolation/peer pressure • The "graying of providers" • Inadequate economic/other incentives • Lack of clinical training opportunities

recommendations 2

• Children with a delayed or unsatisfactory response to treatment of the first febrile UTI • Children with an abdominal mass or abnormal voiding (dribbling of urine) • Children with a first febrile UTI caused by an organism other than E coli • Children with recurrence of a febrile UTI after they have had a satisfactory response to treatment of the initial febrile UTI

Levonorgestrel as Emergency Hormonal Contraception: Mechanism of Action

• Depending on time taken during menstrual cycle, interferes with fertilization - Inhibit or delay ovulation (most likely effect) • When given 2 d prior to luteinizing hormone (LH) surge, levonorgestrel delays or inhibits ovulation. • Once LH begins, levonorgestrel has no impact on ovulation. - Inhibits tubal transport of egg or sperm Unlikely mechanism of action -Emergency hormonal contraception, with levonorgestrel results in minimal to no alteration to endometrium, therefore unlikely to inhibit implantation of a fertilized egg.

do not need imaging

• Infants and children with a first febrile UTI who are assured follow-up, who respond promptly to treatment (afebrile within 72 h), and who have a normal voiding pattern (no dribbling) and no abdominal mass • Infants and children with cystitis

Assessment History & physical

•Detailed menstrual and contraception history. ›Age; age at menarche and menopause; cycle length, duration, estimated amount of flow; when menstrual pattern changed. ›Contraceptive: type, length of use, side effects. •Complete medication history. •Physical and pelvic examination. •Speculum and bimanual examination.

Etiology

•Empirical and theory testing research: integrative etiology links genetics, environmental stressors, hormonal processes. •Manifestation of PMS: combination of multiple stressors, heightened stress response, few supports, vulnerable period of biologic reactivity. •History of sexual abuse common among women seeking treatment for severe PMS.

Etiology

•Identifying cause of perimenstrual cyclic pelvic pain difficult & complex. •Etiologic mechanisms for cyclic perimenstrual physical and mood discomforts, PMS, PMDD even less clear.

Therapeutic Process/Management of Severe PMS

•Raise self-awareness; validate experience. •Set goals and outcome criteria. •Combination of treatments.

Pharmacologic Symptom Management

›Antidepressants ›Anxiolytic Drugs ›Hormonal Therapy ›Diuretics

Cyclic Perimenstrual Pain & Discomfort (CPPD) Symptom Clusters •Primary symptom is a cluster of symptoms both before and after cycle

›Cyclic pelvic pain & dysmenorrhea ›Perimenstrual physical discomforts ›Perimenstrual mood discomforts

Goals for Treating AUB

›Normalize bleeding. ›Correct any anemia. ›Prevent cancer. ›Restore quality of life.

Laboratory and Diagnostic Testing

›Pregnancy test. ›Complete blood count. ›Thyroid-stimulating hormone (TSH). ›Prolactin level. ›Pap test. ›Nucleic acid amplification test (NAAT). ›Microscopic examination of vaginal secretions. ›Coagulation studies. ›Serum progesterone. ›Serum ferritin test. ›Transvaginal ultrasonography (TVS). ›Endometrial biopsy. ›Hysteroscopy. ›Magnetic resonance imaging (MRI). ›Computerized tomography (CT) scan.

Abortus or "A"

■ Abortus or "A": The number of pregnancies that were lost more than 20 weeks (includes both induced and spontaneous abortions).

Exam Tips

■ Always treat asymptomatic bacteriuria and UTIs in pregnant women. ■ UTIs in pregnant women are classifi ed as "complicated UTIs." ■ UTI is defined as 100,000 colony forming units (CFU) or 105. ■ Signs and symptoms of UTI in pregnant women are the same as in nonpregnant state. ■ Amoxicillin is not first line for empiric UTI treatment (high resistance rates). ■ Use nitrofurantoin to treat UTI in pregnant women on the exams. ■ There are usually only a few questions that address pregnancy. Do not overstudy this subject area for the exam because the number of questions has decreased over the past few years.

Coombs test

■ Coombs test: detects presence of Rh antibodies in the mother (indirect Coombs test) and the infant (direct Coombs test). This test is done as part of the labs done early in pregnancy. ■ RhoGAM 300 mcg IM fi rst dose is at 28 weeks. ■ Give second dose within 72 hours (or sooner) after delivery. ■ RhoGAM decreases the risk of isoimmunization of the maternal immune system by destroying fetal Rh-positive RBCs that have crossed the placenta.

Exam Tips

■ Do not use the 50-g OGTT test for screening. Use the 75-g OGTT. ■ 75-g OGTT is now used both for screening and diagnosis of GDM (ADA, 2012). ■ First-line treatment for GDM is lifestyle (correct diet and scheduled exercises). ■ None of the oral type-2 antidiabetic oral drugs is approved for use in pregnancy (FDA and ADA, 2012). ■ Learn the risk factors for GDM

Probable Signs of Pregnancy

■ Goodell's sign (4 weeks): cervical softening. ■ Chadwick's sign (6-8 weeks): blue coloration of the cervix and vagina. ■ Hegar's sign (6-8 weeks): softening uterine isthmus. ■ Enlarged uterus. ■ Ballottement (seen in mid-pregnancy); when the fetus is pushed, it can be felt to bounce back by tapping the palpating fingers inside the vagina. ■ Urine or blood pregnancy tests (beta hCG). ■ "Quickening": the mother feels the baby's movements for the first time. Starts at 16 weeks.

Gravida or "G"

■ Gravida or "G": The number of pregnancies of the mother (no matter what the outcome).

Use of Antihypertensives With Pregnant Women Used for women with preexisting hypertension or for moderate-to-severe preeclampsia or eclampsia.

■ Methyldopa (Aldomet) ■ Hydralazine (Apresoline) ■ Labetalol (Normodyne)

Weight Gain

■ Most weight gained in third trimester (about 1-2 lbs per week). ■ Best weight gain: 25 to 35 lbs if healthy weight before pregnancy (body mass index [BMI] of 18.5-24.9). ■ For underweight patients: gain 28 to 40 lbs (BMI <18.5). ■ For obese patients: gain up to 11 to 20 lbs (BMI >30). ■ After delivery: will lose up to 15 to 20 lbs in fi rst few weeks. ■ Twins: weight gain increases (37-54 lbs), but it is not doubled.

Live Vaccines

■ Mumps, measles, and rubella (MMR), oral polio, varicella, FluMist contraindicated in pregnancy. ■ Flu vaccine is inactivated virus and is safe to use in pregnant women. - Recommend for pregnant women especially if they are pregnant during the fall and winter seasons. Only use the injectable flu vaccine (no FluMist). - Live attenuated infl uenza virus vaccine (LAIV) fl u vaccine by nasal spray (FluMist) - Only for healthy nonpregnant persons age 2 months to 49 years with no nasal abnormalities. Avoid use if person has close contact with immunocompromised or with pregnant women. *After a live virus vaccine, advise reproductive-age women not to get pregnant (and use reliable birth control) in the next 4 weeks (MMR) or 3 months (varicella vaccine).

Nitrofurantoin and Sulfa Drugs

■ Nitrofurantoin and sulfa drugs should be avoided near term (38-42 weeks), during labor, and during delivery. It is also contraindicated in neonates less than 4 weeks of age. ■ These drugs increase the risk of hyperbilirubinemia (immature liver). ■ Complication of hyperbilirubinemia is called "kernicterus" (serious nerve/brain damage). ■ Do not use if suspect G6PD anemia (causes hemolysis). ■ Nitrofurantoin: Serious adverse effects. ■ Pulmonary reactions (interstitial pneumonitis, pulmonary fibrosis). ■ Hepatoxicity, neuropathy, and others.

Exam Tips

■ Palpation of fetal movements by the mother is not considered a positive sign of pregnancy (quickening). It is classifi ed as a "probable sign." ■ Memorize the three "last names" of probable signs. ■ Urine/serum pregnancy tests are considered probable signs (do not confuse as a "positive sign"). Beta hCG presents also in molar pregnancy and ovarian cancer. ■ Questions asking for one of the signs will mix them up (mix a positive sign with a probable sign). Make sure the answer option contains the two signs from the same category. ■ Memorize the three positive signs of pregnancy. It is the shortest list. By the process of elimination, you can rule out (or in) the correct answer choice.

Para or "P"

■ Para or "P": The number of births more than 20 weeks no matter what the outcome (including miscarriages, stillborn). Twins and multiples are counted as one pregnancy.

HEALTH EDUCATION

■ Prenatal vitamins with 400 mcg of folic acid daily (start 3 months before conception). ■ Always wear seatbelt (lap belt below uterine fundus). ■ Avoid soft cheeses (blue cheese, brie), uncooked meats, raw milk (listeria bacteria). ■ Sex is safe except during vaginal bleeding, incompetent cervix, placenta previa, or preterm labor. ■ Cat litter or raw beef can cause toxoplasmosis (congenital infection). ■ No raw shellfi sh or raw oysters (vibrio vulnifi cus infection). ■ Careful with cold cuts, uncooked hotdogs, and deli meat (listeria bacteria). Pregnant women are 20 times more likely to become infected with listeria monocytogenes. ■ Smoking (IUGR), alcohol (fetal alcohol syndrome). ■ Regular coffee (8 ounces/day) is okay. No excessive caffeine (premature labor). ■ No hot tubs, saunas, or excessive heat.

Exam Tips

■ Swallowing noises may be heard in breastfeeding, especially in younger babies. ■ If clicking noises are heard during breastfeeding, it is abnormal. Advise the mother to use her index finger to pull down the baby's chin so that the baby's lower lip will be outside the mouth. ■ When there is proper latch-on, there should be no noise present when breastfeeding. ■ If a question describes a mother who complains of sore nipples, do not advise her to stop breastfeeding, to supplement with formula, or to use formula at nighttime feedings. The best answer is to advise the mother that it is a common problem and will resolve.

Exam Tips

■ There is usually one question asking about the EDD (use Naegele's rule). ■ The LMP month on the exam will either be: January (01), February (02), or March (03). If LMP is: Expected EDD: January ----> October February ----> November March ----> December ■ Heart murmur in pregnancy occurs during systolic (systolic flow murmur). ■ Chloasma/melasma is due to high estrogen level. ■ Placenta previa is vaginal bleeding (bright red) without a hypertonic tender uterus. ■ Placenta abruptio is vaginal bleeding that is intermittent with hypertonic, hard and tender uterus. ■ Fundi at 12 weeks is above symphysis pubis ■ Fundi at 16 weeks is between the symphisis pubis and the umbilicus. ■ Fundi at 20 week is at the umbilicus ■ Askenazi Jews (European Jews) should be screened for Tay-Sachs disease.

Uncomplicated Chlamydia Infection (Cervicitis, Urethritis)

■ Treating Chlamydia trachomatis infection in the mother will help to prevent the transmission (vertical transmission) of the infection to the newborn through the birth canal. - Example: conjunctivitis (or inclusion conjunctivitis of the newborn) and pneumonia. Labs ■ Nucleic acid amplifi cation tests (NAAT) such as the GenProbe, Amplicor, or ProbeTec. ■ GenProbe can only be used on the cervix and urethra. Do not use to collect specimens from the eyes. ■ Test of cure in 3 weeks after completing treatment. Treatment Plan ■ First line: Azithromycin 1 g orally (single dose). ■ Alternative is amoxicillin 500 mg PO TID × 7 days (lower cure rate than azithromycin). Sexual Partners ■ First line: Azithromycin 1 g orally (single dose). ■ Doxycycline 100 mg PO BID × 7 days. ■ Avoid sexual activity for 7 days. Avoid unprotected intercourse until both partners are treated. Test for other STDs (gonorrhea, syphilis, HIV).

Fundal Heights: 20 Weeks (Fifth Month)

■ Uterine fundus at level of the umbcus. ■ FHT heard with fetoscope or stethoscope by 20 weeks.

Fundal Heights: 12 Weeks (Third Month)

■ Uterine fundus first rises above symphysis pubis (uterus is the size of a grapefruit). ■ FHT heard by Doppler by 10 to 12 weeks.


Kaugnay na mga set ng pag-aaral

Current Issues 4: What is RAID 0, 1, 5, & 10?

View Set

What is another name for a client operating system?

View Set